Health Assessment Final Exam

Ace your homework & exams now with Quizwiz!

The nurse is evaluating a new​ nurse's ability to recognize the different types of fever in clients. Which statements by the new nurse indicates accurate ​knowledge? Select all that apply. ​"A relapsing fever is when the body has short febrile periods of a few days that are interspersed with periods of normal body​ temperature." ​"The body can have a constant fever when the temperature remains​ elevated." ​"Intermittent fever occurs when the body goes through a wide range of body temperatures throughout a​ 24-hour period." ​"A fever spike is when there is a rapid rise in body temperature with a return to normal temperature within a couple of​ hours." ​"Remittent fever is when the body temperature alternates between normal body temperature and elevated​ temperature."

"A relapsing fever is when the body has short febrile periods of a few days that are interspersed with periods of normal body​ temperature." ​"The body can have a constant fever when the temperature remains​ elevated." ​"A fever spike is when there is a rapid rise in body temperature with a return to normal temperature within a couple of​ hours." Relapsing fevers are characterized by short febrile periods of a few days that are interspersed with periods of 1 or 2 days with normal body temperature. Constant fever is when the body temperature remains above normal with minimal fluctuations in temperature. A fever spike is a rapid rise to a fever with a return to normal body temperature within a few hours. Remittent fever is when the body has a wide range of body temperatures throughout a​ 24-hour period. All temperature fluctuations are above normal range​ (more than 3.6​°F ​[2​°​C]). An intermittent fever is where the body temperature alternates at regular intervals between normal body temperature and periods of raised body temperature.

What questions would you ask a client to rule out ​infection? Select all that apply. "How long does your menstrual cycle ​last?" "Are you experiencing any discharge from the ​urethra?" "Do you take bubble ​baths?" "Have you had unexplained weight ​loss?" "Are you having trouble​ concentrating, reading, or ​remembering?"

"Are you experiencing any discharge from the ​urethra?" "Do you take bubble ​baths?" "Are you having trouble​ concentrating, reading, or ​remembering?" Discharge from the urethra indicates a likelihood of an infection in progress. Difficulty​ concentrating, reading, or remembering may indicate a urinary infection or renal dysfunction. Bubble baths can cause urinary problems or infections. Determining how long an​ individual's menstrual cycle lasts will not help determine if an infection is present. Unexplained weight loss may indicate cancer.

Which of the following are internal factors that contribute to the​ client's risk for breast​ disease? Select all that apply. ​"Describe your weight from childhood to​ now." ​"Do you work at​ night?" ​"Did you breastfeed your​ children?" ​"Do you have breast​ implants?" ​"Have you been exposed to excessive radiation from repeated​ x-rays?"

"Describe your weight from childhood to​ now." ​"Did you breastfeed your​ children?" ​"Do you have breast​ implants?" Internal questions refer to internal or physiological responses such​ as, "Did you breastfeed your​ children?" (breastfeeding for at least a year reduces the risk of breast​ cancer); "Do you have breast​ implants?" (breast implants do not increase breast cancer risk but can decrease the visualization of breast​ tissue); and​ "Describe your weight from childhood to​ now" (obesity is a predisposing factor for breast​ cancer). Questions about radiation exposure and working at night are associated with a​ person's external environment. Excessive exposure to​ radiation, like that from frequent​ x-rays, increases a​ person's risk of breast cancer. Working at night has been linked to breast​ cancer, which may be related to disruption of​ melatonin, a hormone affected by light.

An affirmative response to what question may indicate that the client has a mass or​ tumor? "Did you start menses before age ​11?" "Are you able to completely empty your bladder with each ​urination?" "Do you have difficulty starting the flow of ​urine?" "Do you get up during the night to ​urinate?"

"Do you have difficulty starting the flow of ​urine?" Difficulty starting the flow of urine may indicate that the client has a mass or tumor. There is no direct correlation between age of onset of menses and the development of a mass or tumor. Whether or not a client gets up during the night to urinate is influenced by heart or kidney disease or the amount and timing of the ingestion of fluids. When an individual is unable to completely empty the bladder during​ urination, urinary stasis can lead to urinary tract infections.

Which question focuses on a serious symptom of the male genitourinary​ system? ​"Do you have a change in skin color on the penis or​ scrotum?" ​"Do you do testicular​ self-exams?" ​"Do you have any itching around the​ penis?" ​"Do you have​ bloody, foaming, or dark​ urine?"

"Do you have​ bloody, foaming, or dark​ urine?" Hematuria is a serious symptom and needs to be followed up by a specialized health care provider immediately. Itching of the penis may indicate a reaction to soaps or lotions and is not a serious symptom of the genitourinary system. Testicular​ self-examinations provide early detection of a lump or​ mass, but that is not as emergent as hematuria. A change in skin color of the penis or scrotum may indicate​ inflammation, but is not as serious a symptom as​ bloody, foaming, or dark urine.

What interview questions focus on issues that may affect the genitourinary health of an adult male ​client? Select all that apply. ​"Do you have​ diabetes?" ​"Have you been exposed to​ HIV?" ​"Have you had urinary tract​ infections?" ​"How many children do you​ have?" ​"Do you have high blood​ pressure?"

"Do you have​ diabetes?" ​"Have you been exposed to​ HIV?" ​"Have you had urinary tract​ infections?" ​"Do you have high blood​ pressure?" Asking about exposure to HIV opens the discussion about risky behaviors that may affect the genitourinary health of an adult male client. Hypertension can contribute to renal problems. Diabetes can contribute to renal problems and renal failure. Knowing about past urinary tract infections helps to determine ongoing difficulties with the urinary tract. The number of children a client has is not directly related to genitourinary​ health; it is more important when assessing infertility.

What questions are relevant to ask regarding the neurologic system of an infant or​ child? Select all that apply. "Is the child an only ​child?" "Has the child had​ fevers, seizures, or frequent ​falls?" "Did you take​ medications, alcohol, or drugs while ​pregnant?" "Are there any surfaces in your home painted with​ lead-based ​paint?" "Did you have any health problems when pregnant with the ​child?"

"Has the child had​ fevers, seizures, or frequent ​falls?" "Did you take​ medications, alcohol, or drugs while ​pregnant?" "Are there any surfaces in your home painted with​ lead-based ​paint?" "Did you have any health problems when pregnant with the ​child?" Symptoms or health problems during pregnancy could be early signs of an illness or disease process. The use of some​ medications, alcohol, or drugs while pregnant can cause neurologic problems in children. Lead ingested by children can cause developmental​ delays, brain​ damage, and peripheral nerve damage.​ Fevers, seizures, frequent​ falls, and signs of clumsiness may indicate neurologic illness. Whether a child has siblings or not is not relevant to the child​'s neurologic assessment.

What question would assess the possibility of a current urinary tract​ infection? ​"Do you have blisters or lumps in the genital​ area?" ​"Can you recall your diet for the past 24​ hours?" ​"Have there been any skin color changes in the urethral​ area?" ​"Has there been any change in the quality of urine in terms of​ color, odor, or pattern of​ urination?"

"Has there been any change in the quality of urine in terms of​ color, odor, or pattern of​ urination?" Changes in the quality of urine may indicate an infection of the urinary tract. A change in color of the skin in the urethral area can indicate inflammation but does not focus on urinary tract infection symptoms. The client​'s diet in the past 24 hours provides nutritional and hydration information but does not provide information about current urinary symptoms. Blisters or lumps in the genital area may indicate a variety of infections or conditions but are not directly related to urinary symptoms.

What question is related to the client​'s external​ environment? "Are there foods that you ​avoid?" "Have you ever been exposed to ​pesticides?" "What do you commonly ​eat?" "What medications are you ​taking?"

"Have you ever been exposed to ​pesticides?" Pesticide exposure is linked to the client​'s external environment and is relevant because pesticides can contribute to neurologic illnesses and disease. Foods that the client eats and avoids relate to the internal environment and are relevant because a balanced diet is necessary for health and​ well-being. Medications taken by the client are part of the internal environment and are relevant because some medications cause neurologic problems as a side effect.

Mr. Mangano is a​ 41-year-old client who has come into the clinic with urinary problems. What response to your interview questions may indicate the​ cause? ​"I use recreational​ drugs." ​"I have been​ circumcised." ​"I do not perform testicular​ self-exams regularly." ​"I have not had previous reproductive system​ problems."

"I use recreational​ drugs." Urinary problems that result in a decrease in libido and may contribute to​ hypertension, dehydration, and kidney failure can occur from substance abuse. Circumcision would not cause urinary problems. Testicular​ self-exams are helpful in detecting a mass or lump but are not relevant to the​ client's urinary problems. Reproductive system problems such as​ herpes, chlamydia, or other infections would provide further​ information, but would not cause urinary problems.

The nurse is assessing the thoracic cavity of a child. Which statement demonstrates that the nurse understands the anatomy of the​ chest? ​"The thoracic cavity of a child can only be auscultated from the​ front." ​"I will palpate the ribs​ anteriorly." ​"I will​ inspect, auscultate, and palpate the thoracic cavity from the base of the neck to the​ diaphragm." ​"The posterior ribs are the best place to assess the thoracic cavity of a​ child."

"I will​ inspect, auscultate, and palpate the thoracic cavity from the base of the neck to the​ diaphragm." The thoracic cavity of a child or an adult is assessed from the base of the neck down to the​ diaphragm, both front and​ back, by​ percussion, inspection, and auscultation. The size of the child does not change the description of the anatomy of the thoracic cavity.

The nurse is caring for an adult client with a heart rate of 60 beats per minute. What statements indicate that the nurse understands the electrical conduction system of the ​heart? Select all that apply. ​"The ventricles do not receive an electrical​ impulse." ​"The SA node is the initial electrical​ signal." ​"An obstruction can occur in the bundle of​ His." ​"The bundle of His is the pathway for the electrical impulses to travel to both​ ventricles." ​"The electrical impulse travels from the SA node directly to the bundle of​ His."

"The SA node is the initial electrical​ signal." ​"An obstruction can occur in the bundle of​ His." ​"The bundle of His is the pathway for the electrical impulses to travel to both​ ventricles." The SA node is the initial electrical impulse that signals the heart to contract. The signal goes through the AV node and then on to the bundle of​ His, where there are pathways that propagate the signal down and around the ventricles causing them to contract. A blockage or obstruction in the bundle of His can result in a low heart​ rate, which can be pathological.

The nurse is working in an acute care emergency room setting. What statement describes the purpose of triage in any emergency disaster​ situation? ​"Triage is organized by each facility and its​ policies." ​"Triage prioritizes clients in the order of the severity of​ needs." ​"Triage offers a way to treat the least injured first up to the most critical​ clients." ​"Triage is only used in a​ disaster."

"Triage prioritizes clients in the order of the severity of​ needs." Triage is a universal system that prioritizes the care of clients for the most efficient care according to the needs of the clients. Triage is used in any situation where more than one client is being cared for. It is a universal system used by​ EMS, by emergency​ rooms, and in internal and external disasters. Triage is a universal system where the care is prioritized from the most critical to the least critical clients for efficient care and is not unique to an individual facility.

The nurse is evaluating a new​ graduate's ability to assess a radial pulse. Which statement by the new nurse indicates the need for further​ teaching? ​"The number of pulsations felt per minute is called the heart​ rate." ​"The pulse is the reflection of the amount of blood ejected with each​ heartbeat." ​"I will need to determine the rate and rhythm of the​ pulse." ​"When checking the pulse I will measure the elasticity using the scale​ 0, 1+,​ 2+, 3+, or​ 4+."

"When checking the pulse I will measure the elasticity using the scale​ 0, 1+,​ 2+, 3+, or​ 4+." The scale​ 0, 1+,​ 2+, 3+, or​ 4+ is used to measure the amplitude of the client​'s pulse. Elasticity is not measured on the amplitude scale. While assessing the pulse the nurse assesses the​ rate, rhythm,​ amplitude, and elasticity of the artery. The pulse is the reflection of the amount of blood ejected with each heartbeat. The number of pulsing sensations felt each minute is the heart rate.

The nurse is assessing the abdomen of a child who complains of pain. What question would be important to ask parents when assessing the possible causes of abdominal​ pain? ​"Is the father of the child​ available?" ​"When was the last time the child had a bowel​ movement?" ​"What type of insurance is the child covered​ under?" ​"Who is the child​'s ​pediatrician?"

"When was the last time the child had a bowel​ movement?" Establishing the bowel habits of the child would help to assess the state of the digestive system and possible constipation. It may be important later to determine who the child​'s pediatrician​ is, what type of insurance the child is covered​ under, or whether the child​'s father is​ available, but when assessing the abdomen for​ pain, questions should be focused on the​ abdomen, diet, bowel​ movements, and any other medical information to assess the abdominal pain the child is having.

When measuring the amplitude of a client​'s ​pulse, what is the scale indication for a weak or barely palpable​ pulse? ​0+ ​3+ ​2+ ​1+

1+ A diminished pulse is scored as​ 1+. A​ 0+ pulse indicates a nonpalpable pulse. A normal pulse is​ 2+, and a bounding pulse is​ 3+

The nurse has just completed a Glasgow Coma Scale test on four clients. The nurse would be most concerned about the client with what Glasgow​ score? 10 6 3 15

3 The Glasgow Coma Scale ranges from 3 to​ 15, with 3 being the lowest score or the client who is most compromised. The client with the lowest​ score, in this case​ 3, would be the priority for the nurse.

You are educating a client about how to monitor his blood pressure at home. You describe the sounds that the client will hear as he takes his blood pressure. How many sounds will be heard throughout this​ process? 5 3 4 6

5 Blood pressure sounds​ (also called Korotkoff​ sounds) change over the duration of the blood pressure monitoring procedure. The five phases are described as​ follows: 1. Initial sounds are faint with a clear tapping quality. 1. Distinctive swishing sounds are heard. 1. Similar tapping sounds as Phase 1 yet with greater intensity. 1. Muffled sounds ensue. 1. The sounds disappear.

You would expect which pulse rate range when assessing the pulse of an​ 82-year-old client? 60-100 ​(beats/min) 50-110 ​(beats/min) 80-120 ​(beats/min) 50-70 ​(beats/min)

60-100 (beats/min) An older adult​'s expected pulse rate is between 60 and 100 beats per minute. A pulse of 50dash-60 beats per minute may be too slow for an older person and may result from possible medication side effects. A pulse rate greater than 100 beats per minute is too fast. Pulse rates that are too slow or too fast should be further investigated for cause.

The cardiac cycle for an adult takes about eight seconds. How many cycles would the nurse expect to palpate and auscultate in one minute during the cardiovascular assessment of an adult​ client? 42 72 226 122

72 The average heart rate for an adult is 72 beats per minute. The normal range for an adult is 60dash100 beats per minute. As the nurse assesses cardiovascular health for an adult​ client, it is important to recognize normal signs of cardiac function.

How many pairs of spinal nerves are at the cervical spinal​ level? 5 8 1 12

8 There are 8 pairs of spinal nerves at the cervical spinal level. There are 12 pairs of spinal nerves at the thoracic spinal​ level, 5 pairs at the lumbar and sacral spinal​ levels, and 1 pair at the coccygeal spinal level.

What is the expected oral temperature range of a healthy​ adult? 96.7°F-98.5°F 98.7°F-100.5°F 97.7°F-99.5°F 95.7°F-97.7°F

97.7°F-99.5°F

What male anatomic structure serves as a duct for the transportation of sperm and as a reservoir for mature​ sperm? Select the correct answer choice below. A ductus deferens B prostate C urethra or corpus spongiosum D teste

A

The nurse is assessing the hearing of an adult client. What findings during the nursing exam could indicate that the client is experiencing hearing​ loss? Select all that apply. A client hears better out of one ear than the other. The client can repeat words and clearly pronounces them in the order heard. The eardrum is translucent. The client is unable to repeat phrases the nurse whispers. A perforated eardrum is obvious on exam.

A client hears better out of one ear than the other. The client is unable to repeat phrases the nurse whispers. A perforated eardrum is obvious on exam. In the whisper​ test, the nurse whispers into one of the client​'s ​ears; if the client has difficulty repeating the statement word for​ word, there may be hearing loss. When the client hears sound better in one ear than the​ other, as determined by the Weber​ test, there may be hearing loss. A perforated eardrum would lessen the ability to hear. A translucent eardrum is normal. A client repeating exactly what is heard is a normal finding.

The nurse is caring for an adult client with an inferior wall myocardial infarction. What type of cardiovascular injury is a myocardial​ infarction? Congenital heart disease A disease or injury to the myocardium Valvular disease Electrical rhythm disturbance

A disease or injury to the myocardium An inferior wall myocardial infarction is an acute injury to the myocardium or heart muscle. Valvular disease involves the valves of the heart and not the myocardium. Electrical rhythm disturbances may be the result of the injury to the area of the​ myocardium, but a myocardial infarction is considered a direct injury to the muscle itself. Congenital heart disease is the result of injury or disease from birth.

Which assessment findings require further nutritional​ evaluation? Select all that apply. A​ 16-year-old client who is 68 inches tall and weighs 140 pounds A female client with no weight change at 6 months of pregnancy A​ 36-year-old client with a history of diabetes A​ 75-year-old female client with complaints of swollen and bleeding gums An​ 18-year-old male with a body mass index​ (BMI) in the 75th percentile after adjusting for gender and age

A female client with no weight change at 6 months of pregnancy A​ 36-year-old client with a history of diabetes A​ 75-year-old female client with complaints of swollen and bleeding gums A pregnant client is generally expected to gain 25dash-35 pounds over the course of the entire pregnancy. An adolescent would have symptoms of overnutrition if the BMI was greater than the 95th percentile after adjusting for gender and​ age; the 75th percentile would be appropriate. Any client with a history of diabetes requires a nutritional evaluation. In elderly​ clients, undernutrition is often not diagnosed. Swollen and bleeding gums may indicate a deficiency of vitamin C. A​ 16-year-old client who is 68 inches tall and weighs 140 pounds has a body mass index of​ 22, which is considered a healthy weight.​ Therefore, further nutritional evaluation is not required for this client.

The nurse is assessing an adult client with hypertension for the first time. What would be included in a complete cardiovascular assessment for this ​client? Select all that apply. A history of drug use A review of cultural beliefs and practices A physical exam including heart rate and blood pressure The insurance coverage for the client A complete medical history including family history of heart disease

A history of drug use A review of cultural beliefs and practices A physical exam including heart rate and blood pressure A complete medical history including family history of heart disease A complete​ history, a physical​ exam, assessing cultural beliefs and​ customs, and knowing the drug history of a client would give a better picture of the cardiovascular health of the client. Knowing what insurance coverage a client has may be important when deciding what type of care to provide later but does not influence cardiac health.

The nurse is working on a safety and quality assurance committee for employee safety and is reviewing product information. What would be an example of a product that ensures provider safety when caring for a​ client? A needleless system TB tests for employees Paperless documentation Brightly colored scrubs

A needleless system A needleless system is an example of safety equipment that provides a way of administering medications and intravenous fluids without the risk of needle stick injuries for employees. Brightly colored scrubs may be a facility​ choice, but is not a product that the committee would recommend to prevent provider injury according to OSHA standards. TB testing does not protect the employee from exposure to TB. It only screens for a transmission that has already happened. Paperless documentation is a safety procedure for client​ privacy, but may not be reviewed by the committee to provide equipment for safety of the employee.

The nurse is preparing to assess the client​'s vital signs. Which principles would the nurse ​consider? Select all that apply. The client should be told that the nurse will count their respirations before the nurse begins. A pulse oximeter is a device used to measure the oxygen saturation of the blood. Respirations should be relaxed and regular. The client should be allowed to rest for 15 minutes before the blood pressure is taken. A Doppler can be used to assess a pulse that is not palpable.

A pulse oximeter is a device used to measure the oxygen saturation of the blood. Respirations should be relaxed and regular. A Doppler can be used to assess a pulse that is not palpable. The client​'s respirations should be​ relaxed, regular,​ automatic, and silent. To assess​ respirations, count the number of respirations per minute. The client should not be made aware that the nurse will count their respirations because the data obtained will be incorrect. Using a pulse oximeter is a noninvasive technique to measure the oxygen saturation of the blood. Oxygen saturation is the percentage that oxygen fills hemoglobin. When the nurse is unable to palpate a peripheral​ pulse, the Doppler ultrasound technique can be used. A Doppler is an ultrasonic device that detects the sound of moving blood in the artery.​ To get an accurate measurement of the client​'s blood​ pressure, the client should be at rest for 5 minutes before the blood pressure is assessed.

The nurse is palpating the jaw of a client and asks the client to open and close his mouth. What would the nurse suspect when palpation reveals crepitus during the​ exam? An enlarged thyroid Inflammation of the​ temporal artery A normal finding in an adult A temporomandibular joint abnormality

A temporomandibular joint abnormality Crepitus or crackling when palpating the jaw may indicate a joint problem. This is an abnormal finding in an​ adult; the joints should open and close without crackling or crepitus. An enlarged thyroid would not affect the temporomandibular joints. Inflammation of the temporal artery would not be palpated over the temporomandibular joint.

The nurse is assessing an older adult client. What signs of aging would be apparent when assessing the mouth of this​ client? Increased vascularity of the lips A tongue with fissures and impairment of the function of the tongue ​Pink, moist, smooth tongue Pink gums

A tongue with fissures and impairment of the function of the tongue Fissures of the tongue are abnormal but are an expected finding in an older adult client. Older adults also often have some form of impairment of the ability to move the tongue​ rapidly, which is abnormal but expected in an older adult. The older adult client experiences a decrease in vascularity of the lips as well as the​ gums, so gums may be pale. The tongue of an older client may be dry and cracked due to a decrease in saliva​ production, which may result from atrophy of salivary glands or side effects of medications.

Which describes a remittent​ fever? A wide range of temperature fluctuations within a​ 24-hour period, all above normal ranges Short febrile periods of a few days mixed with periods of 1 or 2 days of normal temperatures A rapid rise to a fever with return to normal body temperature in a short time Body temperature fluctuates at regular intervals from normal to above normal temperatures

A wide range of temperature fluctuations within a​ 24-hour period, all above normal ranges Remittent fever is described as a wide range of temperature fluctuations within a​ 24-hour period, all above normal ranges. A relapsing fever is described as short febrile periods of a few​ days, mixed with periods of 1 or 2 days of normal temperatures. An intermittent fever is described as body temperature fluctuating at regular intervals from normal to above normal temperatures. A fever spike is a rapid rise to a fever with return to normal body temperature in a short time.

What would be normal findings on inspection of the abdomen in an adult male ​client? Select all that apply. Abdomen is mottled and slightly paler compared to arms and legs. Abdomen is symmetrical. Abdomen is rounded. Abdomen shows outline of the colon and normal gas pattern. Client looks rested and relaxed.

Abdomen is symmetrical. Abdomen is rounded. Client looks rested and relaxed. The adult male client would appear rested and​ relaxed, and not intense or ill. A normal abdomen is​ rounded, not​ distended, and symmetrical without lumps or​ protrusions, and the client would not complain of pain. The abdomen would not be mottled or​ pale; it would have the same general skin tone as the arms and legs. The abdomen would be smooth and rounded without showing an intestinal outline or gas pattern.

When physically inspecting a client​'s neurologic​ system, what does asking the client to identify the similarities and differences between a shoe and a jacket​ assess? Ability to make judgments Abstract thinking ability Mood and body language Memory

Abstract thinking ability By asking the client to identify the similarities and differences between a shoe and a​ jacket, you are assessing the client​'s ability to think abstractly about simple similarities and differences. When testing​ memory, questions about the name of​ children, the client​'s birth​ date, and significant events are employed. When assessing the ability to make​ judgments, the client should be able to discuss​ his/her health,​ family, and future plans without emotional impairment. Mood and body language are observed for appropriateness of facial expressions and body language to questions asked.

Mrs. Ramirez is 7 months pregnant and has come to the clinic complaining of heartburn and chest pain after eating. What is the most likely​ cause? Acid indigestion Gallstones Appendicitis Intestinal obstruction

Acid indigestion Pregnancy causes decreased peristalsis and acid indigestion is common due to the pressure of the baby on the stomach. A client with an intestinal obstruction would present with aching or spasmodic abdominal pain. Appendicitis is characterized by the sudden onset of epigastric and periumbilical pain localized to the right lower quadrant. Gallstones would be indicated if there is pain when palpating the liver.

Mr. Schulte is an elderly male with a history of chronic pain due to rheumatoid arthritis and osteoarthritis who has an appointment at your Outpatient Rheumatology Service clinic. Which of the following findings would​ you, the clinic​ nurse, expect during your routine pain assessment of this​ gentleman? Acknowledgment of pain only if asked directly Restlessness and anxiety Elevated blood pressure and pulse Rubbing of affected area with facial grimacing

Acknowledgment of pain only if asked directly It is important to directly question the client about pain signs and symptoms. Most clients with chronic pain may not exhibit overt signs of​ pain, such as​ restlessness, anxiety, and outward displays of pain. These clients may typically keep symptoms of pain to themselves and subsequently may be depressed and withdrawn. Changes in vital signs are not reliable indicators of pain in either acute or chronic pain conditions.

The nurse is discussing safety and prevention with the parent of a toddler. What factor would be the basis for instructing the parents to turn down the temperature on a hot water heater for the prevention of​ burns? Sexual orientation Lifestyle Age and development Health status

Age and development Age and development influence safety and the prevention of accidents. A toddler is not able to understand the consequences of hot water and may turn on a hot water​ faucet, causing burns. It would be important to educate the parents of a toddler about appropriate safety. Lifestyle does influence​ safety, but in this case age and development are the keys to education for parents. Sexual orientation has no bearing on the safety of a toddler around hot water. Health status does influence the safety of a​ client, but in the case of a​ toddler, regardless of health​ status, it would be most important to recognize age and development when educating parents.

Ms. Link is a young adult who came to your clinic for a routine health examination. Upon percussing her lung​ fields, she​ asks, "What causes the sounds that you are listening​ for?" Based on the underlying physiology of lung​ tissue, which information should you include in your answer to her​ question? Trapped air causes a​ low-pitched, soft tone. Air in the underlying area of percussion is responsible for the tones that are heard. The dense tissue produces a softer and longer tone. Less dense tissue produces a softer and shorter​ tone, such as with lung sounds.

Air in the underlying area of percussion is responsible for the tones that are heard. Air in the percussed structure causes the tones. Lung sounds over a dense area produce a​ resonance, which is loud and low​ pitched, not softer and longer in tone. Trapped air causes an abnormally​ loud, low tone. Dense tissue produces a softer and shorter tone.

What sounds are heard during the common assessments requiring​ auscultation? Select all that apply. Tympany during assessment of the stomach. Air moving in and out of the lungs during lung assessment. Peristalsis during assessment of the abdomen. Sounds correlated with the pulse during blood pressure assessment. Opening of the heart valves during heart assessment.

Air moving in and out of the lungs during lung assessment. Peristalsis during assessment of the abdomen. Sounds correlated with the pulse during blood pressure assessment. Sounds correlated with the pulse are heard during blood pressure​ assessment; the sounds of peristalsis are heard during assessment of the​ abdomen; and air is heard moving in and out of the lungs during lung assessment. The​ lub-dub sound of the closing of the heart valves is heard during heart assessment. Tympany is the characteristic sound of an organ filled with air that is heard during​ percussion, not auscultation.

The nurse is completing a nutritional assessment for a client in the clinic. Which factor would the nurse identify as a risk for both undernutrition and​ overnutrition? Poor dental health Alcohol abuse Sedentary lifestyle Extreme age

Alcohol abuse Alcohol abuse places the client at risk for both​ under- and​ overnutrition, and a sedentary lifestyle places the client at risk for​ overnutrition, but not for undernutrition. Poor dental health and age can place the client at risk for​ undernutrition, but not overnutrition.

Which of the following conditions are prevalent among Native​ Americans? Select all that apply. Alcohol and drug abuse Gallstones and gallbladder cancer Hepatitis C H.pylori related peptic ulcers Lactose intolerance

Alcohol and drug abuse Gallstones and gallbladder cancer Lactose intolerance Alcohol and drug​ abuse, gallstones, gallbladder​ cancer, and lactose intolerance are prevalent among Native Americans. Alcohol and drug abuse as well as lactose intolerance are also prevalent among African Americans. Gallstones and gallbladder cancer is also prevalent among Mexican Americans. Hepatitis C is more common among African Americans and Hispanics. Lactose intolerance is more common among Caucasian Americans and Jewish Americans.​ H.pylori relatedpeptic ulcers and is prevalent among Hispanics and African Americans. Next Question

Identify the nutritional assessment tool that helps to collect a​ client's nutritional history. ​Instructions: Use the dropdown menus in the left column to select the nutritional assessment tool for each description in the right column. Tool Description 24 - hour dietary recall Food frequency questionnaire Food record All​ food, drink, nutrition supplements consumed in one day Assesses a variety of food groups over a daily or weekly time period ​3-day food​ diary: 2 weekdays and 1 weekend day

All​ food, drink, nutrition supplements consumed in one day- 24 - hour dietary recall Assesses a variety of food groups over a daily or weekly time period- Food frequency questionnaire ​3-day food​ diary: 2 weekdays and 1 weekend day- Food record

The nurse​'s assessment of the head of an​ 8-year-old child reveals​ large, patchy, bald areas. What condition would the nurse suspect in a child with this abnormal​ finding? Acromegaly Enlarged lymphatic tissue Alopecia Psoriasis

Alopecia Alopecia is the term for baldness or bald areas regardless of the age of the client. Psoriasis causes​ patchy, scaly areas of skin that may be on the​ scalp, but not baldness. Acromegaly is the abnormal growth of bones and facial features. Enlarged lymphatic tissue displays as nodes felt at the back of the skull and would not cause baldness.

The nurse is assessing the ears of an infant. What is the appropriate equipment to use when inspecting the middle and inner ear of an ​infant? Select all that apply. Your eyes are the tools to use on an infant. Percussion Palpation An ear speculum An otoscope

An ear speculum An otoscope Use the largest ear speculum that fits into the​ infant's auditory canal and an otoscope with a working light to position into the ear canal for direct inspection of the middle and inner ear. Your eyes are used when inspecting the outer​ ear, but the naked eye cannot see into the middle ear and the tympanic membrane. An ear speculum and otoscope are needed to complete the exam. Percussion is not used to visualize the middle and inner ear. Palpation is used before inspection to feel for lymph glands below the ear in the​ neck, but palpation does not aid in inspecting the middle and inner ear.

The nurse is caring for a client who presents to the clinic with diaphoresis. What might cause the client to experience ​diaphoresis? Select all that apply. hypothryroidism An infectious disease Fever diabetes ketoacidosis Side effects of medications

An infectious disease Fever Side effects of medications Increased sweating​ (diaphoresis) may be due to a systemic​ disorder, including an infectious disease with fever. Increased sweating can also be a side effect of some medications. Dry​ skin, not diaphoresis can be a sign of hypothyroidism​ (metabolic disorder). Clients with diabetes ketoacidosis​ (a complication of diabetes​ mellitus) will present with symptoms associated with fluid loss or deficits. Diaphoresis is not a typical symptom associated with diabetes ketoacidosis.

Mr.​ Hoyt, a​ 65-year-old client, had hip replacement surgery 6 hours ago. Which pain management modality do you expect the​ client's healthcare provider will​ prescribe? Nonsteroidal​ anti-inflammatory drug Tricyclic antidepressant Placebo medication An injectable opioid medication

An injectable opioid medication Because Mr. Hoyt had a very recent acute surgical​ intervention, an injectable opioid would be indicated to effectively treat the​ surgery-induced nociceptive pain. An​ anti-inflammatory medication will diminish pain intensity and inflammation from the injury but will not likely adequately address the level of pain expected from his very recent surgery. Tricyclic antidepressants are generally effective for​ long-term management of chronic neuropathic pain and generally take up to two weeks to be effective. The use of tricyclic antidepressant medications in the elderly is discouraged because of its significant​ side-effect profile. It is not ethical to utilize a placebo for either acute or chronic pain unless the client specifically requests one. OK

Mr.​ Thompson, who is 48 years​ old, is admitted to the medical unit with decreased hemoglobin and hematocrit levels. This most likely indicates which​ problem? Increased risk for cardiac disease Protein depletion Anemia or blood loss Bone marrow suppression or lymphocyte destruction

Anemia or blood loss A decreased hemoglobin and hematocrit generally indicates blood loss or anemia. Bone marrow suppression would be noted in a client with decreased total lymphocytes. Protein depletion in a client can indicate liver disease or result from trauma or burns. An elevated total cholesterol level is an indicator of cardiac disease.

What area would the nurse palpate to assess the​ kidneys? Select the correct answer choice below. A T10-11ish B L1-2ish C L5ish D Sacrumish

B

Which pain management approach may be effective in the treatment of moderate neuropathic pain in an​ adult? Anticonvulsant class of medication Opioid class of medication Distraction techniques ​Non-narcotic analgesics

Anticonvulsant class of medication An anticonvulsant may be effective in treating neuropathic pain. An opioid better targets nociceptive pain and is generally not as effective in treating neuropathic pain.​ Non-narcotic analgesics are inadequate in treating moderate to severe pain. Distraction techniques are useful and more effective when used in conjunction with a pharmacological treatment. It is important to remember that individuals may respond differently to treatment modalities​ (i.e., what may work for one person may not work for​ another).

The nurse is assessing a child with a history of asthma. What symptoms would the child with an acute asthma episode likely ​exhibit? Select all that apply. Anxiety Hacking cough Systemic allergic rash Wheezing Clear and regular lung sounds in all fields on auscultation

Anxiety Hacking cough Wheezing A child with asthma can have​ wheezing, cough, decreased respiratory​ effort, decreased lung​ sounds, and anxiety.​ Typically, a child having an acute asthma episode will not have a systemic allergic rash. A child would not have clear lung sounds during an asthma attack but may have little to no air movement during inspiration and exhalation. Asthma is often misdiagnosed because children with asthma may have only a cough that does not let them catch their breath during an episode.

A client who has pain caused by contraction of the psoas muscle may have what​ condition? Aneurysm Appendicitis Cholecystitis Ascites

Appendicitis Pain caused by contraction of the psoas muscle indicates inflammation of the psoas muscle that may be caused by appendicitis. Cholecystitis is indicated when there is pain upon palpation of the liver​ (Murphy sign). A​ large, pulsating aorta found upon palpation is an indication of an aneurysm. Ascites is excess fluid in the peritoneal cavity commonly caused by liver disease.

Upon inspection of the​ neck, the client has pain and the range of motion is limited. What might this​ indicate? Select all that apply. Infection Arthritis Inflammation Vascular disorder Muscle spasm

Arthritis Inflammation Muscle spasm Arthritis, muscle​ spasm, and inflammation are all conditions that are indicated by pain and limited range of motion when inspecting the neck. Infection may be indicated with swelling of lymph​ nodes, and a vascular disorder may be indicated by distention or prominence of the carotid arteries or jugular veins.

While inspecting a client​'s ​abdomen, you note asymmetry. Which of the following is not an indication of this abnormal​ finding? Ascites Stricture Mass Adhesion

Ascites Ascites is fluid in the peritoneal cavity and is not indicated by abdominal asymmetry.​ Masses, adhesions, and strictures are all indicated by asymmetry of the abdomen.

The nurse is assessing the mouth of an adult client. What observation of the physical appearance of the client would be related to cultural​ differences? Asians and Alaska natives have large teeth. The oral mucosa in​ dark-skinned clients may be lighter. Socioeconomic status does not affect the appearance of the structures of the mouth. African Americans have a high incidence of tooth decay.

Asians and Alaska natives have large teeth. Asians and Alaska natives have the largest​ teeth, and the smallest teeth are noted in clients of European descent. Mucous membranes may be a darker pink in​ dark-skinned clients. Poor oral hygiene and dental caries are more prevalent in clients from lower socioeconomic groups. African Americans have the lowest incidence of tooth decay and tooth loss compared with other ethnic groups.

The nurse is preparing to perform an assessment on a​ 32-year-old client. Which interventions would the nurse include in the inspection phase of the ​assessment? Select all that apply. Ask the client​'s permission prior to starting the exam. Perform inspection in a dimly lit room for client​'s comfort. Provide rationale for what is occurring to keep client informed. Ensure adequate exposure of body parts while maintaining privacy. Apply critical thinking skills to analyze data obtained.

Ask the client​'s permission prior to starting the exam. Provide rationale for what is occurring to keep client informed. Ensure adequate exposure of body parts while maintaining privacy. Apply critical thinking skills to analyze data obtained. During inspection the nurse would provide adequate exposure to perform assessment while maintaining client privacy. The nurse would insure that there is adequate lighting to perform the assessment. The nurse would keep the client informed about the assessment process and obtain the client​'s consent prior to starting the assessment process. The nurse must use​ critical-thinking skills to analyze and understand observations during the assessment.

Identify the cue that indicates the possible presence of a disease process in a client. ​Instructions: Use the dropdown menus in the left column to select the correct cue for each indication in the right column. Cue Indication Lower extremity edema Asymmetric facial expression Generalized odor Bruised skin Assess cranial nerves Assess wound for drainage Assess use of anticoagulant medication Assess cardiac function and heart sounds

Assess cranial nerves- Asymmetric facial expression Assess wound for drainage- Generalized odor Assess use of anticoagulant medication- Bruised skin Assess cardiac function and heart sounds- Lower extremity edema

Identify the anthropometric measurements taken during the physical assessment portion of the nutritional history based on the description. ​Instructions: Use the dropdown menus in the left column to select the correct anthropometric measurements for each description in the right column. Measurement Description Waist circumference Skinfold measurement Body Mass Index (BMI) Self - reporting Assesses appropriate​ weight-to-height ratio Calipers used to estimate subcutaneous body fat stores Generally unreliable Measurements taken at lateral border of ilium of pelvis

Assesses appropriate​ weight-to-height ratio- Body Mass Index (BMI) Calipers used to estimate subcutaneous body fat stores- Skinfold measurement Generally unreliable- Self - reporting Measurements taken at lateral border of ilium of pelvis- Waist circumference

The nurse is assessing a​ 5th-grade student who complains of back pain. What action by the nurse would indicate that he understands abnormal musculoskeletal conditions found in​ children? Assessing for congenital hip dislocations Assessing the spine for scoliosis Assessing the heart rate of the child Assessing for metatarsal adductus

Assessing the spine for scoliosis The nurse assessing a​ 5th-grade student who complains of back pain would assess the spine for scoliosis. Scoliosis is an abnormal curvature of the spine and is diagnosed in children through adolescence. The heart rate of the child would not be related to a musculoskeletal abnormality. Metatarsal adductus would be seen in newborn infants and is the turning in of the front portion of the foot due to positioning in the uterus. Congenital hip dislocations are discovered within the first year of life.

The nurse is reviewing information about fibroadenomas. What are the characteristics of this type of breast ​tumor?​ Select all that apply. Asymptomatic Usually occur as a single tumor Freely moveable Lead to carcinoma ​Well-defined and firm

Asymptomatic Usually occur as a single tumor Freely moveable ​Well-defined and firm Fibroadenomas usually occur as a single tumor near the nipple or upper outer quadrant. They are benign tumors that are freely​ moveable, well-defined, and firm. These tumors are asymptomatic and are usually found through breast​ self-exams. There is no link between cancer and fibroadenomas.

The nurse is working in the pediatric clinic for an​ ear, nose, and throat provider. What would the nurse consider when assessing the​ ears, nose, and throat of an infant or ​child? Select all that apply. Auditory tube is more horizontal. Baby teeth begin to erupt at around 6 months of age. Auditory canal is shorter. Permanent teeth erupt at age 9 to 10 years of age. Infants salivate at 3 months of age.

Auditory tube is more horizontal. Baby teeth begin to erupt at around 6 months of age. Auditory canal is shorter. Infants salivate at 3 months of age. Children and infants are more prone to bacterial infections because of their shorter auditory canals. The more horizontal auditory tubes in children contribute to bacteria entering the ear canal more easily. Baby teeth begin to erupt at around 6 months of age up until 2 years of age. Salivation begins around 3 months of​ age, and drooling occurs until the swallowing of saliva is learned by the infant. Permanent teeth erupt from age 6 through adolescence.

The nurse is preparing to conduct a physical exam on a client who reports shortness of breath. What is the priority assessment technique for this​ client? Auscultating the lungs Palpating the abdomen Inspecting the pulses Percussing the lungs

Auscultating the lungs The nurse would auscultate the breath sounds on the client with shortness of breath. Palpating the abdomen is part of the total physical​ exam, but it does not give the nurse information about the client​'s breathing. Pulses are​ palpated, not inspected. Auscultating the breath sounds will provide the most information regarding the​ client's shortness of​ breath, not percussion of the lungs.

Ms. Miller is a​ 42-year-old female who came to the clinic with back pain to the left of her spine. You prepare to assess her kidneys and flanks. What technique would not be used during the​ assessment? Auscultation Inspection Palpation Blunt percussion

Auscultation Auscultation is not part of kidney and flank assessment. Inspection and palpation of the right and left costovertebral angles and flank area and palpation of the right and left kidneys are part of the assessment. Blunt percussion of the right and left costovertebral angle is also part of the assessment of the kidneys and flanks.

The charge nurse is teaching a new student nurse the standard systemic way to assess the abdomen. Arrange the steps involved in the assessment of the abdomen in the correct sequence. ​Instructions: Click on the dropdown arrow for each step of the procedure in the right column and select the correct choice from the list. Order Procedure The nurse would listen for bowel sounds, then vascular sounds, and finally a friction rub. Listen to the left lower quadrant (LLQ). Listen to the right lower quadrant (RLQ). Listen to the left upper quadrant (LUQ). Auscultation before percussion and palpation. Listen to the right upper quadrant (RUQ).

Auscultation before percussion and palpation. Listen to the right lower quadrant (RLQ). Listen to the right upper quadrant (RUQ). Listen to the left upper quadrant (LUQ). Listen to the left lower quadrant (LLQ). The nurse would listen for bowel sounds, then vascular sounds, and finally a friction rub.

The nurse is preparing to assess the client​'s abdomen. When would the nurse performing auscultation of the​ abdomen? Percussion is performed before auscultation. Auscultation is the first step in the abdominal assessment. Auscultation is the last step in any assessment. Auscultation is performed before percussion and palpation.

Auscultation is performed before percussion and palpation. Always performing auscultation on the abdomen before palpation and percussion because the latter techniques can alter the natural sounds of the abdomen. Inspection is the first step in the assessment of the abdomen. Auscultation should be performed before percussion and palpation when assessing the abdomen.

The nurse is assessing the​ ears, nose,​ mouth, and throat of an adult client. What would not be included in performing this​ assessment? Rinne test Inspecting the​ gums, tongue, and mucous membranes Auscultation of the sinuses The whisper test

Auscultation of the sinuses Auscultation of the sinuses is not an appropriate way to assess the sinuses of an adult client. The techniques for assessing the​ ears, nose,​ mouth, and throat of an adult include​ inspection, palpation, and percussion. Inspecting the health of the​ gums, tongue, and mucous membranes would be part of the assessment of the mouth. The whisper test and the Rinne test are appropriate ways to assess hearing in an adult client.

The nurse is preparing to care for a group of clients. What clients would the nurse identify as being at the greatest risk for ​overnutrition? Select all that apply. An​ 85-year-old with arthritis A​ 35-year-old client who is a fitness instructor A​ 30-year-old male client with a BMI of 30 who has hypertension A teen athlete A​ 45-year-old client with a sedentary lifestyle

A​ 30-year-old male client with a BMI of 30 who has hypertension A​ 45-year-old client with a sedentary lifestyle Clients who have a sedentary​ lifestyle, are​ overweight, and/or have a chronic health condition are at an increased risk of overnutrition. Clients who are very active are not at risk for overnutrition. Clients who are very young or old and have chronic health conditions are at risk for undernutrition. A BMI greater than 30 is considered obese.

What part of the female genitalia is located in the posterior region of the labia majora and should be free of lumps when​ palpated? Urethra Bartholin glands Skene gland Vaginal wall

Bartholin glands The Bartholin glands are located in the posterior region of the labia majora and should be free of lumps when palpated. The vaginal​ wall, urethra, and Skene glands are not located in the posterior region of the labia majora.

What is the most common disease of the breast in female​ clients? Malignancy of the nipples Benign breast disease Breast cancer Ductal carcinoma

Benign breast disease Benign breast disease is the most common disease of the breast in female clients.​ Pain, cyst​ formation, and tenderness are common​ symptoms, and it has no link to breast cancer. Breast cancer is a common cancer in​ women, but benign breast disease is the most common disease of the breast. Cancer of the nipples and cancer of the milk ducts or breast ducts can​ occur, but they are not the most common diseases of the breast.

Which are abnormal assessment findings associated with cardiac​ disease? Select all that apply. Bilateral pedal edema Clubbing of fingernails Flat jugular veins Bounding carotid artery pulse quality Present pulsation to left carotid artery

Bilateral pedal edema Clubbing of fingernails Bounding carotid artery pulse quality A bounding carotid may indicate fluid​ excess, which is associated with cardiac disease. Fingernail clubbing indicates cardiac or respiratory disease as a result of decreased tissue perfusion. Lower extremity edema indicates fluid​ excess, which is a cardiac issue. Flat jugular veins are an expected finding. An absence of arterial pulsation is an indication of arterial obstruction.

The nurse is caring for a​ 3-year-old client after a tonsillectomy. Which behavioral assessment findings would be consistent with this client and​ pain? Slow to acknowledge pain Blames the pain on parents Experiences decreased sensations of pain Tries to be brave when experiencing pain

Blames the pain on parents Infants and preschoolers want to hold someone accountable for their pain. Older​ adults, not​ children, may experience decreased sensations of pain. The​ school-aged child, not a​ 3-year-old, would try to be brave when experiencing pain. The adolescent​ client, not a​ child, may be slow to acknowledge pain.

Arrange the steps of the cardiac cycle in the correct sequence beginning with ventricular filling. ​Instructions: Click on the dropdown arrow for each step of the cardiac cycle in the right column and select the correct choice from the list. Order Cardiac Cycle Atria are stimulated by the SA node's electrical current to contract. Ventricles are stimulated by electrical current and then contract. Blood enters the ventricles of the atria. Atrioventricular valves close because of increased pressure. Aortic and pulmonic valves open. Aortic and pulmonic valves close. Blood is ejected into the pulmonic and systemic circulation.

Blood enters the ventricles of the atria. Atria are stimulated by the SA node's electrical current to contract. Ventricles are stimulated by electrical current and then contract. Atrioventricular valves close because of increased pressure. Aortic and pulmonic valves open. Blood is ejected into the pulmonic and systemic circulation. Aortic and pulmonic valves close.

The charge nurse is teaching the student nurse about the bones of the body by having the student nurse group the bones according to function. What are the main functions that bones provide for the ​body? Select all that apply. Bones protect internal structures. Bones store fat and minerals. Bones act as transport for waste material. Bones act as levers for movement. Bones produce blood cells.

Bones protect internal structures. Bones store fat and minerals. Bones act as levers for movement. Bones produce blood cells. The skeletal system has a unique design made up of flat​ bones, small​ bones, long​ bones, and irregular bones all working together to protect structures. Some bones act as levers to facilitate​ movement, and others store fat and minerals. Blood cells are also produced in the bone marrow of​ bones, especially long bones. Waste material is excreted by the renal and hepatic systems and through the gastrointestinal​ (GI) tract, not in the bones.

What description best describes the breast tissue of a​ newborn? Breast tissue is absent. Breast tissue can be swollen and raised. A secondary mound over the breast forms the areola and nipple. Nipple tissue is absent.

Breast tissue can be swollen and raised. The breast tissue of a newborn can be swollen and raised due to the high levels of estrogen​ (hyperestrogenism) of pregnancy. Breast tissue is not absent in the​ newborn; it is apparent. Normal nipple tissue is present in the newborn but may not be raised. A secondary mound over the breasts forms the areola and nipple in the preadolescent stage.

What breast changes would not be expected with​ pregnancy? Nipples darken. Breast tissue enlarges. Breasts become​ soft, pliable, and nontender. Breasts become tender.

Breasts become​ soft, pliable, and nontender. Breasts would be tender and firmer during pregnancy. Enlarged breast​ tissue, tenderness, and darker nipple pigmentations are normal expectations during pregnancy.

Mr. Sullivan is a​ 52-year-old man who has abdominal bloating. What is not a normal sound heard when percussing the​ liver? Resonance Dullness Bruit Tympany

Bruit A bruit is a​ loud, blowing vascular sound that would not be heard when percussing the liver. Tympany is a​ loud, hollow sound that is heard when percussing the liver starting at the umbilicus on the right side and moving toward the rib cage. Dullness is the normal sound heard when percussing the lower border of the liver. When percussing downward from the fourth intercostal space along the right midclavicular​ line, the first sound heard is resonance over the​ lung, which changes to dullness as the upper border of the liver is reached.

The nurse is assessing the mouth of an adult client. What part of the anatomy is not considered part of the​ mouth? Uvula Soft and hard palates Buccal mucosa Tongue

Buccal mucosa The buccal mucosa is considered part of the vestibule or opening to the mouth. It is classified separately from the​ mouth, along with the​ lips, outer gum​ surfaces, and cheeks. Both the hard and soft palates are part of the inside of the mouth. The uvula is the part of the mouth that hangs down at the free edge of the soft palate. The tongue is the muscle at the floor of the mouth that aids in​ swallowing, digestion, and speech.

The nurse is assessing the vital signs of a client. How would the nurse assess the client​'s radial​ pulse? By providing firm pressure over the radial pulse By counting the pulse for 30 seconds if the pulse is irregular By placing a finger along the posterior wrist along the radius bone By placing two to three fingers on the anterior wrist along the radius bone

By placing two to three fingers on the anterior wrist along the radius bone The radial pulse is palpated by placing the pads of two to three fingers on the anterior wrist along the radius bone. The radial pulse is located along the anterior wrist along the radius bone. The nurse should count a regular pulse for 30 seconds and then multiply the finding by two to get the pulse rate. An irregular pulse is counted for a full minute. The nurse should press gently over the radial​ pulse, otherwise the pulse can be obliterated.

The nurse is assessing an adult client who complains of feeling sluggish and tired. What area of the head and neck would the nurse assess first for lymph node​ enlargement? Select the correct answer choice below. A- behind ear B- in front of ear C- lateral distal neck D- directly below mandible

C- lateral distal neck

You are performing a cranial nerve assessment on Ms.​ Cordero, a​ 35-year-old client, who has hearing loss and tinnitus. What cranial nerve​ (CN) might be​ affected? CN IV​ (trochlear) CN XII​ (hypoglossal) CN V​ (trigeminal) CN VII​ (vestibulocochlear)

CN VII​ (vestibulocochlear) Hearing loss and tinnitus are abnormal findings of CN VII​ (vestibulocochlear). CN IV​ (trochlear) supplies the nerve to the superior oblique​ muscle, which is one of the muscles of the eye. CN V​ (trigeminal) affects the ability to sense​ touch, pain, and muscle spasms in the face and muscles for chewing. CN XII​ (hypoglossal) affects the muscles of the​ tongue, and abnormal findings related to it would be impaired speech and deviation of the tongue to the paralyzed side.

You are performing a cranial nerve assessment on Ms.​ Cordero, a​ 35-year-old client, who has hearing loss and tinnitus. What cranial nerve​ (CN) might be​ affected? CN XII​ (hypoglossal) CN VII​ (vestibulocochlear) CN IV​ (trochlear) CN V​ (trigeminal)

CN VII​ (vestibulocochlear) Hearing loss and tinnitus are abnormal findings of CN VII​ (vestibulocochlear). CN IV​ (trochlear) supplies the nerve to the superior oblique​ muscle, which is one of the muscles of the eye. CN V​ (trigeminal) affects the ability to sense​ touch, pain, and muscle spasms in the face and muscles for chewing. CN XII​ (hypoglossal) affects the muscles of the​ tongue, and abnormal findings related to it would be impaired speech and deviation of the tongue to the paralyzed side.

The nurse is completing a neurologic assessment on a client. What findings would the nurse report to the health care ​provider? Select all that apply. Can correctly identify the number five when drawn on the hand Can not distinguish hot from cold Does not recognize a sharp object when touched with it Tells the nurse when the vibration from a tuning fork is felt Can correctly identify a shoe with eyes closed

Can not distinguish hot from cold Does not recognize a sharp object when touched with it The inability to distinguish temperature and the inability to identify a sharp or dull touch are abnormal findings and would be reported to the health care provider. The ability to identify a familiar​ object, identify a number drawn on the​ hand, and feel vibration are all normal findings and would not need to be reported to the health care provider.

Mr. Gibson is a​ 75-year-old client admitted to your unit. You begin to interview him but note that he is disoriented to place and time. His caregiver has accompanied him. By what means should you collect Mr.​ Gibson's health history​ information? Medical records Client ​Client's primary care provider Caregiver

Caregiver The caregiver is a secondary source that is best able to provide information about Mr.​ Gibson's health history. The client has exhibited signs of​ disorientation, making him a less reliable source. His medical records have already been accessed in the preinteraction phase of the interview process. The​ client's primary care provider does not need to be contacted to compile a health history.

An adult has bandages over bilateral wrists. For routine pulse​ assessment, which site is best for arterial pulse​ evaluation? Brachial arteries Carotid arteries Dorsalis pedis arteries Femoral arteries

Carotid arteries The carotids are easily accessible for routine pulse assessment. You should palpate them gently to avoid dislodging potential arterial wall plaque. The dorsalis pedis​ arteries, which are accessed on the top of the​ foot, may be compromised due to comorbidities or other factors that affect perfusion. In​ adults, the femoral and brachial arteries are more deeply positioned and difficult to​ find; it is a more invasive technique for a routine pulse assessment. The brachial artery site is preferred and generally easier to access in an infant

While performing a neurological assessment on a client you hear a bruit. What method of assessment did you​ use? Meningeal assessment Glasgow Coma Scale Carotid auscultation Babinski response

Carotid auscultation Carotid auscultation would reveal a​ bruit, which may indicate atherosclerosis where blood flow to the brain is diminished. An abnormal finding of a meningeal assessment would be a stiff neck. On the Glasgow Coma​ Scale, a score of 3 or less indicates a​ nonresponsive, comatose client who requires immediate attention. A positive Babinski response is an abnormal finding when assessing the plantar flexion of an adult client.

The nurse is using percussion to assess the chest of an adult client and notices a dull sound. What abnormal cardiac conditions would the nurse​ consider? Carotid disease or dissecting ascending aneurysm First degree heart block Congestive heart failure Myocardial infarction

Carotid disease or dissecting ascending aneurysm The dull sound upon percussion is related to a possible dissection ascending aneurysm due to the anatomy and physiology of a​ dissection, or to carotid artery disease. The dull sound is most commonly noted with an abnormality in the large artery in the chest and would be correlated with other symptoms and not used as a lone diagnostic tool. The sound heard with percussion over the lungs of a client with a cold or congestive heart failure would be more tympanic. Other diagnostic methods are needed to detect first degree heart​ block, congestive heart​ failure, and myocardial infarction.

Jennie Johnson is a​ 30-year-old woman who is 7 months pregnant. She complains of backache. In your explanation to her as to the relationship of pregnancy to back​ pain, you tell her that the vertebrae are classified as which type of​ joint? Synovial Striated Fibrous Cartilaginous

Cartilaginous A joint is also refered to as an articulation. It is the point with two or more bones in the body come together. Cartilaginous​ joints, such as those in the​ vertebrae, are joined by cartilage. Fiborus​ joints, such as the sutures joining the bones of the​ skull, are referred to as fibrous joints. Synovial​ joints, such as those of the​ knee, separate bones by a​ fluid-filled cavity. Strated refers to a type of muscle.

Ms. Lopez is 38 years old and has a family history of breast cancer. When assessing her breasts and​ axillae, what lymph nodes would you​ palpate? Central axillary nodes Deep cervical nodes Submental nodes Parauterine nodes

Central axillary nodes When palpating the axillae you would palpate the central axillary​ nodes, which are located under the armpit. Parauterine nodes​ (located in the​ abdomen), deep cervical nodes​ (located posterior to and under the sternocleidomastoid muscle in the​ neck), and submental nodes​ (located behind the tip of the mandible at the​ midline) would not be palpated as part of the breast assessment.

The nurse is assessing the bloody nose of an adult client. What are the possible causes of ​epistaxis? Select all that apply. Coagulation problems Rhinitis Hypertension Trauma Earache

Coagulation problems Rhinitis Hypertension Trauma Trauma, rhinitis, high blood​ pressure, and clotting problems can cause a bloody​ nose, or epistaxis. Earache does not cause the nose to bleed.

What structure is the largest and most​ well-developed area of the​ brain? Thalamus Brain stem Cerebellum Cerebrum

Cerebrum The​ cerebrum, the largest and most​ well-developed area of the​ brain, lies in front of the brain stem. The brain stem contains the​ midbrain, pons, and the medulla oblongata and is located between the cerebrum and spinal cord. The cerebellum is responsible for skeletal muscle coordination and smooth movements. The thalamus is a​ large, dual-lobed mass of gray matter buried under the cerebral cortex and is involved in sensory perception and regulation of motor function.

The nurse is assessing an older adult client for hearing loss. What factors may contribute to this client​'s hearing ​loss? Select all that apply. Cerumen accumulation Nodules or swelling Infection Age Shape of the pinnae

Cerumen accumulation Nodules or swelling Infection Age Hearing ability decreases with age. Cerumen accumulation increases with age and may interfere with the ability to hear. Infection can temporarily decrease hearing by blocking hearing receptors. Nodules and swelling can inhibit hearing temporarily. The shape of the external ear does not affect hearing in an adult.

The nurse is preparing to assess a client​'s neurologic system. Which is not part of the central nervous​ system? Cervical spine Cerebral spinal fluid Thalamus Meninges

Cervical spine The cervical spine is part of the peripheral nervous system. The meninges are three connective tissue membranes that​ cover, protect, and nourish the central nervous system. Cerebral spinal fluid is​ clear, colorless body fluid that the brain floats​ in, providing a cushion​ effect, and is part of the central nervous system. The thalamus is a​ large, dual-lobed mass of gray matter buried under the cerebral cortex. It is involved in sensory perception and regulation of motor function and is part of the central nervous system.

The nurse is assessing an adult client who complains of neck pain and swelling. What prominent cervical vertebra is easily palpable when assessing the head and neck of an adult​ client? Cervical vertebra 2 Cervical vertebra 7 The mandible Cervical vertebra 1

Cervical vertebra 7 C7 is the most palpable vertebra and is a landmark when doing a physical exam of the head and neck on an adult client. C1 is the atlas and carries the​ head, but it is not palpable. C2 is the axis and allows movement of the​ head, but it is not palpable. The mandible is part of the jaw. It can be palpated during a physical exam for swelling and lymph​ nodes, but it is not a cervical vertebra

The nurse is caring for a client who has been diagnosed with​ Alzheimer's disease. What assessment finding is attributed to this disease​ process? Sudden change in level of consciousness Inability to follow directions Flat affect and tone Changes in speech and​ long-term memory

Changes in speech and​ long-term memory Changes in speech and language as well as loss of​ long-term memory could be symptoms of​ Alzheimer's disease. A sudden change in a​ client's level of consciousness can indicate a​ stroke, seizure, or hypoglycemia. A flat affect and tone can be the result of emotional problems or depression. Lack of grooming and inappropriate dress can indicate​ depression, not necessarily​ Alzheimer's disease.

The nurse is caring for an adult client with low back pain. The health care provider has ordered a heating pad for this client. What action would the nurse need to take to prevent a burn from this​ equipment? Keep cool water handy in case of burns. Check the heating pad and the client Apply protective ointment before using the heating pad. The responsibility for equipment does not lie with the nurse.

Check the heating pad and the client The nurse would need to assess the client at frequent intervals to ensure that the equipment is providing a therapeutic benefit and is not malfunctioning. The nurse has a responsibility to prevent burns to a client when using a heating pad. Cool water and ointment would not be used without a health care provider​'s order. The nurse is responsible for seeking assistance if the equipment is not working and for assessing a client during the use of hospital equipment.

The nurse is performing a focused assessment of the respiratory system on an adult client in no distress. What would be the normal configurations of the chest for this​ client? Substernal retractions on inspiration Chest is​ symmetrical, and the chest wall rises and falls equally during respiration Intercostal retractions during respiration Asymmetrical chest wall

Chest is​ symmetrical, and the chest wall rises and falls equally during respiration The nurse would inspect the appearance of the thorax or chest wall during a focused physical exam. The chest wall would be symmetrical and equal on both sides of the chest. The rise and fall of the chest during respiration would be equal. There would be no evidence of substernal or intercostal retractions in a healthy adult client not in respiratory distress. The adult male may use more abdominal muscles when​ breathing, but doing so would not cause retractions.

The nurse is caring for a​ 65-year-old client with chronic obstructive pulmonary disease​ (COPD). What assessment finding would the nurse expect to see in this​ client? Hirsutism Kaposi sarcoma Splinter hemorrhage Clubbing of nails

Clubbing of nails Clubbing of nails occurs in chronic respiratory and cardiac conditions in which oxygenation is compromised. Hirsutism is typically caused by endocrine or metabolic dysfunction. Kaposi sarcoma is a malignant tumor common in individuals who are HIV positive. Splinter hemorrhages can occur as a result of trauma and in clients with endocarditis.

The nurse is assessing children with congenital abnormalities in a clinic. What abnormal conditions would be recognized through an assessment of the head and ​neck? Select all that apply. Chronic allergies Down syndrome Fetal alcohol syndrome Congenital hypothyroidism Hyperthyroidism

Chronic allergies Down syndrome Fetal alcohol syndrome Congenital hypothyroidism Children with fetal alcohol syndrome have midfacial hypoplasia and a broad bridge across the nose. Infants with congenital hypothyroidism have a depressed nasal bridge and protruding tongue. Children with Down syndrome have a flat nasal​ bridge, protruding​ tongue, and short​ neck, among other physical signs. Children with chronic allergies have a bluish shadow under their​ eyes, do​ open-mouth breathing, and have swollen nasal turbinates on exam. Children may not have physical signs in the head or neck area on assessment if they suffer from hyperthyroidism.

The nurse is assessing a young adult with chronic bronchitis. What symptoms would the nurse expect to assess to confirm this ​diagnosis? Select all that apply. Chronic productive cough Hyperresonance upon percussion Dyspnea Use of accessory muscles when breathing Wheezes and rhonchi are present on auscultation of the lungs

Chronic productive cough Dyspnea Use of accessory muscles when breathing Wheezes and rhonchi are present on auscultation of the lungs Clients with chronic bronchitis have airway inflammation and​ swelling, which leads to difficulty breathing and a rapid respiratory rate. Often a chronic productive cough​ persists, and the client begins using accessory muscles to help with breathing. Wheezes and rhonchi may be present. The nurse should percuss​ resonance, not​ hyperresonance, when a client has chronic bronchitis.

The nurse is performing a cardiovascular assessment on an adult client. The nurse would need to identify what physical ​landmarks? Select all that apply. Clavicles Angle of Louis Manubrium Umbilicus Sternum

Clavicles Angle of Louis Manubrium Sternum The Angle of​ Louis, the​ sternum, the​ manubrium, and the clavicles are all important landmarks to help identify structures below the rib​ cage, and to find the​ second intercostal space and the midclavicular line for cardiovascular assessment. The umbilicus is well below where the nurse would assess the heart.

The nurse manager is educating student nurses about the Joint Commission requirement for documentation. What information is required to be ​documented? Select all that apply. Client refusals Client assessments Client satisfaction Nursing interventions Nursing diagnoses

Client assessments Nursing interventions Nursing diagnoses The Joint Commission requires that the​ client's record include evidence of client​ assessments, nursing diagnoses and nursing interventions. The Joint Commission does not require that client satisfaction and refusals be documented.

When is it advisable to not seek information from a secondary source on a​ client? Client cannot speak common language Client is competent but unwilling to provide personal information Client is disoriented to​ person, place, or time Client is an infant

Client is competent but unwilling to provide personal information If a client is competent and unwilling to offer information and has not granted permission to seek information from a secondary​ source, respect the wishes of the client and try again at a later time. You may seek to obtain information through secondary sources for​ client's who cannot speak the common language or are​ disoriented, and for​ client's who are infants.

Which is the least reliable indicator of​ pain? Client​'s blood pressure of​ 158/92 mmHg and pulse 102​ beats/minute. Client​'s report that open double quote"my pain is 8 out of 10.close double quote" Client exhibits facial grimacing and restlessness. Client​ states, open double quote"My foot hurts.close double quote

Client​'s blood pressure of​ 158/92 mmHg and pulse 102​ beats/minute. A change in vital signs is the least reliable indicator of​ pain, in both acute and chronic pain conditions. The client​'s report of pain is the most accurate and reliable indicator of pain. If the client is unable to communicate​ verbally, non-verbal signs such as​ restlessness, guarding, and grimacing are often indicators of pain or discomfort.

What structures form the external female ​genitalia? Select all that apply. Clitoris Cervix Mons pubis Labia majora Vagina

Clitoris Mons pubis Labia majora The mons pubis is the external mound of tissue with pubic hair covering the pubic bone. The labia majora are external structures that protect the female genitourinary system and the vagina. The clitoris is the sensitive protrusion located where the two labia minora meet. The vagina is the internal muscular tube connected to the cervix and uterus. The cervix is connected to the vagina and the uterus and is an internal structure.

Heart valve closure causes audible sounds on auscultation. Identify the heart sound based on the location it is best heard and the action of the heart. ​Instructions: Use the dropdown menus in the left column to select the type of heart sound for each description in the right column. Heart Sound Description S3 S4 S1 (lub) S2 (dub) Closure of atrioventricular valves Auscultated before S1 Auscultated after S2 Best heard at apex of heart Louder at base of heart May be heard in 3rd trimester of pregnancy Closure of semilunar valves Coincides with R wave of QRS complex Caused by atrial contraction and ejection of blood into ventricles in late diastole

Closure of atrioventricular valves- S1 Auscultated before S1- S4 Auscultated after S2- S3 Best heard at apex of heart- S1 Louder at base of heart- S2 May be heard in 3rd trimester of pregnancy-S3 Closure of semilunar valves- S2 Coincides with R wave of QRS complex- S1 Caused by atrial contraction and ejection of blood into ventricles in late diastole- S4

Crackles/rales occur at the end of inspiration and are caused by​ what? Blocked airflow Pleural inflammation Collapsed or​ fluid-filled alveoli Obstructed upper airway

Collapsed or​ fluid-filled alveoli ​Crackles/rales occur at the end of inspiration and can be​ high-pitched, short, and​ crackling, or​ loud, moist,​ low-pitched, and bubbling.​ Crackles/rales are caused by collapsed or​ fluid-filled alveoli. An obstructed upper airway would be indicated by​ stridor, a type of rhonchi that is continuous. Pleural inflammation is indicated by friction​ rub, and blocked airflow is indicated by wheezes.

The nurse is completing a neurologic health history for a​ 55-year-old client who reports a history of seizures. What step would the nurse take​ next? Place the client on seizure precautions. Notify the health care provider. Complete the neurologic assessment Administer antiseizure medications.

Complete the neurologic assessment The nurse should first complete a more detailed neurologic assessment of the client. This assessment will direct further care and provide data for the health care provider. There is not enough information to support placing the client on seizure precautions or administering antiseizure medications at this time.

What parts of the nursing plan of care should you include in a​ client's record? Select all that apply. Health care provider orders Comprehensive admission assessment Nursing diagnoses and client needs Client outcomes Medical diagnoses

Comprehensive admission assessment Nursing diagnoses and client needs Client outcomes The nursing plan of care should include a comprehensive admission​ assessment, nursing diagnoses and client​ needs, and desired client outcomes. A nursing plan of care does not include health care provider orders or medical diagnoses.

The nurse is assessing an adult client who appears to have an accumulation of fluid in the abdominal​ area, giving it a protuberant appearance. What conditions would the nurse consider that are associated with ​ascites? Select all that apply. Congestive heart failure Neoplastic diseases Peritonitis Ulcerative colitis Nephrosis

Congestive heart failure Neoplastic diseases Peritonitis Nephrosis Clients with congestive heart failure or kidney disease​ (nephrosis) may have an accumulation of fluid in the abdominal cavity. Peritonitis is the inflammation of the lining of the abdomen and may result in the accumulation of fluid in the abdomen. Neoplastic diseases and cancer may result in an accumulation of fluid in the abdomen. Ulcerative colitis is a chronic inflammatory bowel disorder and is associated with loose​ stools, not ascites.

The nurse is assessing the client​'s cranial nerves and finds that the client has lost the sense of smell. What cranial nerve is​ affected? Cranial nerve VIII Cranial nerve V Cranial nerve I Cranial nerve VII

Cranial nerve I Damage to cranial nerve I would result in the loss of the sense of smell. Cranial nerve VII damage would result in asymmetry or weakness of the facial muscles. Damage to cranial nerve VIII would result in the inability to hear. An inability to feel the sensation of touch to the eye would be the result of damage to cranial nerve V.

Which of the following are nontherapeutic communication​ techniques? Select all that apply. ​Cross-examination Passing judgment False reassurance Focusing Lay terms

Cross-examination Passing judgment False reassurance False reassurance occurs when a client is assured of a positive outcome with no basis for believing it. Passing judgment conveys the message that the client​'s values are not a consideration.​ Cross-examining a client with a litany of questions can cause anxiety and stresslong dashallow the client to set the pace. Focusing is an effective communication skill that helps the client zero in on a topic. The use of lay terms puts the client at ease and helps the client to understand the situation.

The nurse is assessing a​ 5-month-old infant in the emergency department. The infant is irritable and is making a​ loud, high-pitched crowing noise with every respiration. What abnormal respiratory condition might this infant be​ experiencing? Croup Asthma Bronchitis COPD

Croup Infants who make a​ loud, high-pitched crowing sound that sounds like it is in the neck and upper airway are likely to be experiencing croup. Chronic obstructive pulmonary disease​ (COPD) is diagnosed in adults and is a disease of the alveoli. Asthma is an inflammation of the airway but is associated with wheezing and cough. Bronchiolitis and bronchitis are associated with congestion and cough lower in the​ chest, and infants with either disease would not make a​ high-pitched, crowing sound when breathing.

What should you include in the discharge summary for a client who is being transferred to another​ institution? Select all that apply. Copies of all nursing notes during length of stay Current medication list Activities of Daily Living​ (ADL) flow sheets from hospitalization Description of​ client's physical,​ mental, and emotional status at discharge Current functional and​ self-care abilities

Current medication list Description of​ client's physical,​ mental, and emotional status at discharge Current functional and​ self-care abilities The discharge summary should include the current medication​ list, functional and​ self-care abilities, and a description of the​ client's current​ physical, mental, and emotional status. ADL flow sheets and all nursing notes are not included in the discharge summary.

The nurse is assessing the head and neck of an adult with Paget disease. What is the classic symptom of this abnormality of the head and​ skull? Hyperthyroidism Large neck All the bones in the head and body are enlarged. Enlargement of the frontal bone of the skull

Enlargement of the frontal bone of the skull The frontal bone is enlarged in Paget disease and has an​ acorn-like shape. Acromegaly is the disease where all of the bones of the head and body are larger than normal. Protruding eyes may be an abnormal symptom of​ hyperthyroidism, and the cause of a large neck may be a goiter.

The nurse is giving a lecture to students on the variety of abnormal heart conditions that are frequently seen in the clinic. What abnormal cardiovascular conditions would be included in this lecture for students interested in health care for ​adults? Select all that apply. Damage to the myocardium A heart rate of 150 beats per minute A heart rate of 70 beats per minute Valvular disease Septal defects

Damage to the myocardium A heart rate of 150 beats per minute Valvular disease Septal defects Abnormal findings can be a​ murmur, diseases of the myocardium such as a heart​ attack, valvular​ diseases, septal​ defects, congenital heart defects and​ disease, or an abnormality in the electrical system of the heart. Having a heart rate greater than 150 beats per minute could indicate an abnormality in the electrical system. A heart rate of 70 beats per minute is a normal finding.

The nurse working in a new acute care facility is reviewing the disaster plan for the facility. What are the priority areas that would be considered when disaster ​planning? Select all that apply. Decontamination Communication Determining the roles of authority Planning for staffing The number of clean uniforms available

Decontamination Communication Determining the roles of authority Planning for staffing The nurse plays a major role in planning for staffing of the facility in case of a disaster. The role of authority must be established in the disaster plan before a disaster occurs. The disaster plan must also include the process for​ decontamination, and the nurse would be included in this process. Communication between government​ agencies, EMS, the acute care​ facilities, and all stakeholders in the community is an important part of disaster planning. The nurse would play an important role in deciding​ how, when, and where equipment and supplies would be found and​ stored, but the availability of clean uniforms would not be a priority when planning for a disaster.

The nurse is assessing an adult client for respiratory mechanical movement and understands that many factors would be considered. What factor would the nurse expect to cause an increase in mechanical​ movement? Guarding related to pain Muscle atrophy Compromised nerve supply Decrease in fatty tissue and weight loss

Decrease in fatty tissue and weight loss A decrease in fatty​ tissue, as in a lean body​ frame, would increase the mechanical movement rather than decrease it.Compromised nerve​ supply, muscle atrophy and guarding relating to pain would result in decreased mechanical movement. The nurse would note shallow​ breaths, poor chest excursion during the assessment.

The nurse is assessing the abdomen of an older adult. What changes in the digestive system of the older adult would the nurse consider as she evaluates this client​'s current state of ​nutrition? Select all that apply. An increase in liver size Decrease in saliva and digestive enzymes Loose stool as a result of too much fresh fruit consumption ​Ill-fitting, broken, or lost dentures along with tooth​ loss, leading to decreased nutritional status A delay in esophagus emptying

Decrease in saliva and digestive enzymes ​Ill-fitting, broken, or lost dentures along with tooth​ loss, leading to decreased nutritional status A delay in esophagus emptying Older adults with tooth loss and poor quality dentures would not be able to chew the amount of food appropriate for good nutrition. Older adults have fewer digestive enzymes and less saliva to break up food into particles for​ digestion, which can interfere with absorption of nutrients. A delay in esophageal emptying in an older adult can lead to feeling full or​ uncomfortable, causing the client to stop eating before he or she has met his or her nutritional requirements. Older adults suffer from constipation because of a lack of fresh fruits and vegetables. The liver decreases in size with​ age, so an older adult client has a smaller liver on palpation if absent of disease.

Which of the following factors might cause a client​'s blood pressure to​ increase? Select all that apply. Decreased blood viscosity Decreased elasticity of the smooth muscles of the vessels Increased cardiac output Decreased peripheral vascular resistance Increased blood volume

Decreased elasticity of the smooth muscles of the vessels Increased cardiac output Increased blood volume Decreased elasticity of the smooth muscles of the vessel walls results in decreased compliance of those vessels to dilate in response to increased blood​ volume, thus increasing blood pressure. Increased blood volume and increased cardiac output raise blood pressure. Decreased blood viscosity and decreased peripheral vascular resistance result in lower blood​ pressure, as it is easier for blood to flow through the​ vessels, resulting in less effort exerted by the heart.

The nurse is preparing to assess an​ 87-year-old client​'s skin. Which assessment findings would the nurse expect to find in this ​client? Select all that apply. Decreased perspiration Fine facial hair Cherry angiomas Dry hair and skin Liver spots

Decreased perspiration Cherry angiomas Dry hair and skin Liver spots Due to a decrease in sebum​ production, older adults often suffer from dry hair and skin. The activity of sweat glands decreases with​ age, causing a decrease in perspiration. Hyperpigmented spots called liver spots increase with age. Cherry angiomas are​ dark-red nonmalignant spots that increase with age. Due to hormonal changes with​ aging, facial hair becomes coarse.

Identify the laboratory tests based on the indicators of nutritional status. ​Instructions: Use the dropdown menus in the left column to select the laboratory test for each indicators of nutritional status in the right column. Laboratory Test Indicator Serum albumin Cholesterol and triglycerides Hemoglobin and hematocrit Decreased​ value: anemia or hemodilution Increased​ value: vitamin A deficiency Increased​ value: increased risk for cardiac arterial disease Increased​ value: hemoconcentration due to dehydration Decreased​ value: desired outcome Decreased​ value: protein depletion or decreased liver function

Decreased​ value: anemia or hemodilution- Hemoglobin and hematocrit Increased​ value: vitamin A deficiency- Serum albumin Increased​ value: increased risk for cardiac arterial disease- Cholesterol and triglycerides Increased​ value: hemoconcentration due to dehydration- Hemoglobin and hematocrit Decreased​ value: desired outcome- Cholesterol and triglycerides Decreased​ value: protein depletion or decreased liver function- Serum albumin

What is the most accurate description of Kussmaul​'s ​breathing? Respirations less than 12 per minute Shallow breathing with periods of apnea Difficulty breathing unless in a sitting position Deep and rapid breathing

Deep and rapid breathing Kussmaul​'s breathing is a type of respiration found in clients with metabolic acidosis​ (such as diabetic​ ketoacidosis) and is characterized by a deep and rapid breathing pattern. Biot​'s ​respirations, often in clients with increased intracranial​ pressure, are described by shallow breathing with periods of apnea. Orthopnea is difficulty breathing in a supine​ position; the client needs to sit or stand in an upright position to obtain relief. Respirations of less than 12 per minute is called bradypnea.

A​ 65-year-old male client presents to the clinic for a routine examination. The client is moderately obese. The nurse would provide health teaching regarding the association of obesity with the development of what health ​problems? Select all that apply. Type 1 diabetes mellitus Degenerative joint disease Hypertension Elevated blood cholesterol Urinary tract infection

Degenerative joint disease Hypertension Elevated blood cholesterol Hypertension, elevated blood​ cholesterol, and degenerative joint disease are associated with obesity. Urinary tract infections are not directly associated with​ obesity, and type 1 diabetes mellitus is not associated with obesity.

The nurse is conducting an assessment of the client​'s nails. What findings would the nurse report to the health care​ provider? Delayed capillary refill ​Smooth, strong nail beds ​Clean, well-groomed nails No clubbing present

Delayed capillary refill Capillary refill is assessed by depressing the nail edge briefly. Color returns to healthy nails instantly. A delay in capillary refill should be reported to the health care provider.​ Smooth, healthy​ nails; no​ clubbing; and​ smooth, strong nail beds are all expected findings of normal nails and do not need to be reported to the health care provider.

A client who has had excessive exposure to ultraviolet radiation from the sun is at risk for which of the​ following? Select all that apply. Depressed immune system Thickening of the skin Hepatitis C Diabetes mellitus Skin cancer

Depressed immune system Thickening of the skin Skin cancer Excessive exposure to ultraviolet radiation from the sun thickens and damages the​ skin, depresses the immune​ system, and predisposes an individual to skin cancer from altered DNA in the skin cells. Hepatitis C is a viral disease that causes inflammation of the liver that can be contracted when getting pierced or tattooed. Diabetes mellitus is a metabolic disease in which a person has high blood glucose levels and may be considered if the client has sores that are slow to heal.

The nurse begins the physical examination with inspection. Which techniques would the nurse include in the inspection of the ​client? Select all that apply. Determining pupil reaction Viewing the interior of the ear with a scope Determining skin temperature of the feet Determining the texture of the skin Noting green drainage from the left eye

Determining pupil reaction Viewing the interior of the ear with a scope Noting green drainage from the left eye Inspection includes the use of the senses of​ smell, hearing, and sight to detect unusual body​ odors, sounds, and physical appearance. Using an otoscope to examine the ear is inspection. Pupil reaction is determined by inspecting the eye with a penlight. Inspection includes noting drainage from the eye. The texture and temperature of the skin are determined with​ palpation, not inspection.

What symptom indicates an infection in​ progress? Discharge from penis Lump noted in scrotal area Itching in the genital area Unexplained weight loss

Discharge from penis A client experiencing discharge from his penis is likely to have an infection in progress. Unexplained weight loss can indicate cancer. Itching in the genital area may indicate an allergy to soaps or lotions or pubic lice. Detecting a lump in the scrotal area does not indicate an active infection and requires further follow up from the health care provider.

Which of the following are​ trauma-induced ​disorders? Select all that apply. Dislocation Fracture Muscle sprain Fibromyalgia Gout

Dislocation Fracture Muscle sprain Dislocation is a displacement of the bone from its normal anatomical position. Fracture is a partial or complete break in the continuity of a bone due to trauma. Muscle sprain is a partial muscle tear resulting from overstretching or overuse. Gout is a type of arthritis caused by uric acid crystal deposits in the joints. Fibromyalgia is pain in the muscles and soft tissue that surround and support the joints. Both gout and fibromyalgia are not​ trauma-induced disorders.

Detected while palpating a​ client's head and​ neck, which of the following abnormal findings might indicate​ pneumothorax? Displacement of the trachea Crepitus of the temporomandibular joint Enlarged thyroid Tenderness of the temporal artery

Displacement of the trachea Displacement of the trachea is an abnormal finding that indicates pneumothorax as well as pulmonary fibrosis and masses in the neck or mediastinum. An enlarged thyroid may indicate a mass or an autoimmune disorder. Tenderness of the temporal artery may be an indication of inflammation of the artery. Crepitus of the temporomandibular joint indicates joint problems.

The nurse is conducting a health history interview for a​ 33-year-old client. What are the internal factors to consider that may affect this ​interview? Select all that apply. Displaying empathy Having the ability to listen Ensuring privacy Taking notes Demonstrating an optimistic view of others

Displaying empathy Having the ability to listen Demonstrating an optimistic view of others Having the ability to​ listen, displaying​ empathy, and demonstrating an optimistic view of others are internal factors that can positively affect the communication process during an interview. Ensuring privacy and taking notes are external factors that can affect communication during the interview process.

When inspecting the abdomen of a healthy female client who is not​ pregnant, what observation would be​ abnormal? Uniform skin color Rounded contour Symmetrical Distended abdomen

Distended abdomen A distended abdomen in a nonpregnant female would be an abnormal finding. Uniform skin color throughout the entire abdomen is a normal finding. It is normal for a healthy female to have a rounded or flat abdomen depending on general weight. The abdomen of a healthy female would be symmetrical on both sides.

What symptoms would be noted during the physical exam of the client who is in​ right-sided heart ​failure? Select all that apply. Pulmonary congestion Distended neck veins Liver congestion Wet lung sounds Ankle edema

Distended neck veins Liver congestion Ankle edema Right-sided heart failure is the result of the right side of the heart not pumping effectively and blood backing up into the systemic system. Symptoms resulting from this backup include neck vein​ distension, liver​ congestion, and peripheral edema.​ Left-sided heart failure causes blood to back up into the pulmonary​ system, resulting in respiratory symptoms such as pulmonary congestion and wet lung sounds.

Pain can be associated with acute or chronic abdominal conditions. What diseases might be associated with acute inflammation of abdominal organs during an assessment of an adult with abdominal ​pain? Select all that apply. Arthritis Diverticulitis Appendicitis Cholecystitis Pancreatitis

Diverticulitis Appendicitis Cholecystitis Pancreatitis Appendicitis is an acute inflammation of the appendix with pain in the right lower quadrant​ (RLQ). Cholecystitis is inflammation of the gallbladder with acute pain in the right upper quadrant​ (RUQ). Diverticulitis is acute pain and cramping in the left lower quadrant​ (LLQ) that radiates to the back. Pancreatitis is acute pain in the RUQ that is often described as knifelike. Arthritis produces acute​ pain, but the condition is related to chronic inflammation of the​ joints, not the abdomen.

Identify the environment related to the present health history of the skin of a client based on the questions asked by the nurse. ​Instructions: Use the dropdown menus in the left column to select the type of environment for each question in the right column. Environment Question External Environment Internal Environment Do you take​ medications? Do you work in an environment where​ x-rays are​ used? Have you changed your diet​ recently? Do you wear gloves to​ work? Are you experiencing​ stress? How often do you​ travel? Describe your stress level. Have you recently been exposed to extreme​ temperatures? Are you​ pregnant?

Do you take​ medications? internal Do you work in an environment where​ x-rays are​ used? external Have you changed your diet​ recently? internal Do you wear gloves to​ work? external Are you experiencing​ stress? internal How often do you​ travel? external Describe your stress level. internal Have you recently been exposed to extreme​ temperatures? external Are you​ pregnant? internal

Mr.​ Hughes, a​ 68-year-old male​ client, has a​ reddened, edematous right lower extremity with decreased peripheral pulses. Which area of your hand would be most accurate to use in assessing the temperature of the​ leg? Palmer surface of the hand Dorsal surface of the hand Finger pads Ulnar surface of the hand

Dorsal surface of the hand The dorsal surface of the hand is most sensitive to changes in temperature. Mr. Hughes has signs and symptoms of deep vein thrombosis​ (DVT) and this is likely to result in a warmer lower extremity. The finger pads are best used for determining pulses or crepitus. The palmer surface of the hand is best used to determine​ pain, tenderness, and consistency of the skin. The ulnar surface of the hand is helpful when palpating vibrations such as fremitus.

The nurse is caring for a client who is experiencing gait and balance problems. What factor would the nurse recognize as a potential cause of the​ problem? Drug or alcohol intoxication ​Alzheimer's disease Depression Seizures

Drug or alcohol intoxication Unsteady gait and balance may be caused by drug or alcohol​ intoxication, motor neuron​ weakness, or muscle weakness.​ Alzheimer's disease,​ seizures, and depression do not cause an unsteady gait or loss of balance.

What would the nurse expect to find when assessing the skin of an older adult​ client? Increased sebum production Increased pigmentation Milia Dry skin and hair

Dry skin and hair Older adults have decreased sebum production and usually have dry skin and hair. Increased pigmentation and sebum production are not usually related to aging. Milia are​ whitish, pearly bumps found on the skin in newborns.

Identify the clinical indication of different cardiac assessment finding. ​Instructions: Use the dropdown menus in the left column to select the correct clinical indication for each finding in the right column. Indication Finding Carotid disease Superior vena cava obstruction Abdominal aortic aneurysm Elevated right ventricular volume Dull sound upon percussion of cardiac border Palpated thrill to​ sub-xiphoid area ​Dilated, distended chest veins Abdominal pulsations

Dull sound upon percussion of cardiac border- carotid disease Palpated thrill to​ sub-xiphoid area- elevated right ventricular volume ​Dilated, distended chest veins- superior vena cava obstruction Abdominal pulsations- abdominal aortic aneurysm

A client with a​ demanding, high-stress job has abdominal problems. Which of the following is a condition aggravated by​ stress? Cholecystitis Duodenal ulcer Appendicitis Ectopic pregnancy

Duodenal ulcer High levels of stress may cause or aggravate abdominal problems. A duodenal ulcer is an example of a​ stress-related abdominal problem. Appendicitis is caused by an obstruction and inflammation. Cholecystitis can be a result of fatty meals​ and/or an obstruction of bile duct with cholelithiasis. An ectopic pregnancy may be the result of tubal​ damage, pelvic​ infection, or hormonal disorders.

The nurse is assessing a client admitted to the hospital for pain. Which assessment finding would determine if the client has chronic​ pain? Etiology Intensity Duration Location

Duration Duration distinguishes between acute and chronic pain. Location would determine where the client is hurting. The nurse should have the client point to where it hurts. Intensity would tell the nurse how much pain the client is experiencing. The nurse would use a scale such as 0dash-10 when assessing the intensity of the pain. Etiology tells the nurse the cause of the client​'s pain.

The nurse is conducting a health assessment interview with a client who has tattoos. What condition is a client with tattoos at greater risk of​ developing? Hansen​'s disease Hemophilia Hepatitis A HIV

HIV The client who has tattoos accepted the risks of developing HIV and hepatitis C upon receiving the tattoos. Hemophilia occurs when a client​'s blood lacks a clotting factor. Hansen​'s ​disease, a communicable disease formerly called​ leprosy, results from a bacterial infection. Hepatitis A is a communicable disease that is transmitted by the​ fecal-oral route.

The nurse is inspecting the chest of an adult client. Which observations of the respiratory cycle are considered abnormal findings for a healthy ​adult? Select all that apply. Dusky or blue lips Substernal retractions during a respiratory cycle Pink or pale skin tones Respiratory rate of 52​ breaths/min at rest Asymmetry of the chest wall

Dusky or blue lips Substernal retractions during a respiratory cycle Respiratory rate of 52​ breaths/min at rest Asymmetry of the chest wall The normal respiratory cycle of an adult is at a rate between 12 and 20​ breaths/min. The skin is​ pale, pink, or pale with pink undertones. There is no evidence of​ retractions, and the chest appears symmetrical. Asymmetry is considered​ abnormal, as is a respiratory rate of greater than 40​ breaths/min, dusky​ lips, and substernal retractions.

The parents of a toddler with a newly diagnosed ear infection are about to go home from the clinic. Which set of instructions should be a part of parent education to help reduce the risk for recurrent ear​ infections? Click the Exhibit button for additional information. EXHIBIT History LOADING... The toddler presents with crying and pulling at his ears. The parents report that the only way to soothe the toddler is to give him a bottle and lay him flat. The toddler does not finish a prescribed antibiotic if he begins to show improvement because he does not take medicine well. Assessment Findings LOADING... Ear anatomy is normal and appropriate for the​ child's age, the pinnea area is reddened​ bilaterally, and ear wax is present in the canal. The child continues to pull at his ears and cries intermittently. Vital Signs LOADING... The​ child's temperature is​ 100.1°F, heart rate and respiratory rate are within normal​ limits, throat is slightly​ red, tonsils are​ enlarged, and three baby teeth are present. There is no drainage from either​ ear, and the eardrums are​ pink, clear, and without exudate. The variety of hearing tests available Ear anatomy and completing medications Tympanic membrane is the source of infection. Fever control

Ear anatomy and completing medications Instructing the parents on the anatomy of the ear of a toddler may help them to understand that the child drinking from a bottle while lying flat might contribute to the ear infections because of the​ child's short ear canal and the position of the inner ear and nasal passages. It would also be helpful to instruct parents on the importance of the child finishing all of the medications that are prescribed. Instructing the parents on the hearing tests available would not promote fewer ear infections. Instructing the parents on fever control may be part of parent education but would not reduce the risk for recurrent ear infections. The tympanic membrane is where the vibration of the sound waves strikes and is an important aspect of​ hearing, but it is not the only part of the ear that can get infected and it is not where bacteria originate.

The nurse is assessing an adult client. What abnormal findings would indicate cardiac involvement and ​disease? Select all that apply. Edema in the ankles Pink lips and nails Clubbing of fingernails Edema or swelling of the feet Bluish tint or cyanosis to hands and feet

Edema in the ankles Clubbing of fingernails Edema or swelling of the feet Bluish tint or cyanosis to hands and feet Abnormal findings in an assessment could include clubbing of the​ fingernails, edema in the feet or​ ankles, and discoloration of hands or​ feet, lips, or​ skin, and are related to poor heart function. Pink skin​ tones, normal shape to the​ nails, and no edema indicate adequate heart function.

The nurse is preparing to complete documentation on a client. What is the advantage of using the​ problem-oriented medical record​ (POMR) method of​ documentation? Provides efficiency because staff members document in their own specific area Encourages collaboration and easy tracking of client problems Provides documentation of all assessments and interventions Focuses on​ client's concern and strengthens the focus of care

Encourages collaboration and easy tracking of client problems The advantages of the​ problem-oriented medical record​ (POMR) are that it encourages collaboration​ and, because the problem list is easily​ accessible, the tracking of the​ client's problem status is easier. The​ problem-oriented medical record​ (POMR) can be inefficient because assessments and interventions that apply to more than one problem need to be repeated and all health care providers document in the same area. The POMR can be​ time-consuming and requires constant vigilance to maintain an​ up-to-date problem list. Focusing charting is intended to focus on the​ client's concerns and strengthens the focus of care.

The nurse is assessing the heart of an elderly adult client. What heart changes would the nurse​ expect, even in the absence of cardiac ​disease? Select all that apply. Evidence of silent myocardial infarction Enlarged and atrophied heart Thickening of the heart muscle Rigid valves Rigid vessels

Enlarged and atrophied heart Thickening of the heart muscle Rigid valves Rigid vessels The cardiac muscle would undergo changes with​ age, even in the absence of disease. The heart enlarges and becomes atrophied. The heart wall​ thickens, the valves and vessels become​ rigid, and the pump doesn​'t work as well as that in a younger​ person, even in the absence of disease. It can be important to educate the elderly adult about wellness in relation to aging to accommodate these changes in the heart. Aging does not produce silent myocardial infarctions. Educating the client to report chest pain is important.

Upon inspection and palpation of a male client​'s ​breasts, you find that the client has gynecomastia. How is this condition​ described? Enlargement of the breasts Increase in subcutaneous fat Nipple discharge Hard nodules near the nipple

Enlargement of the breasts Gynecomastia is the enlargement of the breasts in a male client. Nipple discharge and hard nodules near the nipple may indicate breast​ cancer, not gynecomastia. An increase in subcutaneous fat in obese males is called pseudogynecomastia.

Which should be included when auscultating body​ sounds? Select all that apply. Ensure a quiet environment during assessment. Instruct client to make audible sounds​ (i.e., cough and​ speak) during assessment. Auscultate the abdomen before palpation and percussion. Use stethoscope against bare skin. Keep client cool during assessment.

Ensure a quiet environment during assessment. Instruct client to make audible sounds​ (i.e., cough and​ speak) during assessment. Auscultate the abdomen before palpation and percussion. Use stethoscope against bare skin. A stethoscope should only be used against bare skin. For greater​ accuracy, it is best to perform auscultation in a quiet environment. Maintaining the privacy and warmth of the client by covering the client with a sheet or blanket also prevents​ shivering, which can mask sounds. Auscultation includes all sounds made by the​ client, including those heard without the use of a stethoscope. Always perform auscultation on the abdomen before palpation and percussion because the latter techniques can alter the natural sounds of the abdomen.

The nurse is preparing to assess a client​'s skin. What layer of skin is exposed to the external​ environment? Subcutaneous Keratin Dermis Epidermis

Epidermis The outermost layer of epithelial tissue is the epidermis and is the part exposed to the external environment. The dermis is a layer of connective tissue that lies just below the epidermis. Keratin is what gives the epidermis its tough and protective qualities. Subcutaneous tissue is loose connective tissue that stores approximately half of the body​'s fat cells.

The nurse is reviewing the structures within the different layers of the skin. In what order would the nurse rank the skin composition from outermost to innermost​ layers? ​Instructions: Click on the dropdown arrow for each skin composition in the right column and select the correct choice from the list. Order Skin Composition Sweat glands Stratum basale Epithelial cells Elastic fibers

Epithelial cells Stratum basale Elastic fibers Sweat glands

The nurse is assessing a pregnant female with complaints of ear pressure and​ pain, but not ear infection. What causes the client​'s pressure in the​ ears? Estrogen levels cause an increase in vascularity. Sodium retention Bleeding of the tympanic membrane Fluid buildup

Estrogen levels cause an increase in vascularity. An increase in vascularity due to increased estrogen levels includes vascular changes in the middle​ ear, which causes a feeling of fullness. There is no extra fluid buildup during​ pregnancy, but vascular changes may occur. Sodium retention during pregnancy does not interfere with hearing or pressure in the​ ears, but may cause peripheral swelling in the hands and feet. Bleeding of the tympanic membranes would not occur as a result of pregnancy.

Which of the following are components of a client​'s psychosocial ​history? Select all that apply. Ethnicity Educational level Birthplace Financial background Address

Ethnicity Educational level Financial background Information about a client​'s educational​ level, ethnicity and financial background is part of the client​'s psychosocial history and provides information about previous experiences that may affect the client​'s health. A client​'s birthplace and address are components of the client​'s biographical data.

When assessing a​ client's description of​ pain, the nurse must ask for which of the following​ descriptions? Select all that apply. Opinion Etiology Location Duration Intensity Emotion

Etiology Location Duration Intensity When assessing a​ client's pain, the nurse must gather information about the​ Location, Duration,​ Intensity, and Etiology.​ Location: have the patient point to each place hurting.​ Duration: the length of the experience distinguishes acute from chronic pain.​ Intensity: use a scale and find out aggravating and alleviating​ factors, and​ Etiology: what caused the pain in the first place. Emotion and Opinion are not a part of the pain description assessment.

People of what culture are known to have the smallest​ teeth? Asian European Alaska native Australian aboriginal

European It is important to note cultural and environmental differences when assessing clients. The size of the teeth varies among​ cultures, and the smallest teeth are seen in people of European descent.​ Asians, Alaska​ natives, and Australian aboriginals have the largest teeth.

Which of the following refers to the movement of the foot whereby the sole of the foot is turned​ laterally? Supination Eversion Adduction Inversion

Eversion Eversion is a joint movement in which the sole of the foot is laterally turned. Adduction is the movement of a limb toward the midline of the body. Inversion occurs when the sole of the foot is turned medially. Supination refers to movement of the radius around the ulnar bone when the palm moves to face anteriorly or superiorly.

The nurse is discussing exercise with an adult client who leads a sedentary lifestyle. What positive effects on the musculoskeletal system would the nurse include when encouraging the client to become more physically ​active? Select all that apply. Exercise promotes healthy​ weight, decreasing stress on​ weight-bearing joints. Exercise prevents osteoporosis. Exercise increases muscle strength Exercise for the adult decreases muscle​ mass, making the adult weak and prone to immobility. Activity increases the chance of falling

Exercise promotes healthy​ weight, decreasing stress on​ weight-bearing joints. Exercise prevents osteoporosis. Exercise increases muscle strength The nurse would encourage an adult with a sedentary lifestyle to increase activity and exercise. Activity and exercise increase muscle​ mass, increase muscle​ strength, help to prevent​ osteoporosis, and improve balance. Exercise and activity for an adult would improve the overall musculoskeletal health and help to prevent falls from a weak musculoskeletal system.

Arrange the procedure for performing palpation in the correct sequence from start to finish. ​Instructions: Click on the dropdown arrow for each step of the procedure in the right column and select the correct choice from the list. Order Procedure Explain procedure to client. Observe area for guarding or rigidity. Assist client into comfortable position. Carefully palpate known painful areas. Palpate slowly with deliberate movements.

Explain procedure to client. Assist client into comfortable position. Palpate slowly with deliberate movements. Observe area for guarding or rigidity. Carefully palpate known painful areas.

A mildly underweight client is at the clinic for a routine assessment. What findings upon your physical assessment indicate a cardiac​ problem? Select all that apply. External jugular vessel visible when in sitting position Pulsation noted to the epigastric area Bilateral periorbital edema No pulsation or heaves noted to palpation at the second intercostal space Whitish sclera

External jugular vessel visible when in sitting position Bilateral periorbital edema Periorbital edema indicates water​ retention, which is a potential sign of a cardiac problem. The external jugular vessel should not be visible unless the client is lying down. Whitish sclera and a lack of pulsations and heaves to the second ICS are expected findings. Pulsations to the epigastric area in an underweight client are considered normal.

The nurse is preparing to assess the​ client's sense of taste. What cranial nerves would the nurse ​assess? Select all that apply. Olfactory Facial Hypoglossal Abducens Glossopharyngeal

Facial Glossopharyngeal The facial nerve is responsible for taste​ (anterior two-thirds of the​ tongue); facial movement such as​ smiling, closing of​ eyes, and​ frowning; production of​ tears; and salivary stimulation. The glossopharyngeal nerve is responsible for taste​ (posterior third of the​ tongue) and produces the gag and swallowing reflexes. The olfactory nerves are responsible for the sense of smell. The abducens nerve is responsible for the extrinsic muscle movement of eye. The hypoglossal nerve is responsible for movement of the tongue for swallowing and for movement of food during chewing and speech.

What are the normal characteristics of the testicles when ​palpated? Select all that apply. Feel smooth and solid Spongy and tender About the size of a walnut Elastic and tender Oval in shape

Feel smooth and solid About the size of a walnut Oval in shape The testicles are oval in shape and are the size of a​ walnut, although one may be slightly larger than the other. The testicles feel smooth and solid when palpated. They are elastic and firm and should be nontender.

Ms. Baransky is a​ 25-year-old woman who has come to the clinic for a breast examination. When you palpate the right​ breast, you detect a​ firm, round, moveable mass about 2 cm in diameter. What does this​ indicate? Fibrocystic disease Carcinoma Mammary duct ectasia Fibroadenoma

Fibroadenoma A fibroadenoma is characterized as a​ firm, round, freely moving mass of 1 to 5 cm that usually appears as a single tumor. Carcinoma of the breast can be characterized by dimpling skin over a​ tumor, alignment deviation of the breast or​ nipple, irregular breast​ shape, edema, or nipple discharge. Mammary duct ectasia is an inflammation of the lactiferous ducts. Fibrocystic disease is marked by masses that are​ soft, well​ demarcated, and freely moving with associated pain and tenderness.

The nurse is assessing the abdomen of an adult and is preparing to percuss the liver. What would be included in the process of assessing the liver of an adult ​client? Select all that apply. Finding that the lower border of the liver descends approximately 2.5 cm below the level of the ribs Explaining the procedure and informing the client that the nurse will be making tiny dots on the client​'s abdomen Having difficulty assessing the size of the liver by percussion Assessing for the upper and lower borders of the liver to determine liver size Finding the borders of the liver at the axillary​ line, the midclavicular​ line, and the midsternal line on the right upper quadrant of the abdomen

Finding that the lower border of the liver descends approximately 2.5 cm below the level of the ribs Explaining the procedure and informing the client that the nurse will be making tiny dots on the client​'s abdomen Assessing for the upper and lower borders of the liver to determine liver size Finding the borders of the liver at the axillary​ line, the midclavicular​ line, and the midsternal line on the right upper quadrant of the abdomen It is important to explain the procedure to the client so the client is relaxed and understands what is happening during the exam. The nurse would assess the size of the liver when percussing this client​'s abdomen and would find the borders of the liver at the axillary​ line, the midclavicular​ line, and the midsternal line on the right upper quadrant of the abdomen. The lower border of the liver may descend below the level of the rib but not more than 2.5 cm. This would depend on the disease​ process, age, and height of the client. Percussion is one of the accepted ways to determine liver size because of the liver​'s location in the abdomen.

The nurse is preparing to palpate the client​'s abdomen. Which technique would the nurse use to best palpate organs of the​ abdomen? Dorsal surface of the hand Finger pads Ulnar surface of the hand Bases of the fingers

Finger pads The pads of the fingers are best used for discrimination of underlying structures and used when palpating the abdomen for organs. The dorsal surface of the hand is not used when palpating the abdomen. The bases of the fingers are best used for feeling vibrations. The ulnar surface of the hand is sensitive to vibrations as in fremitus.

What characteristics pertain to fibrocystic breast​ disease? Select all that apply. First seen in females in their 20s Elimination of caffeine may bring some relief. Cyst formation due to fluctuating hormones Linked to breast cancer Masses are hard and fixed.

First seen in females in their 20s Elimination of caffeine may bring some relief. Cyst formation due to fluctuating hormones Fibrocystic breast disease is first seen in females in their 20s. Fluctuating hormones cause cyst formation. Elimination of caffeine may bring some relief from pain and other symptoms. Benign breast disease is not linked to breast​ cancer, and the masses associated with it are soft and move freely.

What findings would be normal when inspecting the flank areas of a female ​client? Select all that apply. There is a slight elevation where the kidneys lie on either side of the spinal cord. Flank areas are not elevated. No masses or lumps are visible. Skin color is uniform. Skin color is slightly red.

Flank areas are not elevated. No masses or lumps are visible. Skin color is uniform. The flank area should be free of visible​ lumps, masses, or elevation. Uniform skin color over the flank areas that is free of bruising or discoloration would be a normal finding. Skin color should be consistent with the rest of the skin and without bruising or reddening. There should be no elevation in the area of the kidneys. The kidneys lie within the abdominal cavity and are protected from injury by thick muscles so they should not be visible.

The nurse has just completed the client​'s vital signs and prepares to document this information. Which documentation form should the nurse​ use? Problem list Nursing care plan Flow sheet Progress note

Flow sheet Flow sheets are used to document assessment including vital signs and interventions. The problem list is used to document nursing diagnoses. The plan of care is used to document the client​'s care needs. A progress note is used to record the client​'s response to​ treatment, but it is not the primary place to document vital signs.

The nurse at the primary care clinic is clarifying a new​ client's complaint of foot pain. Which type of assessment is the nurse​ completing? ​Cross-examination Focused interview Preinteraction chart review Initial assessment interview

Focused interview A focused interview clarifies a​ client's symptoms or complaints. This is an example of Phase III of the health history. Preinteraction or Phase I of the health history involves review of records and other objective​ data, as well as old health assessments if available. The initial assessment interview is phase II of the health history and is a general and inclusive interview of the​ client's health and family history and lifestyle.​ Cross-examination is a barrier to effective nursedashclient communication and is not part of the health history.

Mrs. Campbell is an​ 80-year-old woman who has been brought into the clinic for a leg injury. She keeps repeating that her leg is​ "hurting" her. Which communication skill would you use to get more​ information? Reflecting Focusing Summarizing Paraphrasing

Focusing Focusing helps the client target a specific issue. Paraphrasing reiterates the​ client's statements to test whether it was understood. Reflecting lets the client know that you empathize and are taking the​ client's feelings into consideration. Summarizing allows you to tie together all the information that has been gathered about the client.

What is an abnormal finding when palpating a male​ client's breast? Breast that is flat No tenderness Breast that feels like a thin disc of tissue Hard nodule

Hard nodule A hard nodule fixed to the nipple and underlying tissue is often an indication of breast cancer in a male client. The normal male breast would look​ flat, feel like a thin disc of​ tissue, and be nontender.

The nurse is reviewing orders written by the health care provider and sees the following​ order: Administer furosemide 40 mg QD using the liquid form of medication of 10​ mg/mL. To ensure the client​'s ​safety, for which reason would the nurse contact the health care provider for further​ clarification? To obtain the brand name of the medication Use of mL instead of ml For clarification of QD The use of mg

For clarification of QD The abbreviation QD can be misinterpreted and lead to an​ error, so use of this abbreviation would require further clarification to ensure it is correctly interpreted. The abbreviation open double quote"mgclose double quote" is acceptable and would not require further clarification. The use of mL is the acceptable way to abbreviate milliliters. Use of the generic names of medications is an acceptable and safe practice.

Bobby Castro is a​ 6-year-old who was brought to the emergency department with respiratory difficulties. You notice that he has a continuous​ high-pitched wheeze. What does this condition​ indicate? Pleural inflammation Croup ​Fluid-blocked airways Foreign body obstruction

Foreign body obstruction A​ high-pitched wheeze on expiration and inspiration may indicate a foreign body obstruction. A client with​ fluid-blocked airways would present with a​ low-pitched continuous​ "snoring" sound on expiration and inspiration. Croup is indicated by a​ loud, high-pitched crowing sound on inspiration. Pleural inflammation is indicated by a​ low-pitched grating, rubbing sound on inhalation and exhalation.

The nurse is assessing a child after a motor vehicle crash​ (MVC) and suspects a traumatic injury to the forearm. What type of traumatic injury would occur if the radius and ulna had a complete​ separation? Dislocation Fracture Joint strain Muscle strain

Fracture The radius and ulna are bones in the forearm. Trauma can cause a partial or complete​ break, or​ fracture, in the continuity of the bone. Muscle strains and joint strains occur when muscles and tendons have been stretched beyond their normal capacity of movement. A dislocation is the displacement of the bone from its normal position.

Mrs.​ Maloney, a new​ mother, comes to the clinic for her​ baby's first checkup. Upon palpating the​ baby, you notice a bump along his clavicle. What is the probable​ condition? Scoliosis Fracture from birth Tibial torsion Genu valgum

Fracture from birth The clavicle is frequently fractured during​ birth, but the fracture often goes unnoticed until a callus forms at the fracture site. Tibial torsion is an inward twisting of the tibia. Genu valgum is​ knock-knees, which often appears at age 4 years. Scoliosis is a curvature of the spine.

The nurse is completing a health history for a client in the hospital. What action by the nurse indicates the need for additional education about how to properly conduct a health​ history? Conducts the interview in the​ client's room Frequently takes notes and writes down all the​ client's answers Listens attentively to the client during the interview. Asks the client to turn TV off during the interview

Frequently takes notes and writes down all the​ client's answers The nurse would take only necessary notes as it breaks eye contact and interrupts the​ client's narrative flow. It also takes the​ nurse's attention off of the​ client's nonverbal behaviors. The nurse would control the environment during the interview. The environment would be free of interruptions from other staff members or health care professionals. Conducting the interview in the​ client's room provides privacy for the client. Listening is active and demanding during the interview process for a health history and positively affects the interview.

The nurse is obtaining biographical information from the primary source during the health history. Which is the best way to obtain this​ information? From a friend From the client From a family member From a health care professional

From the client The client is the primary source of information and is the best source of information. The nurse would gather health history information from the client unless the client is unable to communicate with the nurse. A family member or friend is a secondary source of information. Secondary sources would be used to complement or add to the health history. Prior health care professionals can be used as secondary sources of information or to gather information about past​ care; however, the client is the best source to provide biographical information.

The charge nurse is assisting the student nurse in understanding the abdominal assessment of an adult client. Using the techniques of palpation and​ percussion, it may be possible to estimate the size and location of which of the following ​organs? Select all that apply. Gallbladder Pancreas Full Bladder Liver Spleen

Full Bladder Liver Palpation and percussion may be used to determine the liver size. The gallbladder stores bile and lies on the ventral side of the liver and is not palpable. The spleen is normally not palpable. If the bladder is​ full, it may be possible to determine the bladder size by percussion​ and/or palpation. The pancreas is nonpalpable because of its size and location.

Which of the following abnormal chest configurations are congenital​ deformities? Select all that apply. Barrel chest Scoliosis Funnel chest​ (pectus excavatum) Kyphosis Pigeon chest​ (pectus carinatum)

Funnel chest​ (pectus excavatum) Pigeon chest​ (pectus carinatum) Funnel chest is a congenital deformity characterized by depression of the sternum and adjacent costal cartilage. Pigeon chest is a congenital deformity characterized by forward displacement of the sternum with the depression of the adjacent costal cartilage. Barrel chest occurs normally with aging. Scoliosis is a condition in which there is lateral curvature and rotation of the thoracic and lumbar spine. Kyphosis is an exaggerated posterior curvature of the spine associated with aging.

The nurse is inspecting an adult client who has experienced trauma to the nose. What symptom represents trauma to the​ nose? Noisy breathing Stuffy feeling Clear drainage Inability to breathe through each nostril

Inability to breathe through each nostril An inability to breathe through the nose indicates trauma or illness. Clear drainage or a stuffy feeling may indicate a cold virus or​ allergy, but not necessarily trauma. Noisy breathing would indicate an obstruction or​ infection, but not necessarily trauma.

The nurse is assessing an adult client who is not pregnant but has unilateral discharge from the breast. What abnormal findings would this symptom ​represent?​ Select all that apply. Galactorrhea Malignancy Peau d​'orange Intraductal papilloma Benign breast disease

Galactorrhea Malignancy Intraductal papilloma Benign breast disease Benign cystic breast disease can be the reason for unilateral breast discharge. Intraductal papilloma can cause discharge from the nipple of one breast. Breast​ cancer, or​ malignancy, can be the abnormal finding if there is unilateral discharge from the nipple. Galactorrhea is the abnormal lactation not associated with breastfeeding that will cause a discharge from the nipple. Peau d​'orange is swelling in the breast caused by blocked lymphatic drainage and is not associated with nipple discharge.

Mrs. Jones is a​ 52-year-old woman who came into the clinic with pain in the right upper area of the abdomen. Which of the following organs is located in that​ area? Gallbladder Descending colon Stomach Spleen

Gallbladder The gallbladder is located in the right upper quadrant. The spleen is located in the left upper quadrant. The descending colon is located in the left upper and left lower quadrants. The stomach is located in the left upper quadrant.

When gathering a client​'s health​ history, what information falls into the category of biographical​ data? Select all that apply. Gender Present health history ​Age/date of birth Occupation Family history

Gender ​Age/date of birth Occupation A client​'s ​age/date of birth enables you to begin to evaluate characteristics in relation to the norms across the age span. There are differences according to gender in terms of physical​ development, secondary sex​ characteristics, and reproduction. A client​'s occupation may unearth​ physical, psychological, and environmental factors associated with the client​'s health. A client​'s present health history and family history are essential in building the client​'s health history but are not part of the client​'s biographical data.

The nurse has discovered a possible abnormality in the thyroid of an adult during palpation of the neck. What thyroid conditions might an abnormal thyroid exam ​indicate? Select all that apply. Goiter Congenital hypothyroidism Myxedema Adenomas Graves disease

Goiter Myxedema Adenomas Graves disease Goiter is an enlarged thyroid for an unknown reason. Medications may need to be prescribed if thyroid levels are too high or too low. Graves disease is an increased secretion of thyroid hormone and is the result of an overactive thyroid. Adenomas of the thyroid are abnormal nodules on the thyroid gland. Myxedema is severe hypothyroidism causing general edema and thickening of facial​ features; it can cause coma and death. Congenital hypothyroidism is a condition of an underactive thyroid present since birth. It would be discovered before​ adulthood, so it is not a condition the nurse would consider in the client.

Which of the following may indicate abnormal findings after palpation of the client​'s auricle​ and/or tragus? Select all that apply. Gout Otitis externa Cerumen accumulation Temporomandibular joint dysfunction Mastoiditis

Gout Otitis externa Temporomandibular joint dysfunction Otitis externa​ (infection of the external​ ear) is indicated when a client has pain upon palpation of the auricle and tragus. Gout may be indicated when hard​ nodules, which are uric acid crystal​ deposits, are found upon palpation of the auricle and tragus. Pressing on the tragus may reveal temporomandibular joint dysfunction. Mastoiditis may be indicated if there is a pain when the mastoid process is palpated. Cerumen accumulation may be found upon inspection of the auditory canal with an otoscope.

Mrs. Castelluccio is a​ 65-year-old woman who came to the clinic for her regular hearing test. The exam revealed she has presbycusis. What does she suffer​ from? Protrusion of the tongue Gradual hearing loss Decrease in saliva production Hyperplastic overgrowth

Gradual hearing loss Gradual hearing loss is termed presbycusis and is common among older adults. Hyperplastic overgrowth that forms a mass on the gums is called epulis and is common in pregnant women. A decrease in saliva production is an​ age-related condition that is common in older adults. Protrusion of the tongue is a result of senile tremors and is also common in older adults.

The nurse is caring for a client after hip replacement surgery. The client has just been medicated for reports of pain. What​ non-pharmacologic interventions can the nurse use to help this client with pain ​relief?​ Select all that apply. Tricyclic antidepressant Guided imagery Massage Acupuncture Transcutaneous electrical nerve stimulation​ (TENS)

Guided imagery Massage Acupuncture Transcutaneous electrical nerve stimulation​ (TENS) Transcutaneous electrical nerve stimulation​ (TENS) involves the use of electrical nerve stimulation to help reduce pain impulses and is an example of a​ non-pharmacologic pain intervention. Massage and acupuncture are forms of complementary pain management that can be used in conjunction with pain medication to help reduce pain. Guided imagery is a form of​ non-pharmacological pain management. Tricyclic antidepressants are​ co-analgesics used in conjunction with pain medication for relief of pain. They are a pharmacologic intervention.

The nurse is new to the hospital setting and is reviewing OSHA​ (Occupational Safety and Health​ Administration) standards recommended for hospital workers. What selections are standards recommended by OSHA for the protection of health care ​workers? Select all that apply. Guidelines for the disposal of hazardous wastes Regulation of pay rates for employees Protective equipment to prevent exposure from bloodborne pathogens Needleless systems used throughout the hospital Occupational standards to prevent exposure to tuberculosis

Guidelines for the disposal of hazardous wastes Protective equipment to prevent exposure from bloodborne pathogens Needleless systems used throughout the hospital Occupational standards to prevent exposure to tuberculosis OSHA recommends using needleless systems to prevent needle stick injuries to health care workers. The use of protective equipment such as​ gloves, gowns, and masks is a recommended safety guideline from OSHA. OSHA standards suggest a set of guidelines for hospitals to put into place to protect workers from exposure to​ tuberculosis, and OSHA has specific guidelines for health care facilities to handle hazardous waste material. OSHA offers guidelines for facilities to follow to protect the worker but does not offer guidelines for pay scales.

The nurse is assessing a female adult client with a history of an autoimmune disease that results in severe hypothyroidism. What is the name of the inflammatory disease that is more common in females than males and results in this​ condition? Graves disease Hashimoto thyroiditis Cancer of the thyroid Goiter

Hashimoto thyroiditis Hashimoto thyroiditis is the autoimmune disease of the thyroid that causes severe hypothyroidism. Goiter is an enlarged thyroid that can cause either hypothyroidism or hyperthyroidism. Graves disease is severe hyperthyroidism. Cancer of the thyroid may follow radiation to the thyroid or severe​ goiter; it is not the result of an autoimmune disease.

The nurse is educating a family about home fire prevention. What would be included in a discussion about home ​safety? Select all that apply. Have a home evacuation plan. Change batteries in smoke alarms every 6 months. Use smoke detectors in the home. Ensure that all light bulbs are functioning. Avoid cooking when sleepy or tired.

Have a home evacuation plan. Change batteries in smoke alarms every 6 months. Use smoke detectors in the home. Avoid cooking when sleepy or tired. Having a home evacuation plan ensures that family members know what to do in case of a fire emergency. Educating the family on avoiding cooking when tired helps prevent cooking fires. Smoke detectors warn family members of fire or smoke and allow time for​ evacuation, and changing their batteries every 6 months ensures that they function appropriately. Ensuring that light bulbs are functioning would not be part of a discussion about fire prevention.

Mr. Levine is a​ 68-year-old man who is in the clinic for a neurologic assessment. What symptom is not simply an​ age-related variation and may indicate a degenerative​ disorder? Shuffling gait Slower coordination Head bobbing Impaired sense of hearing

Head bobbing In an older​ adult, slower​ coordination, a shuffling​ gait, and an impaired sense of hearing are all indications of a normal decrease in neurologic functioning. Head bobbing can indicate​ Parkinson's disease, a disorder caused by the degeneration of the nuclei of the brain resulting in symptoms such as head​ bobbing, tremors, and difficulty walking and maintaining balance.

The nurse is assessing the ears of an unconscious adult and finds that the tympanic membrane has a bluish tinge. What is the most common cause of this abnormal​ finding? Perforated eardrum Head trauma Infected outer ear Otitis media

Head trauma Head trauma is suspected because a bluish tinge to the eardrum indicates blood behind the middle ear. Otitis​ externa, or infection of the outer​ ear, involves​ itching, fever, and enlarged lymph glands and is not represented by a bluish tympanic membrane. A​ red, bulging eardrum indicates otitis media. A dark​ spot, not a bluish​ tinge, on the eardrum seen with an otoscope indicates a perforated eardrum.

The adult client presents to the clinic asking for antibiotics for sinusitis. What are the common symptoms of​ sinusitis? Select all that apply. Headache Pressure in the cheeks Facial pain Stiff neck Pain in the teeth

Headache Pressure in the cheeks Facial pain Pain in the teeth Facial pain over the area of the​ sinuses, headache due to sinus​ congestion, pressure and pain in the area of the​ cheeks, and pain and sensitivity in the teeth and jaw are common symptoms of sinusitis. A stiff neck is not a common symptom of sinusitis.

Which of the following factors generally results in a decreased pulse rate when compared to normal values for the adult​ population? Fever Acute blood loss Being an infant or child Healthy teenager

Healthy teenager The expected pulse rate of a healthy teenager is 50dash-90 beats per​ minute, whereas an adult​'s is typically 60dash-100 beats per minute. An infant or​ child, client with a​ fever, or client with acute blood loss will generally have an increased pulse rate compared to the general adult population.

Identify the percussion tone based on its characteristics. ​Instructions: Use the dropdown menus in the left column to select the correct percussion tone for each characteristic in the right column. Percussion Tone Characteristic Hyperresonance Tympany Dullness Resonance Heard over the lungs Heard over solid body organs such as the liver Heard over lungs with trapped air ​Loud, high-pitched,​ drum-like tone ​High-pitched soft tone with a short duration ​Loud, low-pitched, hollow tone with long duration Abnormally​ loud, low tone Heard over​ air-filled intestines

Heard over the lungs-resonance Heard over solid body organs such as the liver- dullness Heard over lungs with trapped air- hyperresonance ​Loud, high-pitched,​ drum-like tone- tympany ​High-pitched soft tone with a short duration- dullness ​Loud, low-pitched, hollow tone with long duration- resonance Abnormally​ loud, low tone- hyperresonance Heard over​ air-filled intestines- tympany

Located in the middle thoracic​ cavity, the mediastinum contains what​ structures? Select all that apply. Lungs Heart Pleural membranes Esophagus Trachea

Heart Esophagus Trachea The mediastinum contains the​ esophagus, trachea, and​ heart, as well as the great vessels. The lungs are situated in the pleural cavities of the thorax on either side of the mediastinum. The pleura are two​ serous-filled membranes that surround each lung cavity.

The nurse is assessing an adult female with abdominal pain. What types of abnormal abdominal pain can be confused with pain related to the heart because of the location and description of the ​pain? Select all that apply. Heartburn from gastroesophageal reflux disease​ (GERD) Pancreatitis Gastritis Ectopic pregnancy Duodenal ulcer

Heartburn from gastroesophageal reflux disease​ (GERD) Pancreatitis Gastritis Duodenal ulcer GERD, or gastroesophageal reflux​ disease, can be confused with a heart attack because of the symptoms of heartburn and chest pain. A duodenal ulcer creates a gnawing pain in the abdomen that may cause nausea. Gastritis is​ acute, severe epigastric pain below the lower half of the sternum and the xiphoid process. Pancreatitis creates a​ knifelike, stabbing pain in the right upper quadrant​ (RUQ) and can radiate to the back with nausea. An ectopic pregnancy presents with lower abdominal pain and fullness in the rectal area. It is not related to pain in the chest.

The nurse is aware that clients who get tattoos are at greater risk for developing what health ​alterations? Select all that apply. Scar tissue formation Hepatitis Herpes zoster HIV Hepatitis A

Hepatitis HIV Tattoos can cause skin​ irritation, and the process of tattooing puts the client at risk for​ infection, hepatitis​ C, and HIV. Hepatitis​ A, herpes​ zoster, and scar tissue formation are not risks with tattooing.

A​ routine, complete nutritional assessment includes which of the following​ components? Select all that apply. Herbal supplements Socioeconomic status Family medical history Health Insurance Current medications

Herbal supplements Socioeconomic status Current medications A complete nutritional assessment focuses on the​ client's nutritional status. Because medications and herbal supplements can influence​ over- and​ undernutrition, and socioeconomic status may influence the food a person is able to​ obtain, these are important components of the assessment. The family medical history and health insurance are not directly applicable to a routine nutritional assessment.

An older adult male Caucasian client is admitted to the medical unit. Which psychosocial factors are likely to hinder adequate management of this client​'s ​pain? Select all that apply. Ethnicity History of chronic pain Anxiety about changes in lifestyle Lack of social support systems Concern about dependency on medications

History of chronic pain Anxiety about changes in lifestyle Lack of social support systems Concern about dependency on medications A history of chronic​ pain, lack of social support​ systems, and anxiety about changes in lifestyle can negatively affect satisfactory pain management. For​ example, a client with a history of poorly managed chronic pain may experience heightened anxiety and may possibly anticipate inadequate pain relief. Worry about his ability to care for himself at home may also​ compound/intensify his pain. Fear of becoming dependent on potentially addictive medications may interfere with effective pain management as well. Ethnicity certainly plays a part in the pain experience as some ethnic groups are very stoic and do not admit to having​ pain; however, this client is Caucasian and more likely to communicate information about pain.

The nurse is caring for a client who is being admitted for new onset of seizures. What assessment findings could contribute to the development of ​seizures? Select all that apply. Chronic obstructive pulmonary disease History of sedative use Drug overdose Takes antidepressant medication Recent infection

History of sedative use Drug overdose Takes antidepressant medication Recent infection A recent​ infection, drug or alcohol​ overdose, sedative​ use, and antidepressant medications are all potential causes of seizures. Chronic obstructive pulmonary disease is not linked to the development of seizures.

Identify the physical assessment test of hearing acuity and equilibrium based on the procedure. ​Instructions: Use the dropdown menus in the left column to select the correct hearing acuity test for each description in the right column. Category Procedure Romberg Test Weber Test Hold tuning fork by handle and strike against palm of your hand. Evaluate​ client's ability to maintain position. Ask client if sound is heard equally on both sides or better in one ear. If client has difficulty maintaining a balanced​ state, document positive test. Uses bone conduction to evaluate hearing in person who hears better in one ear Ask client to stand with feet together and arms at​ sides, first with eyes open and then closed. Sound heard equally is normal​ response, which is documented as​ "no lateralization." Assesses equilibrium Place base of vibrating fork against middle of​ client's frontal bone. Mild swaying is documented as negative test.

Hold tuning fork by handle and strike against palm of your hand.-Weber Evaluate​ client's ability to maintain position.-Romberg Ask client if sound is heard equally on both sides or better in one ear.-Weber If client has difficulty maintaining a balanced​ state, document positive test.- Romberg Uses bone conduction to evaluate hearing in person who hears better in one ear- Weber Ask client to stand with feet together and arms at​ sides, first with eyes open and then closed.- Romberg Sound heard equally is normal​ response, which is documented as​ "no lateralization."- Weber Assesses equilibrium- Romberg Place base of vibrating fork against middle of​ client's frontal bone.- Weber Mild swaying is documented as negative test.- Romberg

The nurse is assessing an adult with severe compromise to the musculoskeletal system and impaired mobility. What psychosocial conditions would the nurse consider when assessing the ​client? Select all that apply. Hopeless feelings Ineffective coping Depression Congenital heart defect Anxiety

Hopeless feelings Ineffective coping Depression Anxiety The nurse assessing an adult client with an impaired musculoskeletal system would assess the client for ineffective​ coping, stress,​ hopelessness, and anxiety that would interfere with activities of daily living. The nurse is responsible for assessing both the emotional and physical status when assessing an adult client. Heart defects are repaired in early infancy and are a physical​ concern, not a psychosocial concern

dentify the shapes of the bones of the body. ​Instructions: Use the dropdown menus in the left column to select the shape for each bone in the right column. Shape Bone Irregular Bones Flat Bones Short Bones Long Bones Humerus Sternum Tarsals Femur Carpals Parietal bone of the skull Vertebrae Ribs Hip bones

Humerus- long Sternum- flat Tarsals- short Femur- long Carpals- short Parietal bone of the skull- flat Vertebrae- irregular Ribs- flat Hip bones- irregular

What condition that affects pregnant women is referred to as chloasma​ gravidarum? Hyperpigmentation of the face Cutaneous skin tags Stretch marks Dark line extending from umbilicus to pubic area

Hyperpigmentation of the face Hyperpigmentation of the face is known as chloasma​ gravidarum, as well as melasma gravidarum or mask of​ pregnancy, and disappears after pregnancy in some women but may be permanent in others. A dark line extending from the umbilicus to the pubic area is known as linea nigra. Cutaneous skin tags are tags of nonmalignant skin tissue usually seen on the neck and upper chest. Stretch marks are called striae gravidarum and usually fade after pregnancy.

The nurse in the emergency department is assessing an adult client with emphysema. What symptoms of emphysema would NOT be obvious from inspection and direct observation by the​ nurse? ​Pursed-lip breathing and clubbing of fingers Hyperresonance sounds from the lungs Barrel chest Shortness of breath

Hyperresonance sounds from the lungs Clients with emphysema and COPD would have hyperresonance sounds during an assessment using percussion. These sounds would not be obvious on inspection or observation. A client with emphysema may be obviously short of​ breath, even at rest. A barrel chest is common in those with emphysema and would be obvious on​ inspection, even if the client has a shirt on.​ Pursed-lip breathing and clubbing of the fingers can be observed without​ palpation, auscultation, or percussion.

When completing an assessment of the client​'s cranial​ nerves, which nerves would the nurse recognize as motor ​nerves? Select all that apply. Olfactory Optic Hypoglossal Accessory Trochlear

Hypoglossal Accessory Trochlear The​ hypoglossal, trochlear, and accessory nerves are motor nerves. The olfactory and optic nerves are sensory nerves.

The nurse is caring for a client who is unable to regulate body temperature. What part of the brain is responsible for temperature​ regulation? Cerebellum Hypothalamus Cerebrum Thalamus

Hypothalamus The hypothalamus functions as a thermostat for the body. It sets a desired body temperature and stimulates heat production or retention to raise the blood temperature to a higher setting or sweating and vasodilation to cool the blood to a lower temperature. The cerebellum is responsible for skeletal muscle coordination and smooth movements. It is also involved in some cognitive​ functions, such as attention and​ language, and in regulating fear and pleasure responses. The thalamus is a​ large, dual-lobed mass of gray matter buried under the cerebral cortex. It is involved in sensory perception and regulation of motor function. The cerebrum lies in front of the brain stem and is the largest and most​ well-developed area of the brain.

Mr. Ramirez is a​ 48-year-old man who came into the clinic with concerns about body temperature fluctuations. What central nervous system structure is responsible for body​ temperature? Brain stem Thalamus Cerebellum Hypothalamus

Hypothalamus The hypothalamus is the thermostat of the body. It sets a desired body temperature and regulates either heat production or retention to raise the blood temperature or increase sweating and vasodilation to cool the blood. The thalamus is involved in sensory perception and regulation of motor function. The cerebellum is responsible for skeletal muscle coordination and smooth movements. It is also involved in some cognitive functions such as attention and language and in regulating responses to fear and pleasure. The brain stem coordinates motor control signals from the brain to the body and also controls​ life-supporting autonomic functions of the peripheral nervous system.

The nurse is assessing the head and neck of an adult client with complaints of weight​ gain, fatigue, and constipation. The nurse palpates the thyroid and finds no abnormality. What abnormal condition is associated with these symptoms and a​ normal-feeling thyroid​ gland? Pneumothorax Hypothyroidism Hyperthyroidism Lymph node enlargement

Hypothyroidism Hypothyroidism is an underproduction of thyroid hormone. The thyroid gland may feel normal at​ first, but the symptoms of weight​ gain, fatigue, and constipation may be related to low production of thyroid​ hormone, which is determined with a blood test. Pneumothorax is an abnormality of the lung that pushes the trachea one way or the other. Lymph node enlargement is due to infection or illness and is not associated with weight gain or constipation. In hyperthyroidism the thyroid may feel normal on​ palpation, but hyperthyroidism is associated with weight loss because of an increase in metabolism.

The nurse is caring for a client in the hospital after back surgery. Which of the following are pharmacological interventions for ​pain? Select all that apply. Spinal cord stimulator Ibuprofen Tricyclic antidepressants Morphine Relaxation

Ibuprofen Tricyclic antidepressants Morphine Morphine is an opioid analgesic used to treat pain. Ibuprofen is a​ non-steroidal anti-inflammatory medication used for the treatment of pain and inflammation. Tricyclic antidepressants are​ co-analgesics that are used in the treatment of pain. A spinal cord stimulator and relaxation are examples of​ non-pharmacological interventions for pain.

The nurse is preparing to assess the client​'s cranial nerve I. What finding indicates an intact cranial nerve​ I? Identifies the smell of an orange Swallows with ease Has​ 20/20 vision Pupils reactive to light

Identifies the smell of an orange A client with an intact cranial nerve I​ (olfactory) would be able to correctly identify smells. Intact vision is an example of the function of cranial nerve II. Pupil reaction is a function of cranial nerve III. Swallowing is a function of cranial nerve X.

What assessments would the nurse include when assessing a​ client's sensory ​function? Select all that apply. Identify vibrations. Identify temperature. Test kinesthesia. Identify sharp from dull. Romberg test.

Identify vibrations. Identify temperature. Test kinesthesia. Identify sharp from dull. The ability to identify the touch of a sharp edge and the touch of a dull edge at various​ locations, the correct temperature of a vial of water when​ touched, when a vibration is felt and when the vibration is no longer​ felt, and movements of the great toe would be assessed for sensory function. The Romberg test is a motor function test that assesses coordination and equilibrium and would not be included when assessing sensory perception.

The nurse is assessing the nutritional status of an obese preschool child. What would the nurse be particularly aware of in terms of nutritional risk for this ​child? Select all that apply. The child will have lifelong problems with hypercholesterolemia. If a parent is​ obese, the child​'s risk for adult obesity is greater. The child is at high risk for chronic disease. The child is at high risk for adult obesity. The child is at risk for type 1 diabetes mellitus.

If a parent is​ obese, the child​'s risk for adult obesity is greater. The child is at high risk for chronic disease. The child is at high risk for adult obesity. Childhood obesity increases the child​'s risk for chronic disease. The child is at high risk for adult obesity and type 2 diabetes​ mellitus, not type 1 diabetes. An obese child with one obese parent is at high risk for adult obesity. Although an obese child is at high​ risk, there is no certainty that the child will have lifelong problems with hypercholesterolemia.

A client comes into the​ clinic, and your examination reveals a lateral curvature of the​ spine, that​ is, scoliosis. Which of the following are potential causes of this ​disorder? Select all that apply. Improper posture Congenital abnormality Unequal leg length Pregnancy Rickets

Improper posture Congenital abnormality Unequal leg length Scoliosis is often a congenital abnormality or the result of a disease or​ injury, unequal leg​ length, improper​ posture, or a weakening of musculature. Pregnancy can cause an exaggeration of the normal lumbar curve called lordosis. Rickets and other disease processes like tuberculosis and syphilis can cause​ kyphosis, that​ is, an exaggeration of the normal convex curve of the thoracic spine.

What abnormal findings pertain to cranial nerve VI​ (abducens)? Select all that apply. Inability to shrug shoulders Inability to hear Inability to follow an object with the eyes Involuntary movements of the eyes Double vision

Inability to follow an object with the eyes Involuntary movements of the eyes Double vision Abnormal findings of cranial nerve VI​ (abducens) include the inability to follow an object with the​ eyes, involuntary movements of the​ eyes, and double vision. The inability to hear is an abnormal finding of cranial nerve VII​ (vestibulocochlear). The inability to shrug the shoulders is an abnormal finding of cranial nerve XI​ (accessory).

Identify the preferred term for abbreviations that are incorrectly used. ​Instructions: Use the dropdown menus in the left column to select the preferred term for each incorrectly used or mistaken abbreviation in the right column. Preferred Term Potential Problem Subcutaneously Morphine sulfate Discharged Three times weekly Incorrect​ abbreviation: T.I.W. Incorrect​ abbreviation: S.C. or S.Q. Incorrect​ abbreviation: MSO4 Mistaken for​ "discontinued" Mistaken for​ "sublingual (SL)" Mistaken for​ "three times a​ day" Mistaken for​ "magnesium sulfate" Incorrect​ abbreviation: D/C

Incorrect​ abbreviation: T.I.W.- Three times weekly Incorrect​ abbreviation: S.C. or S.Q.- Subcutaneously Incorrect​ abbreviation: MSO4- Morphine sulfate Mistaken for​ "discontinued"- Discharged Mistaken for​ "sublingual (SL)"- Subcutaneously Mistaken for​ "three times a​ day"- Three times weekly Mistaken for​ "magnesium sulfate"-Morphine sulfate Incorrect​ abbreviation: D/C- Discharged

The nurse is assessing a client who is experiencing acute somatic pain from a fractured tibia. What assessment finding would the nurse anticipate for this​ client? Increased blood pressure Burning in the leg Neuropathic pain Referred pain

Increased blood pressure In acute pain there is a sympathetic nervous system​ response: increased blood​ pressure, respiratory​ rate, and pulse​ rate; diaphoresis; and dilated pupils.Burning is typically seen in clients experiencing​ neuropathic, not​ somatic, pain. Bone pain from a break is not usually referred to other parts of the body. The client exhibits signs of somatic​ pain, which is not normally accompanied by neuropathic pain.

Identify the diseases of the myocardium and the pumping capacity of the heart based on the description. ​Instructions: Use the dropdown menus in the left column to select the type of cardiac disease for each description in the right column. Disease Description Myocardial Infarction Myocardial Ischemia Heart Failure Increased cardiac oxygen demands lead to increased cardiac​ workload, resulting in unmet oxygen needs of myocardium Interruption or cessation of delivery of oxygen and nutrients that can lead to death of myocardial tissues Inadequate pumping of myocardium that results in unmet body demands

Increased cardiac oxygen demands lead to increased cardiac​ workload, resulting in unmet oxygen needs of myocardium- myocardial ischemia Interruption or cessation of delivery of oxygen and nutrients that can lead to death of myocardial tissues- myocardial infarction Inadequate pumping of myocardium that results in unmet body demands- heart failure

What signs or symptoms would the nurse assess in an infant with ​hydrocephalus? Select all that apply. Increased intracranial pressure Head appears larger than normal Swelling of the extremities Expansion of the skull Swelling of the neck

Increased intracranial pressure Head appears larger than normal Expansion of the skull In hydrocephalus there is an inadequate drainage of cerebrospinal fluid causing excessive accumulation within the​ ventricles, resulting in the enlargement of the infant​'s head. The excessive accumulation of cerebrospinal fluid also increases intracranial pressure. The skull of the infant enlarges to accommodate the increase in cerebrospinal fluid if not treated. The neck and extremities do not enlarge with​ hydrocephalus, as the cerebrospinal fluid is contained in the brain and spinal column so the head is the area that enlarges to accommodate the excessive fluid buildup.

What are common methods of ​percussion? Select all that apply. Zone percussion Indirect percussion Tapping percussion Blunt percussion Direct percussion

Indirect percussion Blunt percussion Direct percussion Percussion methods include​ direct, blunt, and indirect. Tapping and zone percussion are not methods of percussion.

The nurse is assessing the client by placing only the hyperextended middle finger of the nondominant hand over the area being examined and uses only wrist action of the dominant hand to deliver two sharp blows using only the middle finger of the dominant hand. What is the best description of the assessment technique the nurse is​ using? Direct percussion Auscultation Light palpation Indirect percussion

Indirect percussion Indirect percussion is performed by placing the hyperextended middle finger of the nondominant hand over the area being examined and using the wrist action of the dominant hand to deliver two sharp blows using only the middle finger of the dominant hand. Direct percussion involves the technique of tapping the body with the fingertips of the dominant hand. Auscultation requires use of the nurse​'s sense of hearing as well as special​ instruments, such as a stethoscope or Doppler ultrasonic stethoscope. Palpation requires that the nurse move the examining hand slowly and intentionally across the skin to determine specific characteristics of the body.

What conditions can contribute to renal​ problems? Select all that apply. Influenza Diabetes mellitus Chlamydia Spinal cord injury High blood pressure

Influenza Diabetes mellitus Spinal cord injury High blood pressure Renal problems can accompany high blood pressure and diabetes​ mellitus, and the client may not be aware of them. Spinal cord injury can cause renal problems due to paralysis. Influenza and other infections can contribute to the risk of developing a renal abnormality. Chlamydia is a sexually transmitted infection that does not contribute to renal problems.

The nurse is assessing an adult client with jaw pain. What techniques would the nurse use when assessing the jaw of an adult ​client? Select all that apply. Palpate the tonsils and the back of the throat. Inspect the face for symmetry. Use a sterile applicator to touch the salivary glands. Inspect the teeth. Palpate under the jaw line and inside the mouth.

Inspect the face for symmetry. Use a sterile applicator to touch the salivary glands. Inspect the teeth. Palpate under the jaw line and inside the mouth. Inspecting the face for symmetry would reveal if there is swelling at the jaw line on either side of the face. Inspecting the teeth for condition or debris would assist in finding any signs of​ infection, broken​ teeth, abscess, or swelling that may contribute to jaw pain. Using a gloved hand to palpate the jaw line and the inside of the cheeks would help detect signs of lumps or nodules on all surfaces. With gloves​ on, touching the area of the salivary glands to assess their size using a sterile applicator ensures that they are visible and intact. It is not appropriate to palpate the tonsils and the back of the throat due to the gag​ reflex, but it is appropriate to inspect the tonsils and the back of the throat using a tongue blade to ensure that there is no obstruction. Tonsils may be absent in an​ adult, but the throat should be​ pink, moist, and slightly vascular.

Arrange the steps of an abdominal assessment in the correct sequence. ​Instructions: Click on the dropdown arrow for each step of the procedure in the right column and select the correct choice from the list. Order Procedure Palpation Auscultation Inspection Percussion

Inspection Auscultation Percussion Palpation

The nurse is preparing to performing a physical assessment on the​ client's abdomen. In which order would the nurse performing the​ assessment? ​Instructions: Click on the dropdown arrow for each step of the procedure in the right column and select the correct choice from the list. Palpation Auscultation Percussion Inspection

Inspection Auscultation Percussion Palpation

What techniques would be used to assess the head and neck of an adult​ client? Palpation and auscultation Inspection and palpation Inspection and auscultation Percussion and palpation

Inspection and palpation The head and neck are inspected and palpated during a focused assessment. Auscultation is typically not used in the assessment of the​ head, although it may be used to assess the vascular system in the neck. Percussion is typically not used when assessing the head of a client.

The charge nurse is reviewing medical records on a surgical unit. What documentation finding indicates the need for further staff​ education? 0.1 mL of digoxin​ (Lanoxin) Insulin 10 U given at 0800 ECG LMP

Insulin 10 U given at 0800 According to the​ "Do Not Use​ List," U should not be used because it can be mistaken for​ zero, four, or cc. Instead the nurse should use Units. The charge nurse should recognize this as a need for further teaching. The abbreviation​ "LMP" is appropriate for use. The abbreviation mL is the correct term to use instead of cc. The abbreviation ECG is appropriate for use.

The nurse is assessing an adult male client who complains of low back pain and spasm. What occupation would not be related to the development of low back pain and​ strain? Firefighter Insurance salesperson Professional athlete Construction worker

Insurance salesperson When considering occupations that are associated with heavy lifting and repetitive movements that may lead to back pain and​ strain, the nurse would associate construction​ work, firefighting, or being an athlete to musculoskeletal low back pain. The occupation of salesperson would not generally be associated with heavy lifting related to musculoskeletal injury.

A client with tiny growths of epithelial cells that project into the lumen of the lactiferous ducts has what​ condition? Intraductal papilloma Gynecomastia Carcinoma of the breast Mammary duct ectasia

Intraductal papilloma Intraductal papilloma is characterized by tiny growths of epithelial cells that project into the lumen of the lactiferous ducts. Mammary duct ectasia is characterized by inflammation of the lactiferous ducts behind the nipple. Carcinoma of the breast is characterized by dimpling skin over the​ tumor, alignment deviation of the breast or​ nipple, the irregular shape of one​ breast, edema, or nipple discharge. Gynecomastia is enlargement of the male breast tissue.

Mr. Jones is a​ 64-year-old man who presents to the emergency department with complaints of shortness of breath. What is the first thing you want to​ ascertain? Does the client have a history of stomach​ problems? Is the client currently taking​ medication? Is the condition constant or associated with​ activity? When was the​ client's last​ meal?

Is the condition constant or associated with​ activity? Determining if the condition is constant or associated with a particular activity enables you to direct the questions to gather an accurate picture of the issue at hand. Whether a client is taking medication is relevant but not primary to the cause of the​ client's shortness of breath. A​ client's stomach issues and what the client last ate are not of primary concern for someone with shortness of breath.

Mr. George is at the clinic for an evaluation of his heart. His diagnostic testing includes an​ electrocardiogram, which provides an image of the electrical conduction of the heart. You are aware that the sinoatrial​ (SA) node is a critical element of the​ heart's conduction system. Which is the best way for you to describe the function of the SA node to Mr.​ George? It receives the current that has spread throughout the​ atria, and it can initiate the electrical impulse of the heart if there is a failure of the atrioventricular node. Extending from the right and left bundle​ branches, the SA node spreads electrical current from the myocardial tissues into the cardiac tissue. It initiates the electrical impulse of the​ heart, discharging​ 60-100 times per minute. It disseminates the atrial contraction current to the ventricles.

It initiates the electrical impulse of the​ heart, discharging​ 60-100 times per minute. The SA node discharges​ 60-100 times per minute and is referred to as the pacemaker of the heart. The AV node receives the current that has already spread throughout the atria and may initiate contraction of the heart if the SA node fails. Purkinje fibers penetrate into the myocardial tissue to spread the current into the cardiac tissue. The bundle of His channels the atrial​ contraction's rhythm to the ventricles.

What color variation is due to increased bilirubin in the​ blood? Uremia Cyanosis Pallor Jaundice

Jaundice Jaundice, or skin with a yellow​ undertone, is due to increased bilirubin in the blood. Pallor is a loss of color in skin that is due to the absence of oxygenated hemoglobin.​ Cyanosis, a mottled blue color in skin and its​ appendages, is due to inadequate tissue perfusion with oxygenated blood.​ Uremia, a shade of pale​ yellow, is due to retention of urinary chromogens in the blood.

The small intestine is the primary digestive and absorption organ. Which of the following are parts of the small​ intestine? Select all that apply. Cecum Jejunum Sigmoid colon Duodenum Ileum

Jejunum Duodenum Ileum The​ duodenum, jejunum, and the ileum are the three segments of the small intestines that extend from the stomach to the ileocecal valve. The cecum and the sigmoid colon are parts of the large intestines.

People of what cultures are more likely to be lactose ​intolerant? Select all that apply. Asian American Jewish American Mexican American African American Caucasian

Jewish American Caucasian Caucasians and Jewish Americans are more likely to be lactose intolerant.

The nurse is assessing a child who is complaining of pain in the​ ankles, knees, and wrists. On​ inspection, the child​'s joints are​ swollen, and they are tender on palpation. What childhood inflammatory condition would the nurse​ suspect? Juvenile rheumatoid arthritis Fibromyalgia Psoriatic arthritis Ankylosing spondylitis

Juvenile rheumatoid arthritis A child who presents with​ pain, swelling, and immobility in more than one joint would be evaluated for juvenile rheumatoid arthritis. Fibromyalgia is associated with unexplained muscle pain in adults. Ankylosing spondylitis is an inflammatory condition of the spine and vertebrae. Psoriatic arthritis is inflammation of the joints in clients who also have severe psoriasis.

The nurse is preparing to enter the client​'s health assessment information into the computerized medical record. Which actions by the nurse maintain the client​'s ​confidentiality? Select all that apply. Keeping the assigned password a secret Documenting only at the end of the shift so that computer login occurs once per shift Preventing unlicensed assistive personnel access to private records Logging off the computer before walking away Keeping computer screens turned away from public areas

Keeping the assigned password a secret Logging off the computer before walking away Keeping computer screens turned away from public areas A personal password should not be shared with​ anyone, including other health care team members. Logging off the computer before walking away will help prevent others from accessing files under the nurse​'s name. Computer screens positioned away from public areas will help keep others from reading the screens. Documenting should be done as care is provided. Keep documentation accurate and​ timely, and available for other appropriate personnel as this prevents forgetting details. Unlicensed assistive personnel may access private records as​ needed, and they should have their own usernames and passwords.

The nurse is assessing an adult client who complains of difficulty urinating. What organs related to the urinary system would be included in the abdominal​ assessment? Spleen Liver Intestines Kidneys and bladder

Kidneys and bladder The kidneys are the body​'s filtration system and produce the urine. The bladder stores urine until it is released. All parts of this urinary system are within the abdominal cavity and are included in the abdominal assessment. The intestines deal with digestion and absorption as well as elimination of feces. The spleen filters blood and destroys old red blood cells. The liver aids in digestion and produces substances for blood coagulation. It also helps to detoxify the body. It is not part of the urinary system.

Which​ statement(s) are true about factors that affect core body​ temperature? Select all that apply. Lack of subcutaneous fat can increase sensitivity to temperature changes in the environment. Strenuous exercise can increase core temperature by 2°-5°F. A decrease in epinephrine and norepinephrine during times of stress increases metabolic activity and temperature. Core body temperature does not generally vary throughout the day. Illness that impairs the function of the hypothalamus can cause hyperthermia.

Lack of subcutaneous fat can increase sensitivity to temperature changes in the environment. Strenuous exercise can increase core temperature by 2°-5°F. Illness that impairs the function of the hypothalamus can cause hyperthermia. Lack of subcutaneous fat can increase sensitivity to temperature changes in the environment. Strenuous exercise can increase core temperature by 2​°dash-5​°F. Illness that impairs the function of the hypothalamus can cause​ hyperthermia, and at times hypothermia in the elder adult. Core body temperature can vary throughout the day as much as 1.8​°F. An increase in epinephrine and norepinephrine during times of stress increases metabolic activity and temperature.

What condition found in newborns is described as​ fine, downy​ hair? Lanugo Mongolian spots Vernix caseosa Milia

Lanugo Lanugo is​ fine, downy hair found on newborns and is replaced with vellus hair within a few months. Milia are tiny white facial papules due to sebum that collects in the openings of the hair​ follicles; they disappear a few weeks after birth. Mongolian spots are​ gray, blue, or purple spots in the sacral and buttocks area of newborns of​ African, Asian, and Native American descent that fade by the age of 3. Vernix caseosa is a whitish​ cheese-like mixture of sebum and epidermal cells found on the skin at birth.

Identify the sources of pain based on their origin. ​Instructions: Use the dropdown menus in the left column to select the source of pain for each point of origin in the right column. Pain Source Origin Visceral pain Neuropathic pain Referred pain Deep somatic pain Large internal organs​ (e.g., gallbladder,​ kidneys) Another location due to innervation of the same spinal nerve Peripheral nerves due to abnormal processing of pain message Ischemia or trauma to​ joints, muscle, or bone

Large internal organs​ (e.g., gallbladder,​ kidneys)- visceral pain Another location due to innervation of the same spinal nerve- referred pain Peripheral nerves due to abnormal processing of pain message- neuropathic pain Ischemia or trauma to​ joints, muscle, or bone- deep somatic pain

Which of the following organs is not considered an accessory digestive​ organ? Gallbladder Pancreas Large intestine Liver

Large intestine The large intestine is an essential organ in the digestive process. The main function of the large intestine is to absorb water and to eliminate waste. The liver is responsible for the production and secretion of bile for fat emulsification and plays a major role in the metabolism of​ proteins, fats, and carbohydrates. The gallbladder is the storage organ for bile and assists in the digestion of fats. The pancreas secretes insulin for carbohydrate metabolism. The​ liver, gallbladder, and pancreas are all considered accessory digestive organs.

The nurse is reviewing the anatomy and physiology of the alimentary canal and recognizes that what organ absorbs water from indigestible food ​residue? Select all that apply. Large intestine Stomach Esophagus Small intestine

Large intestine The nurse would recognize that the large intestine absorbs water from indigestible food residue and eliminates the residue as feces. The esophagus propels food and fluid from the mouth to the stomach. The stomach mixes food with digestive juices in the​ stomach, transforming it to the chyme that enters the small intestine. The small intestine continues the breakdown of food and absorbs nutrients from digested foods.

The nurse is preparing the discharge summary for a client who is going home after surgery. What information documented by the nurse indicates the need for further education about proper​ documentation? Last three sets of vital signs ​Follow-up appointment with the health care provider Lifting restrictions ​Diet, activity, and exercise needs

Last three sets of vital signs A description of the client​'s ​physical, mental, and emotional status at discharge or transfer should be included in the discharge​ summary; however, it would not be necessary to include multiple sets of vital signs. Including this information in the summary indicates a need for further education regarding proper documentation. Restricted activities such as​ lifting, stair​ climbing, walking,​ driving, work, and​ diet, as well as bathing restrictions such as sponge​ bath, tub, or shower should be included in the discharge summary. Client education provided in relation to disease​ process, activities and​ exercise, special​ diet, medications, specialized care or​ treatments, and​ follow-up appointments are also included in the summary.

The nurse is preparing to assess the skin of a newborn infant. Which assessment finding is not consistent with this age​ group? Milia Lanugo Stork bite Linea nigra

Linea nigra Linea nigra is a dark line extending from the umbilicus to the pubic area in the pregnant client. It may disappear after delivery. Lanugo is​ fine, downy hair on the newborn. It is replaced within a few months by vellus hair. Stork bites are irregular red and pink patches found on the back of the neck in newborns. They usually disappear within the first year of life. Milia are tiny white facial papules caused by sebum that collects in the openings of hair follicles and disappear a few weeks after birth.

Which of the following findings of the​ ear, nose,​ mouth, and throat are indicative of a client under​ stress? Select all that apply. Feelings of sadness Eating disorder Lip biting Mouth ulcers Tics

Lip biting Mouth ulcers Tics Psychosocial considerations such as stress may be the result of assessment findings such as lip​ biting, tics, and mouth ulcers. Feelings of sadness are indicative of problems with interpersonal relationships such as the loss of a spouse. A client may experience gastrointestinal problems when under​ stress, but would not normally develop an eating disorder.

How would the nurse use auscultation in an assessment of the head and neck of an adult ​client? Select all that apply. Auscultate heart sounds Listening over a tumor Listening over the thyroid gland Auscultation of the temporal arteries Auscultation of the carotids

Listening over the thyroid gland Auscultation of the temporal arteries Auscultation of the carotids Auscultation of the temporal arteries can determine if there is​ stenosis, which is abnormal. Listening over the thyroid gland can identify a​ bruit, which is an increase in blood flow to the thyroid and is abnormal. Listening to the carotid arteries on either side of the thyroid for blood flow and a bruit is helpful when assessing the neck and vascular system of an adult client. Auscultation is helpful in assessing the head and neck of both children and adults. Auscultation is not used to listen over a​ tumor, but a bruit identified while listening over the thyroid gland is an indication of increased blood flow to the thyroid gland and may indicate a tumor. Auscultate heart sounds is incorrect as the heart sounds are not found in the head and neck.

The nurse is preparing to educate an adult client on healthy living and safety. What factors in everyday life would put the client at risk for injury and would be part of the client ​education? Select all that apply. Living in a​ high-crime neighborhood Access to illicit drugs ​Risk-taking behaviors by the client Sufficient income to purchase safety equipment Living in an unsafe environment

Living in a​ high-crime neighborhood Access to illicit drugs ​Risk-taking behaviors by the client Living in an unsafe environment Living in an unsafe environment would put the client at risk for​ injury, as would living in a​ high-crime neighborhood due to the exposure to violence. Having access to illicit drugs can put the client at risk for​ injury, as can any​ risk-taking behaviors by the client including sexual​ behavior, drinking,​ smoking, and drug use. Insufficient income to buy safety equipment would put the client at risk for​ injury; those who can afford to purchase safety equipment lower their risk for injury.

The nurse is preparing to assess the client​'s oxygen saturation. What finding should the nurse anticipate would impact the measurement of the client​'s oxygen​ saturation? Weight Increasing age Male gender Low hemoglobin

Low hemoglobin Factors that can affect the oxygen saturation level are low​ hemoglobin, circulation, activity​ level, and carbon monoxide.​ Weight, age, and gender will not impact oxygen saturation readings.

Mr. Huff is at the clinic for a routine visit. You plan to perform a thorough cardiac assessment. In what position should Mr. Huff be placed for you to best inspect the internal and external jugular​ vessels? High Fowler position with extra lighting Lying down at a 45degrees angle with extra lighting Sitting up in a darkened room while you use a flashlight Lying flat with general lighting

Lying down at a 45degrees angle with extra lighting Lying down at a​ 45° angle using extra lighting is best to visualize the jugular vessels. These vessels are not visible when the client is sitting up. Extra lighting is needed to properly assess the vessels.

Ms. Kemper is a​ 45-year-old woman who has come to the clinic for a breast examination. In what position should she be when palpating her​ breasts? Leaning forward from the waist Sitting upright Standing with hands pressed against waist Lying flat

Lying flat In order to properly palpate the​ breasts, the client would lie flat. The client would be sitting upright when palpating the axillae. When observing the breasts for​ shape, surface​ characteristics, and bilateral pull of sensory muscles the client would be standing with hands pressed against the waist and leaning forward from the waist.

The nurse is caring for a client who presents to the emergency department with​ Bell's ​ palsy, blurred​ vision, and headaches. What disease would the nurse​ suspect? Brain abscess Myelitis Lyme disease Meningitis

Lyme disease Lyme disease is an infection that can cause​ Bell's palsy, visual​ disturbances, headaches, and damage to nerves in the extremities. A brain abscess is an infection of the brain​ cells, usually from a systemic infection. Meningitis is an infection of the brain and spinal cord. Myelitis is an inflammation of the spinal cord that usually develops after an infection such as measles or​ gonorrhea, or after vaccination for rabies.

When inspecting and palpating a client​'s lymph​ nodes, which of the following is not a normal​ finding? Lymph node is discrete. Lymph node is tender upon palpation. Lymph node is movable. Lymph node is soft.

Lymph node is tender upon palpation. An abnormal finding for palpable lymph nodes would be tenderness upon palpation. Normal findings are that the lymph nodes are​ discrete, soft, and movable.

The nurse is using inspection and palpation to assess the lymphatic system of an adult client. What would be the normal findings when palpating lymph ​nodes? Select all that apply. Lymph nodes are movable. Lymph nodes are discrete and may not be obvious. Lymph nodes may not be palpable. Lymph nodes are solid and nonmovable. Lymph nodes are soft and nontender.

Lymph nodes are movable. Lymph nodes are discrete and may not be obvious. Lymph nodes may not be palpable. Lymph nodes are soft and nontender. Some lymph nodes may not be palpable. Movable lymph nodes are a normal finding. It is normal for lymph nodes to be discrete and difficult to palpate. Nodes would be nontender and​ soft; painful and hard nodes are an abnormal finding. Lymph nodes that are solid and do not move may indicate a mass or tumor.

The nurse is preparing to teach a group of adult clients about the risks for overnutrition. What information would the nurse include in the ​teaching? Select all that apply. Reducing dairy and meat intake Maintaining an appropriate body weight Ways to manage fat intake How to reduce calories to less than 1000 per day Increasing the use of supplements

Maintaining an appropriate body weight Ways to manage fat intake Information about appropriate body weight is useful information. Education regarding fat intake is appropriate to include. Use of supplements can lead to overnutrition and increasing the intake should not be encouraged. Education about dairy and meat as potentially high in saturated fat is​ important, but reducing intake is not necessarily required for all clients. Severe calorie reductions are not an appropriate intervention.

The nurse assesses a​ thick, sticky discharge from the nipples of a nonlactating female client. In what breast condition would this symptom​ occur? Menopause Intraductal papilloma Mammary duct ectasia Carcinoma

Mammary duct ectasia Mammary duct ectasia is inflammation of the ducts behind the nipple. A​ thick, sticky discharge is​ common, and the condition is benign and usually​ self-resolving. A clear or bloody discharge is usually seen in​ cancer, or​ carcinoma, of the breast. Intraductal papilloma is tiny growths of epithelial cells in the​ ducts; there can be bloody or clear drainage from the nipples if trauma occurs. A​ thick, sticky discharge from the nipples is not a normal symptom of menopause.

A nurse is reviewing the structures of the breast and axillae before educating a client. What structure extends from each axilla to the groin and is termed the milk​ line? Areola Mammary ridge Acini cells Axillary tail

Mammary ridge The mammary ridge extends from each axilla to the groin and is also called the milk line. The acini cells are contained within the glandular tissue and produce milk. The areola is where the nipples lie and is the pigmented field of wrinkled skin. The axillary tail is the breast tissue that extends into the axilla and is also called the tail of Spence.

What breast and axillae structure extends from each axilla to the​ groin? Suspensory ligament Axillary tail Mammary ridge Acini cell

Mammary ridge The mammary​ ridge, also known as the milk​ line, extends from each axilla to the groin. The axillary​ tail, also known as the tail of​ Spence, is breast tissue that extends superiolaterally to the axilla. Acini cells are the cells that produce milk and are contained in the lobules of the breast. Suspensory​ ligaments, or Cooper​ ligaments, extend from the connective tissue layer through the breast and attach to the fascia underlying the breast.

The nurse is assessing an adult client with bluish sclera. What cardiovascular abnormality would the nurse​ suspect? Marfan syndrome Fluid retention Cushing syndrome Yellow fat pads around the eyes

Marfan syndrome Blue sclera is a symptom directly related to Marfan​ syndrome, a degenerative disease of the connective tissue that can cause the ascending aorta to either dilate or dissect and can result in death. Yellowish fat pads may indicate early plaque in the arteries of the heart. Periorbital edema may indicate fluid​ retention, and a ruddy or flushed face may indicate Cushing syndrome related to cardiovascular and thyroid disease.

The nurse is completing the health history of a client. Which information would the nurse obtain regarding biographical ​data? Select all that apply. Family history. Marital status. Name. Insurance. Allergies.

Marital status. Name. Insurance. The biographical data provides a data set from which the nurse can begin to make judgments that are used to relate and compare individual characteristics to established expectations and norms for physical and emotional health. Marital​ status, name, and insurance information are all part of the biographical data. Family history is part of the family history piece of the health assessment.​ Allergies are part of the past medical history piece of the health history interview.

Identify the terms that describe the effectiveness of the cardiac cycle based on the description. ​Instructions: Use the dropdown menus in the left column to select the correct term used for measuring cardiac cycle for each description in the right column. Term Description Cardiac index Cardiac output Stroke volume Measurement of effectiveness of pumping action of heart Amount of blood ejected with each heartbeat Amount of blood ejected from left ventricle over 1 minute Cardiac output​ / heart rate for 1 full minute Cardiac output​ / body surface area Stroke volume​ / heart rate for 1 full minute

Measurement of effectiveness of pumping action of heart- cardiac index Amount of blood ejected with each heartbeat- stroke volume Amount of blood ejected from left ventricle over 1 minute- cardiac output Cardiac output​ / heart rate for 1 full minute- stroke volume Cardiac output​ / body surface area- cardiac index Stroke volume​ / heart rate for 1 full minute- cardiac output

The nurse is assessing the musculoskeletal system of a newborn. What abnormal conditions related to this system may be found in the newborn during the ​assessment? Select all that apply. Genu valgum Metatarsal adductus Tibial torsion Flat feet Congenital hip dislocation

Metatarsal adductus Tibial torsion Congenital hip dislocation The newborn assessment might find abnormal conditions that have occurred during birth or as a result of how the infant was positioned in utero. Tibial torsion is the inward twisting of the tibia and improves as the infant grows. Metatarsal adductus is when the bones in the front portion of the foot turn in toward the​ body, also a result of positioning in the uterus. Newborns can also have congenital dislocations of the hips during delivery or afterward. Newborns are normally born with flat feet. Genu valgum is positioning of the knees so that they knock together. This condition may occur around the age of 4 years and would not be assessed in a newborn.

Identify the thoracic planes based on the imaginary lines that run vertically from the apex of the lungs to the diaphragm. ​Instructions: Use the dropdown menus in the left column to select the type of thoracic plane for each imaginary line in the right column. Thoracic Plane Imaginary Line Posterior Thorax Lateral Thorax Anterior Thorax ​Mid-sternal line Right and left​ mid-axillary line Vertebral spine line Right and left​ mid-clavicular line Right and left scapular line

Mid-sternal line- Anterior Thorax Right and left​ mid-axillary line- Lateral Thorax Vertebral spine line- Posterior Thorax Right and left​ mid-clavicular line-Anterior Thorax Right and left scapular line- Posterior Thorax

Which should be included in your palpation​ technique? Select all that apply. Move quickly but tentatively to palpate. Minimize pain to the client. Perform deep palpation if rigidity is suspected in underlying organs. Use the dorsal surface of the hand to assess temperature. Palpation may be completed with or without gloves.

Minimize pain to the client. Use the dorsal surface of the hand to assess temperature. Palpation may be completed with or without gloves. Palpation may be completed with or without​ gloves, but standard precautions should be used when needed. Pain to the client should be minimized. Using the dorsal surface of the hand is best to assess temperature. The nurse should move slowly but purposefully during palpation. Deep palpation is contraindicated if rigidity or inflammation is suspected in the underlying tissues.

Which of the following is an abnormal finding when inspecting a​ client's breast? Nipples are same color as areolae. Skin color is consistent with rest of body. Moles are tender and have changed. Venous patterns are the same bilaterally.

Moles are tender and have changed. Moles and markings that have changed and are tender are abnormal findings and would be further investigated. Skin color that is consistent with the rest of the​ body, venous patterns that are the same on both sides​ (but may be more prominent in pregnant and obese​ women), and nipples that are the same color as the areolae are all normal findings.

The nurse is assessing the breasts of a young adult female client who expresses concerns about tiny bumps around the nipple area. What normal breast anatomy explains these​ bumps? Areola Mammary ridge Montgomery glands Acini cells

Montgomery glands Montgomery glands are tiny sebaceous glands that are speckled on the surface of the areola and are a normal part of the breast anatomy. The areola is the pigmented field of skin of the breast where the nipple lies. Acini cells produce milk in the breast. The mammary ridge extends from the axillae to the groin.

Which of the following abnormal findings can be associated with​ stress? Select all that apply. Mouth ulcers Lip biting Gastrointestinal difficulties A calm appearance without tics or tremors Tics

Mouth ulcers Lip biting Gastrointestinal difficulties Tics Mouth ulcers may indicate that the client is under stress. Adults may chew or bite the lips if under​ stress, and this may be obvious to others but not the client. Tics may be obvious to the observer and can be the result of stress. GI difficulties can occur with stress. Most frequently the nurse will find out about this by questioning the client about bowel habits or abdominal pain. A calm appearance without tics or tremors is normal and does not inform the observer about whether the client is under stress or not.​ However, it is wrong to assume that the client has no​ stress, as further questioning during the exam might reveal the opposite.

The nurse is caring for a​ 33-year-old client who is being admitted to the emergency department. The client reports​ tingling, weakness, and numbness on the left side of the body. What disease process would the nurse​ suspect? Myasthenia gravis Multiple sclerosis ​Alzheimer's disease ​Parkinson's disease

Multiple sclerosis Multiple sclerosis generally affects clients between 20 and 40 years of age. They present with​ tingling, weakness, and numbness affecting one limb or one side of the body. Myasthenia gravis presents with gradual increased muscle weakness.​ Parkinson's disease presents with​ "pill rolling,"​ tremors, head​ bobbing, and difficulty initiating movement. Clients with​ Alzheimer's disease report loss of​ short-term memory and appear disoriented and confused.

The nurse is completing a neurologic assessment on a​ 33-year-old client who is having muscle weakness. For what familial neurologic disease would the nurse assess the​ client? Multiple sclerosis Seizures Lyme disease Lead poisoning

Multiple sclerosis Multiple sclerosis is a hereditary neurologic disease that affects the muscles and would be assessed. Seizure disorder is not considered a hereditary disease. Lead poisoning is an environmental condition that may put the client at risk for developing neurologic problems. Lyme disease is a bacterial infection​ that, if left​ untreated, can cause neurologic problems.

Ms. Monda is a​ 25-year-old woman who has come to the clinic with​ tingling, weakness, and numbness in her right arm. What condition might these symptoms​ indicate? Myasthenia gravis Multiple sclerosis ​Parkinson's disease ​Huntington's disease

Multiple sclerosis Multiple sclerosis usually affects people​ (women more than​ men) between the ages of 20 and 40 and begins with symptoms of​ tingling, weakness, and numbness affecting one limb or one side of the body.​ Huntington's disease is marked by movement disorders such as jerky movements and​ slow, uncoordinated fine movements.​ Parkinson's disease is marked by​ tremors, head​ bobbing, and difficulty initiating movement. Myasthenia gravis is a neuromuscular disorder that presents with gradual increased weakness of the voluntary muscles.

What illnesses are classified as neurologic​ illnesses? Select all that apply. Diabetes Rheumatoid arthritis Multiple sclerosis Myasthenia gravis Stroke

Multiple sclerosis Myasthenia gravis Stroke Stroke, multiple​ sclerosis, and myasthenia gravis are all illnesses that affect the neurologic system. Diabetes is a metabolic disease in which a person has high blood sugar. Rheumatoid arthritis is an autoimmune disease that causes inflammation of the joints.

Mrs. Darling is a​ 46-year-old woman who is an avid runner. She came into the clinic with pain in her calf after a long run. What is Mrs. Darling most likely suffering​ from? Boutonniere deformities Muscle strain Hallux valgus Synovitis of the knee

Muscle strain Muscle strain is a partial muscle tear resulting from overstretching or overuse of the muscle. Synovitis of the knee is a distention of the suprapatellar area and lateral aspects of the knee. Boutonniere deformities occur when the proximal interphalangeal joint is flexed in conjunction with distal interphalangeal joint hyperextension. Hallux valgus is the abnormal adduction of the great toe at the metatarsophalangeal joint.

What would you be assessing by having the client try to turn his or her head to the right and left while placing your hand on the side of the client​'s face for​ resistance? Position of the trachea Range of motion Symmetry of midline and accessory muscles Muscle strength

Muscle strength You would be testing a client​'s muscle strength when you have the client try to turn his or her head to the right and left while placing your hand on the side of the client​'s face for resistance. When inspecting for range of​ motion, you would have the client touch chin to​ chest, turn the head to the right and​ left, and try to touch the head to the shoulders​ (without raising the​ shoulders). You would use inspection to assess position of the trachea and symmetry of the midline and accessory muscles.

dentify the parts of the heart based on the description. ​Instructions: Use the dropdown menus in the left column to select the correct part of the heart for each description in the right column. Anatomy Description Myocardium Thin sac surrounding heart Endocardium Muscle that provides pumping action of heart Tissue that lines inner surface of heart chambers and valves Pericardium

Muscle that provides pumping action of heart- myocardium Tissue that lines inner surface of heart chambers and valves- endocardium Pericardium- thin sac surrounding the heart

The nurse is caring for a client who is having neurologic symptoms. What disease would the nurse recognize as an autoimmune​ disease? Amyotrophic lateral sclerosis Myasthenia gravis ​Huntington's disease ​Parkinson's disease

Myasthenia gravis Myasthenia gravis is thought to be an autoimmune disorder.​ Huntington's disease has a genetic link and leads to death.​ Parkinson's disease is caused by the degeneration of the nuclei of the brain. Amyotrophic Lateral Sclerosis​ (ALS, also called Lou​ Gehrig's disease) is a degenerative disease of the neurons in the spinal​ cord, brainstem and cerebral cortex.

Which of the following thyroid conditions is characterized as a severe form of hypothyroidism that causes nonpitting edema throughout the body and thickening of the facial​ features? Hashimoto thyroiditis Graves disease Thyroid adenoma Myxedema

Myxedema Myxedema is a severe form of hypothyroidism that that causes nonpitting edema throughout the body and thickening of the facial features and affects major organ systems. Graves disease is the most common form of hyperthyroidism. Thyroid adenoma is benign thyroid nodules that frequently occur in older adults. Hashimoto thyroiditis is an autoimmune disease that results in primary hypothyroidism.

The nurse places the following documentation in the​ client's medical ​record: ​10/10 dash- 0830 Client is alert and oriented. Denies pain at this time.​ Resting in bed with spouse at bedside. Vital signs within normal range. Morning care provided. The nurse identifies this as which type of​ charting? Case management charting Focus charting ​Problem-oriented record​ (POR) charting Narrative charting

Narrative charting This is an example of narrative charting. Narrative charting consists of written notes that include routine​ care, normal​ findings, and client​ problems, which are frequently documented in chronological order.​ Problem-oriented record charting has four basic​ components: database, problem​ list, plan of​ care, and progress note. The progress note is usually in SOAP or​ SOAPIE(R); or​ AP, APIE, or APIER format. Focused charting usually uses three columns for​ recording: date and​ time, focus, and progress notes. The case management documentation model uses a multidisciplinary approach to planning and documenting​ care, using critical pathways.

Ms. Cho brought her​ 2-year-old daughter Katie into the clinic because she is having trouble breathing through her nose. Upon inspection you note that she has noisy breathing and nasal discharge. What could these symptoms​ indicate? Benign growth Chronic allergies Sinus inflammation Nasal obstruction

Nasal obstruction Based on the​ client's age, her noisy​ breathing, and the accompanying nasal​ discharge, she may have a nasal obstruction. Chronic allergies would be indicated by tenderness of the sinuses and a swollen nasal cavity. A benign growth would present as a​ smooth, pale lump in clients with allergies. Sinus inflammation would present with pain and tenderness of the sinuses.

Identify the nutritional deficiency disease based on the clinical manifestation. ​Instructions: Use the dropdown menus in the left column to select the correct nutritional deficiency disease for each clinical manifestation in the right column. Disorder Clinical Manifestation Rickets Pellagra Kwashiorkor Niacin deficiency Generalized edema and scaly areas of decreased skin pigmentation Scaly skin lesions on​ sun-exposed areas of the body Calcium and vitamin D deficiency Protein deficiency Osteomalacia in adults and enlargement of epiphyseal growth plates in children

Niacin deficiency- Pellagra Generalized edema and scaly areas of decreased skin pigmentation- Kwashiorkor Scaly skin lesions on​ sun-exposed areas of the body- Pellagra Calcium and vitamin D deficiency-Rickets Protein deficiency- Kwashiorkor Osteomalacia in adults and enlargement of epiphyseal growth plates in children- Rickets

What is an abnormal finding when palpating a​ client's breasts and​ axillae? No palpable lumps Skin with uninterrupted contour Areolae that are free of masses Nipple discharge

Nipple discharge Nipple discharge may indicate benign breast​ disease, an intraductal​ carcinoma, or cancer and warrants further evaluation. Normal findings when palpating a​ client's breasts and axillae are skin that has a smooth texture and uninterrupted​ contour, no lumps or masses being​ detected, and the areolae being free of masses.

What statements are true about assessing the lymph system of the breast and axillae ​area? Select all that apply. Nodes are found around the clavicles. The lymphatic system drains lymph from the breasts. Nodes are found inside the breast. There are no nodes in the axillary area. The lymph nodes normally are not palpable.

Nodes are found around the clavicles. The lymphatic system drains lymph from the breasts. Nodes are found inside the breast. The lymph nodes normally are not palpable. The lymphatic system drains lymph and returns it to the bloodstream. The lymph nodes are not usually palpable on​ exam, so it is important to palpate all the areas where nodes are present to assess the health of the lymph system. Nodes inside the breast are called internal mammary nodes. The supraclavicular and subclavicular lymph nodes are found above and below the clavicles. The nodes in the axillary area are called​ central, brachial, or​ lateral; subscapular posterior​ axillary; and pectoral or anterior axillary nodes and are palpated during a physical exam.

The nurse is preparing to assess the client​'s pain. Which factors could affect the client​'s pain ​levels? Select all that apply. Noisy environment Previous experience with pain Meaning of pain Age of 35 years Ethnicity

Noisy environment Previous experience with pain Meaning of pain Ethnicity Behavior related to pain is a socialized behavior. Cultural background affects the amount of pain an individual is willing to tolerate. Members of some ethnic groups are more stoic and less expressive when experiencing pain. The client​'s ethnicity may affect the client​'s perceptions and reporting of pain. Being in a strange environment such as a hospital with the​ noises, lights, and activity can impact clients​' perception of their pain. Without a support​ network, the pain may even feel more severe to the client. Clients in certain family​ roles, especially the traditional​ caregiver, may feel uncomfortable verbalizing the severity of their pain. Previous pain experiences alter the perception of pain due to the anticipation of what is to come. Success or lack of success with pain relief measures influences the pain experience. The client​'s developmental stage and age would influence the client​'s reaction to and expression of​ pain; however, being 35 years old should not affect the client​'s pain. Nursing interventions would vary depending on the client​'s age. Children may be less able to articulate their needs related to pain.

Which statements are true regarding the evaluation and management of​ pain? Select all that apply. The nurse should be able to reason with a toddler who is in pain. The nurse is the best judge of a​ client's pain. Nonpharmacological therapies such as massage and stress management strategies may be effective in decreasing pain. It should be expected that the older adult client will always have some pain. Fatigue and unplanned weight loss may be signs of pain in the older adult.

Nonpharmacological therapies such as massage and stress management strategies may be effective in decreasing pain. Fatigue and unplanned weight loss may be signs of pain in the older adult.

Mr. Jordan is at the clinic for a routine cardiac examination. Which assessment finding would you suspect is an indication of an abdominal​ aneurysm? Dull percussion to the lower cardiac border Respiratory distress Notable pulsations to the abdominal wall ​Dilated, distended veins in the abdomen

Notable pulsations to the abdominal wall A noted pulsation of the abdominal wall in a client who is not extremely thin is an indicator of an abdominal aneurysm and should be evaluated further by the health care provider. Dull percussion to the lower cardiac border can indicate a dissecting ascending aortic aneurysm.​ Dilated, distended veins in the abdomen would most likely indicate an obstruction of the superior vena​ cava, not aneurysm. Respiratory distress can cause a cardiac​ event, but is not an indicator of an aneurysm.

Identify whether the element affecting the communication process is an external or internal factor. ​Instructions: Use the dropdown menus in the left column to select the factor that affects the communication process for each element in the right column. Factor Element Internal Factors External Factors Note taking Empathy Physical environment Optimistic view of others Uninterrupted interview Ability to listen Client​ dress/nurse dress Ensuring privacy

Note taking- external Empathy- internal Physical environment- external Optimistic view of others- interrnal Uninterrupted interview- external Ability to listen- internal Client​ dress/nurse dress- external Ensuring privacy- external

When palpating the chest of a client with shortness of​ breath, the nurse feels crunching under the skin of the chest wall in the​ mid-clavicular area. What action indicates that the nurse understands the significance of this​ finding? Tells the client that obesity can present as crepitus Explains that this is a normal finding Notifies the health care provider and prepares for a chest​ x-ray Explains that crepitus always ends in respiratory arrest

Notifies the health care provider and prepares for a chest​ x-ray Crepitus indicates air leaking into subcutaneous tissue. It is not a normal finding and should be reported to the health care provider. A chest​ x-ray may be indicated to see how the lungs are​ filling, and further testing may need to be done. Obesity does not cause air to leak into the​ tissue, but obesity may make it difficult to feel the crepitus if an air leak is present. Respiratory arrest would be​ considered, but crepitus does not always lead to that emergency. Assessment of other respiratory symptoms should be evaluated such as respiratory​ rate, skin​ color, and level of distress the client is experiencing.

Mr.​ Harris, a​ middle-aged adult client assigned to your​ care, received an oral pain medication one hour ago for treatment of acute​ surgical-site pain. Which is the most appropriate pain assessment tool for you to use in assessing medication effectiveness and his current pain​ status? ​"Faces" pain scale Brief Pain Inventory Initial baseline pain assessment Numeric pain rating scale

Numeric pain rating scale Brief numeric pain rating scales are assessment tools that provide the​ client's subjective reports about current pain status. The​ "faces" pain scale is most appropriate for use with children or if there is a communication barrier between the nurse and client. The Brief Pain Inventory is a rating tool that assesses pain in the previous 24 hours. An initial pain assessment is a detailed evaluation conducted at the time of​ admission; it documents past and baseline history​ (to the point in time when the client accessed the present health care​ services).

The nurse is preparing to assess the client​'s health history. What would the nurse assess to determine the client​'s nutritional health ​status? Select all that apply. Muscle mass Nutrient requirements Blood lipids Nutrient intake Body mass index​ (BMI)

Nutrient requirements Nutrient intake Nutrient intake and nutrient requirements are part of the balance of nutritional health. Body mass index​ (BMI) is an assessment​ measure, not part of nutritional health. Blood lipids is a blood test that indicates cardiac risk. It is not part of the assessment of nutritional health. Muscle mass is an assessment measure but is affected by many factors other than nutrition.

The nurse is assessing the chest of an adult and observes dilated and distended veins. What abnormal cardiovascular condition is associated with superficial dilated and distended veins of the​ chest?" An increase in respiration rate Engorged nipples A cerebral aneurysm Obstruction of the superior vena cava

Obstruction of the superior vena cava Distended and dilated veins in the chest would indicate a possible obstruction of the superior vena cava due to a decrease in venous return. Engorged nipples are not related to cardiac disease. An aneurysm might be noted as a bulge in the chest between the​ ribs, but a cerebral aneurysm located in the brain would not be visible. An increased rate of respiration would not indicate disease. If the rise and fall is rapid with retractions or if the client appears to be having difficulty breathing then that would be abnormal and could be related to the heart.

The nurse is completing a health history on a client who is being seen in the clinic for the first time. Which information would the nurse obtain during the psychosocial ​assessment? Select all that apply. Occupation Medication history Social supports Past hospitalizations Educational level

Occupation Social supports Educational level The psychosocial history includes information about the​ client's occupational​ history, educational​ level, financial​ background, roles and​ relationships, ethnicity and​ culture, family​ spirituality, and​ self-concept. The health history includes the reason for seeking care as well as identification of health care beliefs and​ practices, health​ patterns, health​ goals, and information about medications and therapies.

The nurse is preparing to assess the​ client's temperature. Which assessment finding would the nurse recognize as an elevated​ temperature? Tympanic 36.5°C ​(97.7°​F) Temporal 37.5°C ​(99.5°​F) Oral 37.7°C ​(99.9°​F) Axillary 36.3°C ​(97.3°​F)

Oral 37.7°C ​(99.9°​F) The normal range for an oral temperature is 36.5degrees°C to 37.5degrees°C ​(97.7degrees°F to 99.5degrees°​F). An oral temperature of 37.7degrees°C ​(99.9degrees°​F) would be considered elevated. The normal range for an axillary temperature is 35.9degrees°C to 36.9degrees°C ​(96.7degrees°F to 98.5degrees°​F). A tympanic temperature reading of 36.5degrees°C is considered low. The normal range for tympanic temperatures is 36.8degrees°C to 37.8degrees°C ​(98.2degrees°F to 100degrees°​F). A temporal temperature reading of 37.5degrees°C ​(99.5degrees°​F) is considered within normal limits.

Upon inspection of a client​'s mouth and throat you find lesions on the buccal mucosa and tongue. The client reports the lesions being there for several months. What could this​ indicate? Herpes simplex virus Gingivitis Dysfunction of the hypoglossal nerve Oral cancer

Oral cancer Persistent lesions of the buccal mucosa and tongue may be an indication of oral cancer. Lesions or blisters on the lips are an indication of herpes simplex virus. Gingivitis may be indicated if a client has bleeding gums. Dysfunction of the hypoglossal nerve is indicated by tremors of the tongue.

The nurse is assessing a​ 65-year-old client who complains of being short of breath when lying flat. The nurse places the client in high Fowler position and the client​'s breathing improves. Which unexpected finding is the client​ experiencing? Orthopnea Kussmaul breathing Hyperventilation Apnea

Orthopnea Orthopnea is difficulty breathing when lying flat. This client​'s breathing improved when sitting up so this client is experiencing orthopnea. Kussmaul breathing consists of deep and long respirations. Apnea is cessation of breathing. Hyperventilation is​ deep, rapid breathing usually from​ hypoxia, anxiety, or exercise.

The nurse is assessing a child who complains of ear pain. Which selection does not represent the anatomy or function of the inner​ ear? Cochlea Ossicles Semicircular canals Receptors for hearing

Ossicles Ossicles are the three bones in the middle​ ear: the​ malleus, incus, and stapes. These bones vibrate to sound waves. The cochlea is found in the inner​ ear, as are the receptors for hearing. The semicircular canals are in the inner ear and are the​ body's balance organs. The inner ear is one of the most important working parts of the ear for hearing and balance.

The strength and mineral density of bones differ based on people​'s ethnic and racial backgrounds. Which of the following statements are ​accurate? Select all that apply. Osteoarthritis occurs twice as frequently in African American women as in Caucasian women. Asians have higher bone density compared to those of European descent. Hispanics from Spain are shorter and heavier than Hispanics from Mexico. Bone density is higher in those of African descent than in those of European descent. Osteoarthritis occurs more frequently in Caucasian men than in African American men

Osteoarthritis occurs twice as frequently in African American women as in Caucasian women. Bone density is higher in those of African descent than in those of European descent. Osteoarthritis occurs twice as frequently in African American women as in Caucasian women and is more frequent in African American men as well. Bone density is higher in those of African descent than those of European descent. Asians have a lower bone density compared to those of European descent. Hispanics from Mexico are shorter and heavier than Hispanics from Spain. Osteoarthritis occurs less frequently in Caucasian men than in African American men.

The nurse assesses a hyperresonance sound when percussing the left side of a client​'s chest. What abnormal conditions are associated with this type of sound during ​percussion? Select all that apply. Pleural effusion Overinflation of the lungs Pneumothorax Emphysema A surgically removed lung

Overinflation of the lungs Pneumothorax Emphysema Overinflation of the lungs would cause a hyperresonance type of sound during percussion of the chest wall. Emphysema is a condition in which the lungs are in a state of​ overinflation, which would cause a hyperresonance sound during percussion. A pneumothorax would cause a hyperresonance sound during percussion due to the overinflation of the area around the pneumothorax. A pleural effusion is filled with fluid and would give off a dull sound during percussion. The empty space in the pleural cavity where a lung has been surgically removed would give off a dull sound during percussion.

The nurse is caring for a male client who supplements meals with several protein bars and drinks throughout the day. For what nutritional deficiency would the nurse assess in this​ client? Malnutrition Undernutrition Protein calorie malnutrition Overnutrition

Overnutrition Supplementing meals with protein bars and drinks places the client at risk for overnutrition due to their caloric counts.

Which​ statement(s) describe the process of​ respiration? Select all that apply. Oxygen diffuses into a capillary Exchange of oxygen and carbon dioxide Oxygen is obtained from the air and transported to alveoli Chest wall rises and falls Movement of lungs

Oxygen diffuses into a capillary Exchange of oxygen and carbon dioxide Oxygen is obtained from the air and transported to alveoli Inspiratory and expiratory movement of the lungs is called​ ventilation; it is required for​ respiration, but movement of lungs is not termed respiration. All of the other descriptions are included in the process of respiration.

The​ nurse, while documenting in the client​'s medical​ record, correctly uses which​ abbreviation? PRN Q.D. U MSO4

PRN PRN is an approved abbreviation. It stands for​ "as needed." The abbreviation Q.D. is on the Joint Commission​ "Do Not​ Use" list and should be written out as​ "daily" or​ "every ​day."​ MSO4 can be incorrectly interpreted and lead to a medication​ error, so it is better to write this out as​ "morphine sulfate." U can be interpreted incorrectly and potentially lead to medication​ errors, so it is best to write out the term​ "unit."

Ms. Powers is a​ 34-year-old woman who came into the clinic because of persistent headaches. Which of the following symptoms indicates that she suffers from cluster​ headaches? Sinus inflammation Onset of headache is gradual. Pain begins on one side and spreads upward behind the eye. Pain is pulsating and localized in the back of the head.

Pain begins on one side and spreads upward behind the eye. A headache that begins on one side and spreads upward behind the eye is one of several symptoms that are specific to cluster headaches. A migraine is indicated by pain that is pulsating and localized to​ front, side, or back of the head. Gradual onset of headache is indicative of a tension headache. Onset in a cluster headache is sudden and often occurs at night. A tension headache is often associated with sinus inflammation.

Identify the type of headache based on the symptoms. ​Instructions: Use the dropdown menus in the left column to select the type of headache for each symptom in the right column. Type of Headache Symptom Tension Headache Cluster Headache Classic Migraine Pain is pulsating and localized to​ side, front, or back of head. Associated with alcohol consumption Pain may be mild or debilitating. Numerous episodes over days or months Seeing spots or flashes Sudden onset usually at night Acute phase Begins on one side of face and spreads upward behind eye Usually preceded by aura Associated with premenstrual​ syndrome, position, sinus inflammation Numbness or tingling in face or extremities Pain is unilateral or bilateral and ranges from cervical region to top of head. ​Depression, restlessness, irritability Due to sustained contractions of muscles in​ head, neck, and upper back Nausea Onset is gradual. No aura occurs. Pain is steady.

Pain is pulsating and localized to​ side, front, or back of head.- migrane Associated with alcohol consumption- cluster Pain may be mild or debilitating.- migrane Numerous episodes over days or months- cluster Seeing spots or flashes- migrane Sudden onset usually at night- cluster Acute phase- migrane Begins on one side of face and spreads upward behind eye- cluster Usually preceded by aura- migrane Associated with premenstrual​ syndrome, position, sinus inflammation-tension Numbness or tingling in face or extremities- migrane Pain is unilateral or bilateral and ranges from cervical region to top of head.- tension ​Depression, restlessness, irritability- migrane Due to sustained contractions of muscles in​ head, neck, and upper back- tension Nausea- migrane Onset is gradual.- tension No aura occurs.- cluster Pain is steady.- tension

The nurse is assessing a client​'s skin. What findings support a decrease in oxygenation ​levels? Select all that apply. Pallor Erythema Uremia Cyanosis Jaundice

Pallor Cyanosis Pallor can be caused by the absence of oxygenated hemoglobin. Cyanosis is the result of inadequate tissue perfusion with oxygenated blood. Jaundice is a yellow undertone of the skin caused by increased bilirubin in the blood. Uremia is characterized by pale yellow skin and is due to the retention of urinary chromogens in the blood. Erythema is a redness of the skin due to the increased visibility of normal oxyhemoglobin.

The nurse is performing a newborn assessment of the musculoskeletal system. Which assessment technique is best used to assess for vibration and​ masses? Inspection Auscultation Palpation Percussion

Palpation A mass​ and/or vibration is an abnormal finding and would be assessed using palpation

The nurse is assessing a client with a cardiovascular abnormality. The client reports feeling very stressed. Which interventions can the nurse recommend to help the client reduce ​stress? Select all that apply. Participate in a hobby Yoga and meditation Drink alcoholic beverages every evening Psychological counseling Exercise

Participate in a hobby Yoga and meditation Psychological counseling Exercise Understanding the causes of stress can assist the nurse in suggesting ways for clients to deal with stress to improve cardiovascular health.​ Counseling, meditation, finding a​ hobby, and exercise are all good ways to alleviate stress. Alcohol can increase cardiovascular disease and decrease circulation and is not a healthy way to alleviate stress.

A new patient is visiting the clinic for the first time. The nurse obtains information from the patient regarding​ allergies, hospitalizations, and surgeries. Which type of information is the nurse​ gathering? Past health history. Psychosocial history. Present health or illness. Family history.

Past health history. The past history includes information about childhood​ diseases; immunizations;​ allergies; blood​ transfusions; major​ illnesses; injuries;​ hospitalizations; labors and​ deliveries; surgical​ procedures; mental,​ emotional, and psychiatric health​ problems; and the use of​ alcohol, tobacco, and other substances. A psychosocial history includes data about​ occupation, education, financial​ background, roles, and relationships of the patient. A family history includes information about the​ patient's parents',​ siblings', and​ children's health status. Present health or illness includes data about the​ patient's current health concern.

The client notifies the nurse that pain is beginning to become uncomfortable. Which phase of nociceptive pain is the client​ experiencing? Transmission Modulation Perception Transduction

Perception Perception is when the client becomes conscious of the pain. This is what this client is experiencing. Transduction occurs when neurotransmitters are released and send an action potential to the spinal cord. Transmission occurs when pain impulses move from the spinal cord to the brain. Modulation is the body​'s ​built-in system that slows down the processing of pain.

The nurse is preparing to assess the client​'s liver size. Which examination technique is the most appropriate for the nurse to​ use? Auscultation Palpation Inspection Percussion

Percussion Percuss the abdomen to determine the upper and lower borders of the liver. Once the borders are​ determined, the nurse should measure the distance between both​ points, which is referred to as the liver span. Inspection involves looking at the client or body​ part, and the size of the liver cannot be determined using inspection. Auscultation of the liver would provide no data about liver size or location.​ Normally, the liver is​ nonpalpable, except in thin​ client's. If the nurse is able to palpate the​ liver, the lower border is all that is​ palpable, which does not allow determination of liver size.

The nurse is assessing the left upper quadrant​ (LUQ) of a client. What assessment technique is used to determine the size of the​ spleen? Deep palpation Inspection Auscultation Percussion

Percussion Percussion is the standard way to determine the size of the spleen when doing a focused abdominal exam. Auscultation is not helpful because there are no sounds from the spleen. Inspection would not provide an accurate idea of the size of the spleen. The spleen is normally not palpable. When assessing the​ spleen, the nurse must be careful because the spleen is very fragile.​ Therefore, deep palpation is not the best assessment method to determine the size of the spleen.

The nurse is assessing an adult client who is complaining of chest congestion and cough. What​ skill(s) would the nurse expect to use to specifically assess for chest​ congestion? Palpation and percussion Percussion and auscultation Inspection Auscultation

Percussion and auscultation An adult complaining of chest congestion must be thoroughly assessed to determine if the congestion is upper respiratory or cardiac in origin. Percussion is helpful in assessing density or​ fluid-filled spaces, and auscultation assesses the air exchanged during a respiratory cycle. The combination of percussion and auscultation would make for a complete assessment of the lungs when a client complains of chest congestion.

The nurse is working in the pediatric clinic performing assessments of infants and children. What is true about a normal assessment on an infant or ​child? Select all that apply Percussion elicits a greater tympany sound in an infant or child. Abdominal breathing ceases after the age of 3. An infant​'s abdomen is round. The abdominal muscle wall is thinner in infants and children. Infants are abdominal breathers.

Percussion elicits a greater tympany sound in an infant or child. An infant​'s abdomen is round. The abdominal muscle wall is thinner in infants and children. Infants are abdominal breathers. The abdomen of an infant is​ round, and toddlers​' abdomens have a potbelly shape. Infants use abdominal muscles to assist with​ breathing, so the belly looks like it is moving most of the time. The abdominal muscle wall is thinner in an infant or​ child, making it easier to visualize peristalsis. Infants and children swallow more air than​ adults, giving the abdomen a greater tympany sound when doing percussion. Abdominal breathing ceases after the age of 6.

The nurse is preparing to auscultate the client​'s lungs. Which technique used by the nurse indicates a need for the nurse​'s further teaching regarding​ auscultation? Covers the client with a towel or sheet during auscultation. Keeps the client warm. Disinfects stethoscope after each use. Places the stethoscope on the client​'s gown when listening to heart and lung sounds.

Places the stethoscope on the client​'s gown when listening to heart and lung sounds. During auscultation the nurse should listen to the body sounds without the obstruction of clothing by placing the stethoscope directly on the client​'s skin. The nurse should cover the client with a​ towel, sheet, or blanket to preserve the client​'s privacy. The nurse should provide for the client​'s comfort and keep the client warm during auscultation to prevent​ shivering, which can obscure sounds. The nurse should disinfect the stethoscope between​ client's to avoid spreading pathogens.

Identify the method of percussion based on the percussion technique. ​Instructions: Use the dropdown menus in the left column to select the method of percussion for each technique in the right column. Method of Percussion Technique Blunt Percussion Indirect Percussion Direct Percussion Performed by placing the palm of the hand against the body​ surface, then firmly striking the flat hand with a closed fist Administer sharp blows with the middle finger of one hand to the distant phalanx of the​ other, which is placed directly on the body surface Performed by tapping the body with fingertips Used for assessment of kidney pain Used for assessment of organ borders Used in examination of the sinuses of an adult

Performed by placing the palm of the hand against the body​ surface, then firmly striking the flat hand with a closed fist- blunt percussion Administer sharp blows with the middle finger of one hand to the distant phalanx of the​ other, which is placed directly on the body surface- indirect percussion Performed by tapping the body with fingertips- direct percussion Used for assessment of kidney pain- blunt percussion Used for assessment of organ borders- indirect percussion Used in examination of the sinuses of an adult- direct percussion

The nurse auscultates the lungs of the​ client, who is suspected of having​ pneumonia, and hears rhonchi. Which assessment technique is best to use to determine if there is dullness in the​ lungs? Inquire about pain with inspiration. Inspect the quality of breathing. Performing direct percussion. Palpate the lungs for a mass.

Performing direct percussion. Dullness may indicate that air in the lungs has been replaced by either fluid or a mass. It is determined by performing direct percussion on the lungs. Inspecting the quality of breathing and asking about pain will not help the nurse determine if there is dullness in the lungs. The lungs are surrounded by the rib​ cage, which makes it impossible to palpate them.

The charge nurse is reviewing the sequence of anatomy from the outside to the inside of the heart with a nursing student. In what order would the charge nurse place the anatomy for a healthy adult presenting with complaints of​ heartburn? ​Instructions: Click on the down arrow for each response in the right column and select the correct choice from the list. Order responses Myocardium Endocardium Chambers Pericardium Heart valves

Pericardium Myocardium Endocardium Chambers Heart valves

An episiotomy scar that is free of lesions may be found when inspecting what​ structure? Vaginal opening Perineum Anal area Labia minora

Perineum Upon inspection of the​ perineum, an episiotomy scar that is free of lesions may be​ found, which is normal. The vaginal opening is normally pink in color. The labia minora normally appear​ smooth, pink, and moist and without any redness or swelling. The anal area normally is dark in​ color, moist and​ intact, and free of lesions.

The nurse is caring for a hypertensive client in the health clinic. Which circulatory factor could contribute to the client​'s elevated blood​ pressure? Respiratory rate Ethnicity Age Peripheral vascular resistance

Peripheral vascular resistance Peripheral vascular resistance is the resistance the blood encounters as it flows through the vessels and is a circulatory influence on blood pressure. Age is not a circulatory factor affecting blood pressure. Ethnicity is a genetic factor affecting blood​ pressure, not a circulatory factor. The respiratory rate does not affect the client​'s blood pressure.

What structure related to the accessory digestive organs secretes a serous fluid that lubricates the surface of​ organs? Aorta Peritoneum Abdominal wall muscles Kidneys

Peritoneum The peritoneum is a serous membrane in the abdominal cavity that lubricates the surface of​ organs, allowing motion of structures without friction. The muscles of the abdominal wall support and protect the abdominal viscera. The aorta supplies blood to all the abdominal parietal and visceral structures. The kidneys filter nitrogenous wastes that result in the production of urine.

The nurse is assessing a client​'s neurologic health history. What environmental exposure would affect a​ client's neurologic​ health? (select all that apply) Perfume Pesticides Lead Organic solvents

Pesticides Lead Organic solvents Exposure to​ pesticides, organic​ solvents, lead, toxic​ waste, and other pollutants can cause neurologic conditions and the nurse would assess the client for these environmental exposures. Perfume does not contribute to neurologic illness and disease.

A client presents to the clinic for a​ follow-up appointment. During which phase of the health history would the nurse review the​ client's lab reports and medical​ records? Phase I of the health assessment interview. After the interview is completed. Phase III of the health assessment interview. Phase II of the health assessment interview.

Phase I of the health assessment interview. The nurse would review the​ client's previous information during Phase I of the health​ assessment, which occurs before the initial meeting with the client. Phase II is the initial meeting with the​ client, and old data and reports would be reviewed before this phase. Phase III is the focused interview. Review of old data would be used to guide the nurse during the interview and is reviewed prior to the start of the assessment.

Frequent fractures and sprains may indicate that a client is dealing with which of the following psychosocial​ issues? Depression Social isolation Lifestyle changes Physical abuse

Physical abuse Physical abuse should be considered when assessing a client with a history of frequent​ fractures, sprains, or strains. In such a​ case, state guidelines need to be followed as you refer a client to social or protective services. Impaired mobility due to health issues can lead to psychosocial issues including social isolation and depression. Lifestyle changes affect the musculoskeletal​ system, for​ example, a sedentary lifestyle can hasten musculoskeletal changes just as an active lifestyle can promote muscle mass and help prevent or delay osteoporosis.

The nurse has a role in providing safety to clients when giving care. For what nursing activities would the nurse check the health care provider​'s ​order, identify the​ client, and practice hand ​hygiene? Select all that apply. Placing an IV in the arm of an adult Answering the unit telephone Administering oral medications Having a conversation with an​ LPN/LVN Inserting a catheter

Placing an IV in the arm of an adult Administering oral medications Inserting a catheter When placing an​ IV, inserting a​ catheter, or administering oral medications to a​ client, the nurse would check the health care provider​'s ​order, identify the​ client, and practice hand hygiene. The nurse would use telephone etiquette when answering the phone but would not check a health care provider​'s order. Having a conversation with an​ LPN/LVN would not require checking a health care provider​'s ​order, identifying a​ client, or hand hygiene.

What barriers could prevent the nurse from completing a comprehensive health assessment of the breasts on an older adult ​client?​ Select all that apply. Pendulous breast tissue Poor vision Limited range of motion Replacement of fatty tissue with glandular tissue after menopause Client privacy issues

Poor vision Limited range of motion Client privacy issues Older clients may be uncomfortable or​ self-conscious talking about private or sexual issues like​ breasts, making the assessment more difficult. Older adults with poor vision may not be able to inspect their own skin or read printed information about breast​ health, requiring​ large-print materials, large​ mirrors, bright​ lights, and a magnifying glass to inspect the skin. Limited range of motion may make a comprehensive physical breast exam more difficult to perform because of the inability to raise the arms. Breast tissue may be more​ pendulous, but this would not make examining the breasts more difficult for the nurse. Fatty tissue replaces glandular tissue after​ menopause, but this would not make the exam more difficult.

Which of the following is relevant content for the summary at the end of the​ interview? Select all that apply. Marriage status Positive health aspects Health problems identified Schools attended Action plans

Positive health aspects Health problems identified Action plans The summary is a final statement of what was discussed about a client​'s current health issues. Positive health​ aspects, identified health​ problems, and any action plans to be taken are included in the summary. A client​'s marital status or where the client attended school is not necessary to summarize the client​'s current health issues.

The nurse is palpating the abdomen of an adult and finds an area that is​ tender, hard, and painful. What abnormal circumstance do these findings​ indicate? Gastroenteritis Possible​ tumor, mass, or obstruction Peristalsis Diarrhea

Possible​ tumor, mass, or obstruction An area that is​ tender, hard, and painful on palpation is not normal and may indicate a​ mass, tumor, or obstruction. Gastroenteritis may present as pain or tenderness but is not associated with an area of hardness on palpation. Diarrhea does not present during palpation but may be suspected with auscultation of hyperactive bowel sounds. Peristalsis would not present as a​ hard, tender, or painful area with palpation.

The nurse is assessing the abdomen of an adult and upon inspection finds the umbilicus is displaced. What abnormal abdominal conditions might a displaced umbilicus ​indicate? Select all that apply. Pregnancy An abdominal mass Abdominal pain An umbilical hernia Distended bladder

Pregnancy An abdominal mass An umbilical hernia Distended bladder A displaced umbilicus can be seen when the urinary bladder is overdistended. The umbilicus may also look displaced in a female client who is pregnant or if there is a mass in the abdominal cavity. Seen more in​ children, an umbilical hernia may make the umbilicus appear displaced. A displaced umbilicus does not indicate that pain is present.

dentify the type of palpation based on the purpose. ​Instructions: Use the dropdown menus in the left column to select the type of palpation for each purpose in the right column. Type of Palpation Purpose Light Palpation Moderate Palpation Deep Palpation Press downward​ 1-2 cm Assess organs deep in the body cavity Assess​ pulse, skin​ texture, or tenderness Utilized for most bodily structures Safest and least uncomfortable Contraindicated with symptoms of dissecting aneurysm or peritonitis

Press downward​ 1-2 cm- moderate palpation Assess organs deep in the body cavity- deep palpation Assess​ pulse, skin​ texture, or tenderness- light palpation Utilized for most bodily structures- moderate palpation Safest and least uncomfortable- light palpation Contraindicated with symptoms of dissecting aneurysm or peritonitis- deep palpation

The nurse is caring for a female client who is 7 months pregnant. What neurologic changes would the nurse expect in this​ client? Select all that apply. Prominent lumbar curvature Pressure on nerves in the pelvis Decrease in deep tendon reflexes Shift in center of gravity Slowed coordination

Prominent lumbar curvature Pressure on nerves in the pelvis Shift in center of gravity Pressure on the nerves in the pelvis is caused by the growing fetus and is an expected finding in pregnancy. The lumbar curvature of the spine becomes prominent during pregnancy. The center of gravity shifts in pregnant clients due to the weight of the fetus. A decrease in deep tendon reflexes and slowed coordination happen when clients​ age, not during pregnancy.

Ms. Haywood comes to the clinic complaining of generalized​ edema, a change in hair​ color, and scaly skin. You would expect further evaluation for a deficiency of which​ nutrient? Niacin Vitamin C Calcium Protein

Protein The client is describing symptoms of​ Kwashiorkor, a protein deficiency usually caused by a high​ carbohydrate, little or no protein diet. A lack of vitamin C would be indicated by swollen or ulcerated gums. Decreased niacin intake would result in scaly skin lesions on​ sun-exposed areas of the body. Low calcium levels could cause osteomalacia.

The nurse is using​ inspection, palpation,​ percussion, and auscultation to assess the respiratory status of an older female client. What other​ skill(s) would be used during the assessment to ensure client ​satisfaction? Select all that apply. Avoiding percussion because it can be painful to female clients Providing for privacy during the exam Offering a drape to help the female client feel less exposed during the exam Avoiding a physical exam on this older female and opting for a chest​ x-ray Providing safety during the exam

Providing for privacy during the exam Offering a drape to help the female client feel less exposed during the exam Providing safety during the exam An older female client would be more satisfied if a complete exam were done with as much privacy as possible. Offering a sheet or drape for coverage would not only add to the client​'s feeling of privacy and modesty but would add warmth for an older​ female, who might feel cold. These small steps go far in helping to promote client satisfaction when doing a complete exam. Assessing a client​'s chest is a noninvasive and safe​ procedure, so safety is inherently provided. Avoiding percussion is unnecessary because when it is done​ properly, it is painless. Before opting for a chest​ x-ray, the cost and inconvenience involved must be​ considered, and it does not take the place of an exam.

During an​ interview, a client states that she has recently noticed stronger body odor. Which of the following is not a likely cause of​ this? Systemic disorders Diet Developmental changes Pruritus

Pruritus Pruritus or itching would not cause strong body odor. Body odor becomes stronger during heavy activity because of increased excretion of uric acid. Body odor may also be related to a​ client's diet, a systemic​ disorder, or developmental changes.

Identify the eating disorder based on the clinical manifestation. ​Instructions: Use the dropdown menus in the left column to select the type of eating disorder for each clinical manifestation in the right column. Disorder Manifestation Anorexia nervosa Bulimia Psychological disorder typically beginning in adolescence Characterized by bingeing and vomiting or excessive exercise Cycle of decreased food​ intake, increased​ exercise, and overuse of diet pills or laxatives Person may appear normal in weight or even overweight

Psychological disorder typically beginning in adolescence- Anorexia nervosa Characterized by bingeing and vomiting or excessive exercise- Bulimia Cycle of decreased food​ intake, increased​ exercise, and overuse of diet pills or laxatives- Anorexia nervosa Person may appear normal in weight or even overweight- Bulimia

The nurse is palpating the neck of an adult client and notes that the trachea is shifted and the neck feels large. What conditions would these abnormal findings indicate that would require further diagnostic ​work-up? Select all that apply. Pulmonary edema Pulmonary fibrosis Pneumothorax Enlarged thyroid Mass in the neck

Pulmonary fibrosis Pneumothorax Mass in the neck The trachea can be displaced from a mass in the​ neck, from a pneumothorax​ (shifting to the right or the left depending on which lung has the​ pneumothorax), or due to compensation from pulmonary fibrosis. The thyroid is below the​ trachea, and its enlargement would not usually displace the trachea. Pulmonary edema displays as bilateral engorged neck veins and would not displace the trachea.

Identify the ventricle associated with each ventricular hypertrophy. ​Instructions: Use the dropdown menus in the left column to select the correct ventricle that causes the disorder mentioned in the right column. Ventricle Disorder Right ventricle Left ventricle Pulmonary stenosis Systemic hypertension Pulmonary hypertension Aortic stenosis Lung disease Congenital heart disease

Pulmonary stenosis- right Systemic hypertension- left Pulmonary hypertension- right Aortic stenosis- left Lung disease- right Congenital heart disease- left

Mrs. Jones is recovering from a serious viral pneumonia that resulted in​ sepsis/shock and required endotracheal intubation with mechanical ventilatory support. Mrs. Jones is now greatly improved. All of the invasive arterial lines have been​ discontinued, and she has been transferred to your care on the medical respiratory​ step-down unit. Which is the best method for routinely assessing Mrs.​ Jones' oxygen saturation at this point in​ time? Temperature Arterial Blood Gas​ (ABG) sample Respiratory Rate Blood Pressure​ (BP) Pulse Oximeter

Pulse Oximeter Pulse oximetry measures blood oxygen saturation​ non-invasively, without pain and is extremely low risk for the patient. A digital readout allows the nurse to evaluate the findings against previous saturation levels and changes in oxygen therapy. If you do not remove the​ sensor, you should rotate the site of the​ clip-on probe every 4 hr. Replace disposable probes every 24 hr. Clean the probe again and perform hand hygiene. Document the findings on the appropriate hospital record. Arterial blood gas​ (ABG?s) procured from an arterial stick are appropriate in the critical care setting but are​ painful, invasive, and put the patient at risk for complications. Arterial punctures cannot be performed without special training and typically require a specific​ Physician's order or protocol. Blood pressure​ & temperature provide little information about oxygenation. Respiratory Rate is significant but is not a reliable indicator of blood oxygen saturation.

Mrs. Jones is recovering from a serious viral pneumonia that resulted in​ sepsis/shock and required endotracheal intubation with mechanical ventilatory support. Mrs. Jones is now greatly improved. All of the invasive arterial lines have been​ discontinued, and she has been transferred to your care on the medical respiratory​ step-down unit. Which is the best method for routinely assessing Mrs.​ Jones' oxygen saturation at this point in​ time? Assess for circumoral cyanosis Pulse oximeter device Arterial blood gas Count number of respirations for one full minute

Pulse oximeter device A pulse oximeter device safely and​ non-invasively measures blood oxygen saturation usually without discomfort. Although arterial blood gas assessments are the most​ accurate, they are very​ invasive, pose significant​ risks, and are generally not used for routine assessment outside of an ICU setting. Although counting respiratory rate and assessing for circumoral cyanosis are useful assessment​ tools, these provide no measure of oxygen saturation. A client presenting with circumoral cyanosis has already advanced to perilous​ late-stage oxygen deprivation thus it is not an early indicator.

The nurse is working in a primary care clinic. Which client pulse rate should the nurse recognize as an unexpected ​finding? Select all that apply. Pulse rate of 110 in an​ 88-year-old client Pulse rate of 88 in an​ 8-year-old client Pulse rate of 55 in a​ 10-year-old client Pulse rate of 53 in a​ 45-year-old client Pulse rate of 72 in a​ 1-year-old client

Pulse rate of 110 in an​ 88-year-old client Pulse rate of 55 in a​ 10-year-old client Pulse rate of 53 in a​ 45-year-old client Pulse rate of 72 in a​ 1-year-old client The expected pulse rate for a​ 1-year-old is 80-130 beats per​ minute, for an​ 8-year-old and​ 10-year-old client is 70-110 beats per​ minute, for an adult is 60-100 beats per​ minute, and for an older adult is 60-100 beats per minute.

What are normal findings when assessing the carotid​ vessels? Select all that apply. Pulses are synchronous with the apical pulse Equal in strength and intensity Turbulent sounds on auscultation Present and equal bilaterally. Palpated at the fourth intercostal space

Pulses are synchronous with the apical pulse Equal in strength and intensity Present and equal bilaterally. Pulsations should be present and equal bilaterally. The carotid pulses should be synchronous with the apical pulse. Turbulent sounds should not be​ noted; these may be an indicator of partial occlusion. The carotid vessels are palpated in the groove between the trachea and sternocleidomastoid muscle beneath the jaw.

What is the purpose of asking a client about the amount of radiation exposure to the chest​ area? Radiation exposure increases the risk for lung cancer. Radiation exposure increases the risk for breast cancer. Asking about radiation to the chest is inappropriate. Radiation exposure decreases the risk for gene mutations from therapy.

Radiation exposure increases the risk for breast cancer. Radiation increases the risk for breast​ cancer, so this is an important question to ask a client. Radiation does not increase the risk for lung cancer. Radiation increases the risk for gene​ mutations, thus increasing the risk for recurrent cancers of other types.

The nurse is assessing an adult who is experiencing unexplained weight loss. What abdominal organ is responsible for primary​ digestion? Small intestine Stomach Kidneys Large intestine

Small intestine The small intestine is where primary digestion and absorption occurs. The three segments of the small intestine are the​ duodenum, the​ jejunum, and the ileum. The kidneys filter and remove waste products. The large intestine reabsorbs water and eliminates solid​ waste, and the stomach mixes food with digestive enzymes in preparation for the process that happens in the small intestine.

What is the proper position for the adult client during palpation of the​ axillae? Lying flat Lying on the side Lying flat with arms above the head Seated with the arm slightly raised

Seated with the arm slightly raised When palpating the​ axillae, the client is in a seated position as the examiner raises one arm at a time and supports the client​'s arm with the nondominant arm while using the opposite hand to palpate the axillae. To palpate the axillae it is best to have the client sitting upright. Palpating the axillae while the client is lying on the side is not​ recommended, as masses or lumps might be missed. Lying flat with arms above the head is not the position to best assess the axillae of an adult client.

Mrs.​ Gentile, an older woman with a history of heart​ problems, has come to the clinic today for a routine examination. You use the landmarks of the sternum and the angle of Louis to locate which important landmark for cardiac​ assessment? Suprasternal notch Second intercostal space Midclavicular line Thoracic angle

Second intercostal space The second intercostal space is located using the sternum and the angle of​ Louis; this is important to assess normal and abnormal cardiac processes. The suprasternal notch is found on the uppermost border of the manubrium. The midclavicular line is important in cardiac assessment but is found by using the clavicles as a landmark. The thoracic angle is not used for cardiac assessment.

What is the purpose of the bulbourethral​ glands? Produce sperm Secrete clear mucus fluid before ejaculation to lubricate the urethra Secrete testosterone Change the pH of the urine

Secrete clear mucus fluid before ejaculation to lubricate the urethra The bulbourethral glands are located behind the prostate gland and secrete a clear mucus fluid just before ejaculation to help lubricate the urethra and increase the alkaline environment for the sperm as they travel down the urethra. The testicles produce sperm and secrete testosterone. The bulbourethral glands do not change the pH of the urine.

Mrs. Boins has come to the clinic where she receives treatment for her medical problems. She is 79 years old and has concerns about her blood pressure medication dosage. She is not clear as to her medical history. What component of the health history might reveal previous medical​ problems? Family history Biographical data Review of body systems Psychosocial health

Review of body systems The review of body systems allows you to uncover information that may be previously unidentified but allows the client to recall any issues he or she may have forgotten or felt​ wasn't a concern in relation to the current issues. The family history focuses on the health of family members. Psychosocial health includes information about​ education, occupation,​ finances, and culture and relationships. Biographical data includes data such as​ name, age, and birthplace but does not include medical history.

When assessing a​ client, you determine that he has chronic inflammation of the metacarpophalangeal and intraphalangeal joints. What is this condition​ called? Olecranon bursitis Carpal tunnel syndrome Dupuytren contractures Rheumatoid arthritis

Rheumatoid arthritis Chronic inflammation of the metacarpophalangeal and intraphalangeal joints is caused by rheumatoid arthritis.. Olecranon bursitis is swelling of the olecranon bursa in the elbow. Carpal tunnel syndrome is a result of compression of the medial​ nerve, which lies inside the carpal tunnel. This disorder causes decreased motor function and leads to atrophy of the thenar eminence in the wrist. Dupuytren contractures are flexion contractures resulting from hyperplasia of the fascia of the palmar surface of the hand.

Martha Hickock is a​ 57-year-old woman who came into the clinic with complaints of joint pain in both hands. She stated that the pain abates as the day goes on. What is the possible cause of her joint​ pain? Rheumatoid arthritis Neurological problems Acute inflammation Fibromyalgia

Rheumatoid arthritis Rheumatoid arthritis affects symmetric​ joints, is worse in the​ morning, and improves as the day unfolds. It is not exacerbated by movement. Acute inflammation would be noted by​ swelling, heat, or redness of the joints. Fibromyalgia is noted by muscle pain or cramping. Neurologic problems in relation to the musculoskeletal system may be indicated by muscle​ weakness, numbness, or tingling.

Identify the electrical rhythm disturbance based on the clinical manifestation. ​Instructions: Use the dropdown menus in the left column to select the electrical rhythm for each clinical manifestation in the right column. Electrical Rhythm Manifestation Ventricular Tachycardia Atrial Flutter Heart Block Ventricular Fibrillation ​Rapid, regular heartbeat as high as 200 beats per minute ​Chaotic, irregular, and ineffective rhythm originating in ventricles Heart rate of​ 20-40 beats per minute resulting from conduction pathway interruption Rate as high as 200 beats per​ minute, exceeding ventricular response

Rapid, regular heartbeat as high as 200 beats per minute- ventricular tachycardia ​Chaotic, irregular, and ineffective rhythm originating in ventricles- ventricular fibrillation Heart rate of​ 20-40 beats per minute resulting from conduction pathway interruption- heart block Rate as high as 200 beats per​ minute, exceeding ventricular response- atrial flutter

A 25 year old woman reports to you that she is afraid no one will think she is attractive and that she will not be able to breastfeed a baby because her breasts are too small. Which of the following is an​ appropriate, therapeutic​ response? Refer her to a counselor Refer her to an endocrinologist Recommend she get breast implants Reassure her that breasts come in a variety of normal shapes and sizes.

Reassure her that breasts come in a variety of normal shapes and sizes. A client​'s ​self-esteem may often be related to her feelings about her breast size. It is appropriate to reassure her that normal breasts come in a variety of sizes and shapes. This does not impact her ability to breastfeed. Recommending breast implants is not appropriate. Unless there are other compounding​ factors, it is also not appropriate to refer this client to either an endocrinologist or counselor.

Mr. Harvey is at your clinic for a routine​ check-up. Which factors in his history place him at risk for poor​ nutrition? Select all that apply. History of alcohol use 5 years ago Routine dental care Current age of 53 Recent diagnosis of Type 2 diabetes mellitus Recent death of his spouse

Recent diagnosis of Type 2 diabetes mellitus Recent death of his spouse The recent death of Mr.​ Harvey's spouse places him at an increased risk for nutritional​ deficit, as bereavement and loneliness can contribute to poor nutrition. Chronic​ illness, such as Type 2​ diabetes, also places him at increased risk. Mr.​ Harvey's past alcohol use does not increase his risk for poor​ nutrition, but alcohol abuse could​ (although this is not part of Mr.​ Harvey's history). Routine dental care promotes nutrition. His age does not place him at increased risk for poor nutritional health.

Mrs. Rogers has come into the clinic because she has an excessively itchy scalp. Which of the following would be the likeliest​ cause? Changed body moisturizer Recently began swimming regularly Washes hair three times a week with same shampoo Allows hair to air dry

Recently began swimming regularly A recent change in exercise to swimming regularly is the likely cause of Mrs.​ Rogers' scalp itchiness. Swimming regularly in sea salt or chlorinated pools can dry the hair and​ scalp, and cause dandruff. A change in body moisturizer would affect Mrs.​ Rogers' skin all​ over, but not her scalp. Because Mrs. Rogers does not use hair styling implements like a hair dryer or curling​ iron, and she does not wash her hair excessively and has been using her usual​ shampoo, her condition is not likely to be caused by excessive dryness.

The nurse is palpating the breasts of an adult female client and notices skin thickening on the right breast. What action would not be appropriate after discovering this abnormal​ finding? Referring for a mammogram Inspecting the breasts for dimples or lumps Recommending a cortisone cream to be applied to the area Palpating the left breast for comparison

Recommending a cortisone cream to be applied to the area Thickening of the skin in one breast would be further evaluated for possible​ carcinoma, so recommending a cortisone cream is not appropriate. Referring the client for a mammogram is appropriate upon discovery of an abnormality in one breast. Comparing the right breast to the left breast gives the nurse a better understanding of what feels normal in the breasts of the client. Inspecting both breasts carefully for dimples or lumps after feeling skin thickening in one breast gives the nurse more information about the state of the breasts.

What abnormal findings on inspection of the breasts may indicate a malignancy and would need further ​investigation?​ Select all that apply. Redness in an area of the breast or nipple Dimples or puckering in the breast or nipple Enlarged areas of the breast Peau d​'orange Darkened nipples bilaterally

Redness in an area of the breast or nipple Dimples or puckering in the breast or nipple Enlarged areas of the breast Peau d​'orange Any enlarged area or mass that is seen in the breast on one side and not the other is worrisome for malignancy. Dimples or puckering can be signs of a tumor or mass. Redness on inspection or warmth on palpation would indicate inflammation and a possible malignancy. Peau d​'orange is the term for edema from blocked lymphatic​ drainage, which is seen in advanced cancer of the breast. Nipple color can be light or dark depending on the client​'s skin color and is not an abnormal condition if it is bilateral and consistent with the rest of the body.

Which of the following skills would enable effective communication with the client when obtaining a health​ history? Select all that apply. Examining Assessing Reflecting Leading Attending

Reflecting Leading Attending Attending to a client communicates that you are giving your undivided attention. Leading enables you to lead the conversation to reveal specific information. Reflecting ensures that you are empathetic to the client​'s situation. Examining and assessing are not skills that take place when obtaining a client​'s health history.

Which are interventions that should be included in the inspection portion of​ assessment? Select all that apply. Client permission should be obtained after assessment. Remove devices such as splints when possible. Use dim lighting for client comfort. Routinely inform client of your activities during the assessment. Expose body parts for assessment but maintain client privacy.

Remove devices such as splints when possible. Routinely inform client of your activities during the assessment. Expose body parts for assessment but maintain client privacy. Remove devices such as splints when possible in order to adequately assess the​ client's limbs. Maintenance of client privacy is​ priority, exposing body parts for assessment as needed. The client should be kept informed of all of your assessments. Lighting should be adequate for a complete​ assessment, and permission should be obtained before assessment is started. Next Question

What genitourinary system structures carry a large portion of the blood supply to the​ kidneys? Kidneys have their own blood supply. Renal arteries Renal veins Ureters

Renal arteries The renal arteries carry a large portion of the blood supply to the kidneys and the kidneys filter the blood. The ureters are muscular tubes that connect the kidneys to the urinary bladder and carry urine.

Which of the following abdominal characteristics are normal for an​ infant? Select all that apply. Greater sound of tympany with percussion Respiration is abdominal Delayed gastric emptying Round in appearance Peristaltic waves can be seen

Respiration is abdominal Round in appearance Peristaltic waves can be seen An infant​'s abdomen is round in appearance. Respiration is​ abdominal, so there is visible movement in the abdomen. Peristaltic waves can be​ seen, as the infant​'s muscle wall is thinner. A greater sound of tympany with percussion is consistent with​ children, not infants. Delayed gastric emptying is more common in older adults.

Brandon Lewis is an​ 8-year-old boy who was brought to the clinic by his parents because he does not feel well. You palpate his neck and note there is swelling of the lymph nodes located at the junction of the posterior and lateral walls of the pharynx at the angle of the jaw. What are these lymph nodes​ called? Postauricular Occipital Deep cervical Retropharyngeal

Retropharyngeal The retropharyngeal lymph nodes are located at the junction of the posterior and lateral walls of the pharynx at the angle of the jaw. The occipital lymph nodes are located at the base of the skull. The postauricular lymph nodes are located behind the ear. The deep cervical lymph nodes are located posterior to and under the sternocleidomastoid muscle.

Which of the following are developmental considerations of the​ ear, nose,​ mouth, and throat that pertain to a pregnant​ woman? Select all that apply. Auditory canal shorter than a child​'s Rhinitis and epistaxis Snoring caused by hyperemia of the throat Decrease in senses of taste and smell Feeling of fullness in the ear or an earache

Rhinitis and epistaxis Snoring caused by hyperemia of the throat Feeling of fullness in the ear or an earache A pregnant woman may experience a feeling of fullness in the ear or an earache due to vascular changes in the middle ear. Rhinitis and epitaxis may occur due to an increase in blood flow to the sinuses. Snoring may​ occur; it is caused by hyperemia of the throat. The senses of taste and smell are heightened in a pregnant​ woman, but decreased in an older adult. The auditory canal of a child is shorter than that of an adult.

The nurse is caring for a client who is malnourished. What assessment finding would the nurse expect when assessing the client​'s ​nails? ​Ridged, brittle nails Yellow discoloring of the nails Grooves in the nails Swelling around the cuticle

Ridged, brittle nails ​Ridged, brittle,​ split, or peeling nails could be due to protein or vitamin B deficiency.​ Swelling, pain, or drainage around the cuticle is caused by infection.​ Grooves, ridges, or discoloring may be associated with​ cancer, heart​ disease, liver​ disease, or other conditions. Yellowish nails may be due to prolonged use of nail polish.

What are normal findings when palpating the kidneys in an adult male ​client? Select all that apply. Right kidney is not normally palpable. Kidney is spongy and rough. Kidney is tender. Left kidney is not normally palpable. Right kidney pole is solid and smooth.

Right kidney is not normally palpable. Left kidney is not normally palpable. Right kidney pole is solid and smooth. The left kidney is not palpable in the adult male client because of its protected position in the abdomen. The right kidney is not normally palpable unless the client is very thin. If the right kidney pole is​ palpable, it will feel solid and​ smooth, not spongy and rough. Tenderness of the kidneys is not a normal finding.

The nurse listens to the heart sounds of an adult client. What statement describes describes where the nurse would place the stethoscope to auscultate the aortic valve of the​ heart? Left side of​ chest, lower sternal border Right side of​ chest, second intercostal space Left side of​ chest, fifth intercostal​ space, midclavicular line Left side of​ chest, second intercostal space

Right side of​ chest, second intercostal space The aortic valve is located on the right side of the​ chest; the stethoscope is placed over the second intercostal space. The pulmonic valve is located on the left side of the​ chest; the stethoscope is placed over the second intercostal space. The tricuspid valve is located on the left side of the​ chest; the stethoscope is placed over the lower sternal border or fourth intercostal space. The mitral valve is located on the left side of the​ chest; the stethoscope is placed around the left midclavicular line at the fifth intercostal space.

When percussing the​ liver, where do you​ begin? Right​ side, level with umbilicus Midline below the xiphoid process Level with the sixth intercostal space Left​ side, posterior to midaxillary line

Right​ side, level with umbilicus When percussing the​ liver, you start on the right​ side, level with the​ umbilicus, and move toward the rib cage. The left side posterior to the midaxillary line is where you would start when percussing the spleen. Midline below the xiphoid process is percussed when moving over a gastric bubble. The upper border of the liver lies at the sixth intercostal space and is reached after percussing from the umbilicus and moving toward the rib cage.

Mr. Abrams is 64 years old and has come to the clinic for motor function testing. What test will assess his equilibrium and​ coordination? Romberg test ​Heel-to-shin test Rapid alternating movements Walk​ heel-to-toe

Romberg test The Romberg test assesses a​ client's equilibrium and coordination. The client is asked to stand with arms at the​ sides, feet​ together, and eyes​ open, and then is asked to close the eyes. Observe the client for swaying. Gait and balance are tested by asking the client to walk across the​ room, walk​ heel-to-toe while looking straight​ ahead, walk on the​ toes, and walk on the heels. Testing rapid alternating movements involves asking the client to sit with hands on thighs with palms down then asking the client to turn the palms​ up, and continue these movements in a rapid fashion. The client should be able to do this rapidly with rhythm and smoothness. The​ heel-to-shin test assesses the​ client's motor function by observing how smoothly the client​ can, in a supine​ position, place the right heel on the left leg below the knee and slide the right heel down the shin.

The nurse is preparing to assess the​ client's motor function. What test would the nurse include in this​ assessment? Romberg test Glasgow Coma Scale Weber test Rinne test

Romberg test The Romberg test is a motor function test that assesses coordination and equilibrium. The Glasgow Coma Scale is an objective way of recording the conscious state of a person for initial and subsequent assessment. The Weber and Rinne tests assess the client​'s ears and the function of cranial nerve VIII.

When palpating the scrotal​ sac, what would the nurse expect to find in a normal adult male​ client? Scrotum is fixed to the pubic bone. Scrotum moves freely. Scrotal sac is filled with fluid. Scrotum is pink with nodules.

Scrotum moves freely. The scrotum would move freely when palpated. The scrotum would be​ pink, but there should be no nodules. The scrotum should not be fixed and the scrotal sac should not be filled with​ fluid; these would be abnormal findings.

The nurse is educating the parents of a small child about accident prevention for poisoning or choking. What statement explains why poisoning is common in small​ children? Children play with matches and can become overcome with smoke. Small children put objects in their mouths. Visual disturbances affect the ability to read medication bottles. Children are prone to snake bites.

Small children put objects in their mouths.

The nurse is performing a physical assessment on an older adult client. What quick tests can be used when assessing a​ client's hearing and ​equilibrium? Select all that apply. Ask if the client can hear. Romberg test Rinne test Weber test Whisper test

Romberg test Rinne test Weber test Whisper test The whisper test evaluates hearing acuity. The left ear is occluded and the examiner stands 1 to 2 feet from the right ear. The examiner whispers a statement without letting the client see the lips move. The client should be able to repeat the statement. In the Rinne​ test, air conduction and bone conduction are compared by placing a vibrating tuning fork on the mastoid process behind the ear. The client tells when the vibration​ stops, and the examiner measures the number of seconds it takes for the client to stop hearing the sound of the vibration. The Weber test evaluates hearing through bone conduction using the tuning fork. The vibrating tuning fork is placed on the frontal​ bone, and the client tells whether the vibration is heard in both​ ears, which would be normal. The Romberg test is used to test equilibrium. With arms out and feet​ together, the client closes both eyes and maintains balance. Simply asking if the client can hear is not proof of hearing. Older adults may deny that they are experiencing hearing loss or may not even realize that they have lost some of their hearing.

Identify the abdominal sounds based on the clinical indication. ​Instructions: Use the dropdown menus in the left column to select the abdominal sounds for each indication in the right column. Sound Indication Rubbing Vascular Sounds Bowel Sounds Intestinal obstruction Stenosis of renal arteries Paralytic ileus Aortic arterial obstruction Diarrhea Tumor Stenosis of iliac arteries Portal hypertension Gastroenteritis Inflammation of organ

Rubbing- Tumor, Inflammation of organ Vascular Sounds- Stenosis or renal arteries, Aortic arterial obstruction, Stenosis of iliac arteries, Portal hypertension Bowel Sounds- Intestinal obstruction, paralytic ileus, Diarrhea, Gastroenteritis

Identify the clinical indication of each physical assessment finding. ​Instructions: Use the dropdown menus in the left column to select the correct clinical indication for each assessment finding in the right column. Indication Finding Atherosclerosis Fluid retention Protuberant eyes Polycythemia Ruddy skin tone Periorbital edema Xanthelasma High cardiac output

Ruddy skin tone- polycythemia Periorbital edema- fluid retention Xanthelasma- atherosclerosis High cardiac output- protuberant eyes

The nurse is using a stethoscope to assess the heart of an adult client. What heart sounds would be heard in a normal cardiac ​cycle? Select all that apply. S1 and S2 are heard clearly with a stethoscope at both the base and the apex of the heart S2 as the​ "lub lub" S1 is louder than S2 at the apex of the heart S2 as the​ "dub" S1 as the​ "lub"

S1 and S2 are heard clearly with a stethoscope at both the base and the apex of the heart S1 is louder than S2 at the apex of the heart S2 as the​ "dub" S1 as the​ "lub" S1 and S2 can be heard with auscultation at the apex and the base of the heart in a normal adult. The S1 sound would be louder at the apex of the heart. S1 is the​ "lub" sound, indicating the closure of the tricuspid and mitral​ valves, and S2 is the​ "dub" sound, indicating the closure of the pulmonic and aortic valves.

The charge nurse is evaluating a student​ nurse's knowledge of the normal path of the electrical impulse in the conduction system of the heart. In what order would the student nurse place the components to show understanding of the​ pathway? ​Instructions: Click on the down arrow for each response in the right column and select the correct choice from the list. Order Responses AV node Purkinje fibers SA node Cardiac nerves Right and left bundle branches of fibers Bundle of His

SA Node AV Node Bundle of His Right and left bundle branches Purkinje fibers Cardiac nerves

Which of the following synovial joints are found in the​ hand? Select all that apply. Saddle joint Plane joint Hinge joint Pivot joint ​Ball-and-socket joint

Saddle joint Plane joint A plane joint is a joint that allows slipping and gliding​ movements, as in the intercarpal joints in the hands. A saddle joint is one in which opposing surfaces fit​ together, as in the carpometacarpal joints of the thumb. Hinge joints permit flexion and​ extension, and can be found in the elbow and knee. A pivot joint is one in which the rounded end of one bone protrudes into a ring of​ bone, as found in the atlas and axis of the neck. A​ ball-and-socket joint consists of a​ ball-shaped head of one bone that fits into the concave socket of another bone and can be found in the shoulder and hip.

The nurse is assessing the skin of a client admitted to the medicaldashsurgical unit. What findings would the nurse document as secondary ​lesions? Select all that apply. Tumor Scar Ulcer Cyst Keloid

Scar Ulcer Keloid Ulcers, scars, and keloids are considered secondary lesions and should be documented as such. Cysts and tumors are considered primary lesions.

The instructor asks the student nurse to name four inflammatory diseases that involve pain and swelling of an adult​'s joints. What diseases would the student include when ​responding? Select all that apply. Scleroderma Tendonitis Arthritis Polymyositis Bursitis

Scleroderma Tendonitis Arthritis Bursitis Arthritis, bursitis,​ tendonitis, and scleroderma are diseases in which inflammation and swelling damage a joint and cause immobility and decline in an adult​'s musculoskeletal system. Polymyositis is an inflammatory disease of the muscle​ tissue, not the​ joints, and is debilitating.

Arrange the steps of the otoscopic examination of the auditory canal of an adult client in the correct sequence. ​Instructions: Click on the dropdown arrow for each step of the procedure in the right column and select the correct choice from the list. Order Otoscopic Examination Use the opposite hand to straighten the canal by pulling the pinna up, back, and out. Instruct the client to tell you if there is any discomfort but do not move or pull away. Have the client tilt the head away from you toward the opposite shoulder. Hold the otoscope between the palm and first two fingers of dominant hand. With the otoscope light on, use upward or downward position of the handle to insert speculum into the ear. Select largest speculum that will fit into the auditory canal.

Select largest speculum that will fit into the auditory canal. Have the client tilt the head away from you toward the opposite shoulder. Hold the otoscope between the palm and first two fingers of dominant hand. Use the opposite hand to straighten the canal by pulling the pinna up, back, and out. With the otoscope light on, use upward or downward position of the handle to insert speculum into the ear. Instruct the client to tell you if there is any discomfort but do not move or pull away.

Identify the function of the cranial nerves that deliver impulses to and from the brain as​ sensory, motor, or fixed based on their activity. ​Instructions: Use the dropdown menus in the left column to select the function of each cranial nerve in the right column. Function Cranial Nerve Sensory Motor Mixed Olfactory Trochlear Optic Abducens Oculomotor Glossopharyngeal Hypoglossal Facial Trigeminal Accessory Vagus Vestibular

Sensory- olfactory, optic, vestibular Motor- trochlear, oculomotor, hypoglossal, accessory Mixed- abducens, glossopharyngeal, facial, trigeminal, vagus

What disorders would the nurse consider with​ itching, burning, and swelling to the genital area of a male ​child? Select all that apply. Erectile dysfunction Sexual abuse Trauma Infection of the urinary tract Pinworms

Sexual abuse Trauma Infection of the urinary tract Pinworms Known or unknown​ trauma, sexual​ abuse, and pinworms can cause​ swelling, burning, or itching of the genitals. Urinary tract infections can cause itching and burning. Erectile dysfunction is not a disorder of childhood and would not present with itching.

The nurse is preparing to inspect the skin of a client in the clinic. Inspection is used to assess what aspects of the ​skin? Select all that apply. Shape Temperature Consistency Color Hygienic factors

Shape Consistency Color Hygienic factors Inspection is used to determine the​ color, consistency,​ shape, and​ hygiene-related factors of the skin. Skin temperature is determined by using palpation.

Mr. Feinman has a new diagnosis of​ left-sided heart failure and​ states, "What symptoms am I likely to have because of​ this?" You would describe which​ symptoms? Peripheral edema and jugular vein distention Shortness of breath and crackles in the lungs Hypertension and angina pain Ascites and tachycardia

Shortness of breath and crackles in the lungs Mr. Feinman would typically experience shortness of breath and lung crackles due to the backup of blood into the pulmonary system.​ Ascites, peripheral​ edema, and jugular vein distention are symptoms of right heart failure. Tachycardia can be a sign of right or left heart failure. Hypertension would be a sign of​ right-sided heart​ failure, and angina pain is a symptom of ischemia.

Identify the type of pain based on the clinical manifestation. ​Instructions: Use the dropdown menus in the left column to select the type of pain for each clinical manifestation in the right column. Type of Pain Manifestation Nociceptive Neuropathic Skin abrasion Lower leg​ numbness, tingling from poorly controlled diabetes Sunburn Sciatica Acute appendicitis Phantom limb pain Fractured humerus Trigeminal neuralgia

Skin abrasion- Nociceptive Lower leg​ numbness, tingling from poorly controlled diabetes- Neuropathic Sunburn- Nociceptive Sciatica- Neuropathic Acute appendicitis- Nociceptive Phantom limb pain- Neuropathic Fractured humerus- Nociceptive Trigeminal neuralgia- Neuropathic

A​ 15-year-old girl is seen at the primary care clinic for a checkup. The examining nurse takes a health history. The client tells the nurse that she feels she is​ "so fat." The client is 5​' 2​½​" tall and weighs 112 lbs. What tool would be the most accurate to determine if there is a problem with the adolescent​'s ​weight? Weight measurement ​X-rays of the long bones Waist circumference Skinfold measurements

Skinfold measurements Skinfold thickness measurements can estimate subcutaneous body fat stores. Measurements taken at up to eight sites on the body are believed to be predictive of overall body fat composition. Skinfold measurements are made using grade calipers and a flexible measuring tape. Weight alone is not an indicator of problems for this adolescent. Waist circumference indicates cardiovascular disease risk.​ X-ray of the long bones will not provide information about the appropriateness of the client​'s weight.

Which of the following behaviors may contribute to the condition of a client who has musculoskeletal​ pain? Select all that apply. Takes no medications Smokes ​Long-distance runner Primarily works on a computer Lives alone

Smokes ​Long-distance runner Primarily works on a computer A​ long-distance runner is predisposed to trauma or​ "wear and​ tear" injuries that are common in that sport. Frequent repetitive movements​ (like when working at a​ computer) may lead to misuse syndromes such as carpal tunnel syndrome​ (inflammation of the tissues of the wrist that causes pressure on the median​ nerve). Smoking increases a​ person's risk of osteoporosis. Someone who lives alone or does not take any medications does not have an increased risk for musculoskeletal pain

When assessing an​ adult, what lifestyle factors would the nurse assess that directly influence the respiratory health of the adult​ client? Smoking history Weight Age Sexual orientation

Smoking history Smoking has a direct influence on the healthy state of an adult​ client's lungs and respiratory health. Factors such as​ age, weight, and sexual orientation may come up during the assessment but have less to do with the state of the lungs.

The nurse is assessing a client in her third trimester of pregnancy. What effects of estrogen and progesterone on the musculoskeletal system would the nurse expect to​ assess? Brittle bones Hardening of the bones Nothing—estrogen and progesterone have no effect on the bones Softening of the bones

Softening of the bones Estrogen and progesterone soften a pregnant female​'s ​bones, especially the pelvic​ bones, in preparation for delivery. The bones do not become​ brittle, nor do they become harder during pregnancy. As the weight of the pregnancy​ increases, the pregnant client will take on a waddling walk because the pelvic bones have softened.

The sounds auscultated are described in terms of​ intensity, pitch,​ duration, and quality. Identify the term based on the description. ​Instructions: Use the dropdown menus in the left column to select the correct term for each description in the right column. Term Description pitch duration intensity Sound depends upon the force of the blow Caused by the number of vibrations per second Length of time a sound is heard

Sound depends upon the force of the blow- intensity Caused by the number of vibrations per second- pitch Length of time a sound is heard- duration

Which of the following are major bones of the​ head? Select all that apply. Hyoid Sphenoid Manubrium Ethmoid Zygomatic

Sphenoid Ethmoid Zygomatic The​ zygomatic, ethmoid, and sphenoid bones are all major bones of the head that allow for movement of the mandible at the temporomandibular joint. The hyoid bone is found in the neck. The manubrium is the superior portion of the sternum.

The nurse is assessing the client​'s skin. What assessment finding would the nurse recognize as the result of a possible dietary​ deficiency? Spider angioma Hematoma Hemangioma Venous lake

Spider angioma Spider angiomas are a type of vascular dilation that can be caused by a vitamin B deficiency. Hemangiomas are clusters of immature capillaries. Venous lake is caused by increased intravenous pressure in the superficial veins. Hematomas are caused by the leakage of blood into the skin and subcutaneous tissue as a result of trauma or surgical incisions.

The nurse is preparing to inspect the client​'s scalp and hair. What action by the nurse indicates a need for further​ teaching? Starts the assessment by inspecting the scalp for lesions by using a Wood​'s lamp Explains the procedure to the client prior to touching the client Observes the client​'s hair color Examines the distribution of hair over the scalp

Starts the assessment by inspecting the scalp for lesions The scalp is inspected for lesions using a Wood​'s ​lamp; however, the first step in any assessment is for the nurse to explain to the client what the nurse is going to do and why. The client​'s hair is observed for color. Color should be consistent throughout. Hair should be evenly distributed over scalp.

An abnormal finding of a bruit when auscultating a​ client's temporal artery might indicate what​ condition? Stenosis of the vessel Increased blood flow Inflammation Benign or malignant mass

Stenosis of the vessel The finding of a bruit when auscultating a​ client's temporal artery might indicate stenosis​ (or narrowing) of the vessel. A bruit would not indicate​ inflammation, increased blood​ flow, or a benign or malignant mass.

The inability to identify a familiar object by touch with the eyes closed is an abnormal finding on what sensory function​ test? Topognosis test Graphesthesia test Kinesthesia test Stereognosis test

Stereognosis test An abnormal finding on the stereognosis test is the inability to identify a familiar object by touch with the eyes closed. The inability to identify a number drawn on the palm of the hand is an abnormal finding of the graphesthesia test. An abnormal finding on the topognosis test is the inability to identify which portion of the body has been touched. An abnormal finding on the kinesthesia test is the inability to identify the movement of a joint into​ dorsiflexion, plantar​ flexion, and abduction. Next Question

You are performing a musculoskeletal assessment on a client who has come into the clinic. What information would be considered a significant risk factor for a client with musculoskeletal​ issues? Steroid use Annual flu inoculation Daily calcium supplement Seasonal allergy medication

Steroid use Steroids affect the musculoskeletal system by increasing muscle size.Allergy medication and an annual flu shot are not significant risk factors for a client with musculoskeletal issues. A daily calcium supplement helps slow the development of​ age-related musculoskeletal changes such as osteoporosis.

Dullness is a​ short, high-pitched sound heard on percussion during an assessment of the abdomen. What abnormal circumstance would the nurse suspect when assessing an adult client with dullness over the left lower quadrant​ (LLQ)? Stool in the colon An enlarged liver A mass in the stomach or a full stomach Splenomegaly

Stool in the colon Dullness over the LLQ indicates that stool is in the colon. The dullness indicating an enlarged liver would be assessed below the right costal margin. The dullness indicating an enlarged spleen would be found exceeding the left anterior axillary line. The dullness indicating a stomach mass or a full stomach immediately following a meal would be assessed at the midsternal line below the xiphoid process.

The nurse is preparing to assess a​ 33-year-old client​'s skin. What psychosocial condition could affect the client​'s ​skin? Hypertension Diabetes Acne Stress

Stress Stress can exacerbate certain skin conditions such as rashes or acne. Hypertension is a physiological condition. Acne is a skin disturbance that is a physiological disease. It can be exacerbated by stress. Diabetes is a physiological​ disease, not a psychosocial condition.

Which type of rhonchi is indicated by a​ loud, high-pitched crowing sound heard on​ inspiration? Wheezes Friction rub Rhonchi Stridor

Stridor Stridor is indicated by a​ loud, high-pitched crowing sound that occurs on inspiration and can be heard without a stethoscope. Wheezes​ (sibilant) are high pitched and​ continuous, and when severe they are heard on expiration and inspiration. Rhonchi are heard on expiration and inspiration and are​ low-pitched, continuous​ "snoring," rattling sounds. Friction rub is indicated on inhalation and exhalation by a​ low-pitched grating and rubbing sound.

The nurse is performing a breast examination on a​ 32-year-old client. During palpation of the breasts and​ axillae, which lymph nodes will the nurse​ assess? Select all that apply. Subclavicular Central axillary Interpectoral Inguinal Brachial

Subclavicular Central axillary Interpectoral Brachial The​ subclavicular, brachial, central axillary and interpectoral lymph nodes are located in the breast and axilla. When palpating the breast and​ axillae, these nodes are usually nonpalpable. The inguinal lymph node is located in the groin and is not assessed during a breast examination.

Which of the following lymph nodes of the head and neck are located behind the tip of the mandible at the​ midline? Retropharyngeal Superficial cervical Submental Submaxillary

Submental The submental lymphatics are located behind the tip of the mandible at the midline. The submaxillary lymphatics are on the medial border of the mandible. The retropharyngeal​ (tonsillar) lymphatics are at the junction of the posterior and lateral walls of the pharynx at the angle of the jaw. The superficial cervical lymphatics are anterior to and over the sternocleidomastoid muscle.

What lymph nodes are palpated during a breast and axillae​ assessment? Cervical nodes Supraclavicular nodes Central axillary nodes Inguinal nodes Interpectoral nodes

Supraclavicular nodes Central axillary nodes Interpectoral nodes The​ supraclavicular, interpectoral, and central axillary nodes are all palpated when performing a breast and axillae assessment. The inguinal lymph nodes are located in the groin and the cervical nodes are located in the necklong dashthese nodes are not part of the assessment.

The nurse is assessing the clavicles of an older child. What finding on palpation would be an abnormal finding for a child and might indicate​ trauma? Swelling or bruising over the right clavicle A butterfly tattoo on the clavicle Fine hair over the chest and shoulder or clavicle area that appears evenly distributed on both sides of the chest Pink skin over the clavicle and chest​ wall, and the absence of swelling

Swelling or bruising over the right clavicle Both the right and the left clavicles would show no swelling or bruising in the exam of a normal child. Swelling or bruising over one clavicle indicates trauma or possible fracture. Pink skin tones over the clavicle and chest wall without swelling on both the right and left sides would be a normal finding on palpation and inspection. A tattoo of a butterfly on the clavicle area of a child without signs of other deformity or swelling would not be considered an abnormal​ finding, but would be a moral judgment​ and, unless it caused the clavicle to be​ harmed, would not be considered harmful but would be documented in the record. Fine​ hair, if evenly​ distributed, would not be an abnormal sign for a child and is unrelated to trauma.

When inspecting a client​'s ​face, which of the following would be normal​ findings? Select all that apply. Symmetrical eyebrows Symmetrical nasolabial folds Involuntary facial movements Purposeful facial movements Facial expressions appropriate to situation

Symmetrical eyebrows Symmetrical nasolabial folds Purposeful facial movements Facial expressions appropriate to situation When inspecting the​ face, normal findings include facial expressions that are appropriate to the​ situation, as well as symmetrical​ eyebrows, nasolabial​ folds, and sides of the mouth. The client​'s facial movements should be purposeful and without involuntary movements. Involuntary facial movements can be indicative of a neurologic or psychological disorder.

Mrs. Baylor is a​ 57-year-old woman who has come into the clinic for a breast exam. During the exam you feel an enlarged lymph node in the upper outer quadrant of the​ client's left breast. What structure was palpated to determine this​ finding? Montgomery glands Tail of Spence Cooper ligaments Mammary ridge

Tail of Spence The tail of Spence​ (or axillary​ tail) is breast tissue that extends superiolaterally into the axilla. Cooper ligaments​ (suspensory ligaments) extend from the connective tissue layer through the breast and attach to the fascia underlying the breast. Montgomery glands are tiny sebaceous glands speckled on the surface of the areola. The mammary ridge extends from each axilla to the groin.

The nurse is preparing to assess the​ client's respiratory rate. Which assessment finding would be the cause of an increase in the​ client's respiratory​ rate? Temperature 39.2degrees°C ​(102.5degrees°​F) Age 32 Caucasian Blood pressure​ 110/75 mmHg

Temperature 39.2degrees°C ​(102.5degrees°​F) An elevated temperature can cause the​ client's respiratory rate to increase. A normal blood pressure reading will not affect the​ client's respiratory rate. The​ client's normal range for respiratory rate will change with​ age; however, it will decrease as the client becomes an​ adult, and this would not cause the respiratory rate to increase. The​ client's ethnicity will not increase the respiratory rate.

The nurse is preparing to teach an​ in-service regarding the skin. Which function of the cutaneous glands would the nurse include in the​ teaching? Provide skin pigmentation Toughness Temperature regulation Provide elasticity

Temperature regulation The cutaneous glands regulate temperature through the evaporation of perspiration on the skin surface. The melanin is responsible for skin pigmentation. Collagen gives skin its toughness and enables it to resist tearing. Elastic fibers give the skin its elasticity.

The nurse is assessing the skull of a child. What are the major bones of the cranium that form the ​skull? Select all that apply. Temporal bone Frontal bone Parietal bone Ethmoid bone Occipital bone

Temporal bone Frontal bone Parietal bone Occipital bone The frontal bone forms part of the forehead. Parietal bones form part of the skull on both lateral sides of the head. Temporal bones are found on both sides of the skull. The occipital bone forms the posterior portion of the skull. These skull bones should be fused by childhood. The ethmoid bone is one of the major bones in the face.

When inspecting a​ client's salivary​ glands, what would be an abnormal​ finding? Ducts visible Flow of saliva No swelling Tenderness of glands

Tenderness of glands When inspecting a​ client's salivary​ glands, a normal finding would be no tenderness of the glands. Ducts should be visible and​ intact, and there should be a normal flow of saliva when the area close to the duct is touched with a sterile applicator. There would be no​ swelling, pain, or redness.

The nurse is assessing the head and neck of an adult client who presents with a​ new-onset headache and who has no​ pre-existing conditions. What types of headache would the nurse need to consider during the assessment and ​interview? Select all that apply. Tension headache Cluster headache Tumor headache Classic migraine headache Sinus tension headache

Tension headache Cluster headache Classic migraine headache Sinus tension headache Classic migraine headaches are usually preceded by an aura. Cluster headaches are several headaches that occur over a period of time but are not associated with an aura. Tension headaches may be due to stress or muscle strain. Sinus tension headache is caused by muscle strain from a sinus infection. Headaches from a tumor or chronic illness would be a​ pre-existing condition, and the question presumes the client has no previous illness that leads to headache.

What tests would you employ when inspecting cranial nerve V​ (trigeminal)? Have client puff out the cheeks as you try to push out the air by applying pressure. Have client shrug shoulders. Test near vision with the Snellen chart. Test corneal reflex with a wisp of cotton.

Test corneal reflex with a wisp of cotton. One of the ways to test cranial nerve V​ (trigeminal) is to test the corneal reflex by lightly touching the cornea with a wisp of cotton. Having the client puff out the cheeks as you try to push out the air by applying pressure is used as a test of cranial nerve VII​ (facial). Cranial nerve II​ (optic) is tested using the Snellen chart to assess the client​'s near vision. Having the client shrug the shoulders tests cranial nerve XI​ (accessory), which tests the trapezius muscle.

What structure of the male genitourinary system is responsible for producing​ sperm? Testicle Spermatic cord Scrotum Epididymis

Testicle The male gonad organ or testicle is responsible for producing both sperm and testosterone. The spermatic cord is the structure formed by the ductus deferens and surrounding tissue and runs from the abdomen down to each testicle. The scrotum is the sack that contains the testicles. The epididymis is the​ crescent-shaped tissue that forms the male duct system and stores and transports sperm as they mature.

What congenital cardiac birth defect is characterized by the dextroposition of the​ aorta, pulmonary​ stenosis, right ventricular​ hypertrophy, and a ventricular septal​ defect? Septal wall defect Congestive heart failure Electrical rhythm defects Tetralogy of Fallot

Tetralogy of Fallot Tetralogy of Fallot is the dextroposition of the​ aorta, pulmonary​ stenosis, right ventricular​ hypertrophy, and a ventricular septal defect. It can be noted on an ultrasound during prenatal​ care, but it can also be missed and not discovered until birth when the infant shows symptoms. It is a​ life-threatening congenital defect and requires surgical​ repair, often in more than one surgery. Tetralogy of Fallot does not include an electrical​ component, and congestive heart failure is not one of its defects but a possible result if the defects are not corrected.

The nurse is caring for an adult client who is aphasic. What would the nurse include when planning for the safety of the​ client? The client​'s emotional state does not affect risk for injury. The ability to communicate needs The adult client is not at risk for falls. The client​'s environment is not part of the nursing care plan.

The ability to communicate needs The client​'s ability to communicate needs would affect the risk for injury and safety of the client. The adult client who is aphasic may be at risk for falls. The client​'s emotional state may affect​ safety, as would the client​'s ​environment, and both would be considered when planning the care of the client.

What findings would the nurse expect in a normal cardiovascular exam of a ​child? Select all that apply. The carotid pulse would be visible if the child is sitting at a​ 45-degree angle. The skin would be​ pale, dusky, or clammy. The face would be pink and skin tone even. The fingernails would be pink. The sclera would be slightly tinted or cloudy.

The carotid pulse would be visible if the child is sitting at a​ 45-degree angle. The face would be pink and skin tone even. The fingernails would be pink. In a normal cardiovascular assessment of a child the nurse would find a pink face and​ skin, pink nail​ beds, and a visible pulsation of the carotid arteries bilaterally when the child sits at a​ 45-degree angle.​ Good, strong pulses and a heart rate within normal range would also be expected in a normal exam. The sclera of a child with good cardiovascular health would be white and clear with no drainage. The skin of a normal child would not be dusky. A child may have fair skin but would have pink tones. The child would not be clammy.

The nurse is using inspection and palpation to examine the neck of an adult client. What normal findings would the nurse expect to ​find? Select all that apply. The client can move the neck and touch the chin to the chest. The trachea is obvious on inspection. The client can turn the head back and forth against resistance. The client can shrug the shoulders. The trachea is not palpable.

The client can move the neck and touch the chin to the chest. The trachea is obvious on inspection. The client can turn the head back and forth against resistance. The client can shrug the shoulders. The trachea would be obvious and midline in the neck. The client would be able to touch the chin to the chest easily. The client would be able to shrug the​ shoulders, which tests cranial nerve XI function. The client would be able to turn the head back and forth against​ resistance, which gives information about the function of cranial nerve XI. The trachea would be palpable and would move when the client swallows.

Which principles should the nurse include in the plan of care related to pain control for an older adult ​client? Select all that apply. The use of narcotics should be avoided. The client may describe pain as an​ "ache" or​ "discomfort." The client may deny pain. Pain is a natural outcome of the aging process. Pain perception increases with age.

The client may describe pain as an​ "ache" or​ "discomfort." The client may deny pain. Older clients may withhold reports of pain because it may indicate a worsening of their​ condition, which threatens their independence. Older adults may also describe pain​ differently, and use words other than​ "pain." Pain is not a natural outcome of the aging process and is related to some type of pathology. Pain perception does not increase with​ age; it actually decreases as part of the aging process. Although the dosage of narcotics may need to be altered with older​ adults, especially those with reduced kidney or hepatic​ function, narcotics do not need to be avoided.

A client arrives on the unit from the surgical recovery room. What would the nurse not consider when planning pain relief for this​ client? Previous pain may influence current pain. Developmental stage would affect this client​'s pain. Providing a​ quiet, calm environment. The client​'s pain level prior to surgery

The client​'s pain level prior to surgery Reported pain prior to surgery should be considered independent and separate from​ post-surgical pain. The client​'s pain should be assessed related to the current clinical state. There is no indication that the client has any developmental problems that would affect the client​'s pain. Cultural background affects the amount of pain an individual is willing to tolerate. Members of some ethnic groups are more stoic and less expressive when experiencing pain. Being in a strange environment such as a hospital with the​ noises, lights, and activity can impact clients​' perception of their pain. Some clients accept the pain experience more easily than others depending on the circumstance. In​ birth, the pain experience has an​ ending, whereas in knee surgery the client does not know how long the pain will exist or if there will be a lifestyle change due to chronic pain.

The nurse is working with a​ 35-year-old client. The nurse instructs the client to keep a food diary beginning on the next day. The nurse would instruct the client to record food intake on what other​ days? Every other day for a total of 3 days The first​ Thursday, Friday, and Sunday following the visit One day every 2 weeks The next 7​ days, including the weekend

The first​ Thursday, Friday, and Sunday following the visit Recording dietary intake for two sequential weekdays and a weekend day works​ well, as two days of the diary should be sequential. Food diaries longer than 3 days in length tend to be recorded retrospectively with a loss of accuracy. Including some days in succession helps to reveal patterns.

The uterus of a pregnant female enlarges and moves from the pelvis to the abdomen. At what week of gestation would the nurse be able to palpate the fundus of the​ uterus? The​ thirty-sixth week The fourteenth week The sixth week The twentieth week

The fourteenth week At 14 weeks of​ gestation, the fundus of the pregnant uterus can be felt at the level of the pubic bone. The fundus of the uterus is not felt before this time because it is lower in the pelvis. At the​ thirty-sixth week, the uterus takes up most of the abdominal cavity and reaches almost to the​ diaphragm, far beyond the pubic bone. The nurse can palpate the fundus of the uterus at 14​ weeks, which is well before the twentieth week of pregnancy.

The nurse is assessing the apical heart rate of an​ 8-year-old child. What statement is incorrect about the normal heart of a​ child? The apex of the heart of a child is at the fifth intercostal space and the midclavicular line. The heart reaches the adult position in the chest at the age of 13 or puberty. The PMI for a child can be one intercostal space above that of an adult. The heart rate can be palpable at the fifth intercostal space and the midclavicular line.

The heart reaches the adult position in the chest at the age of 13 or puberty. The heart reaches the adult position by the age of 7 and can best be palpated in the chest at the fifth intercostal space and left midclavicular line. Depending on the weight of the​ child, it may be that PMI is found one intercostal space above that of an adult when assessing.

What is an abnormal finding when assessing the kidneys in a healthy​ client? There are no​ protrusions, masses, or elevations. Client does not have any pain or discomfort. The left kidney is always palpable. Skin color is uniform with the back.

The left kidney is always palpable. The left kidney is not normally palpable. In a healthy​ client, the skin color of the abdomen would be uniform with the​ back; there would be no​ protrusions, masses, or​ elevations; and the client would not have any pain or discomfort while being assessed.

You are a novice nurse assessing Ms.​ Tyler, who came in to your urgent care clinic for evaluation of her newly fractured right arm. In​ addition, she complains of a headache. The charge nurse is available to provide guidance to you. Which action by you requires further education by the charge​ nurse? The splint is removed for evaluation of the arm The door to the​ client's room is closed The​ client's permission is obtained for the assessment The lights are dimmed for client comfort

The lights are dimmed for client comfort The lights should not be dimmed. The client needs to be assessed in adequate lighting. The nurse should obtain the​ client's permission for assessment and provide for privacy by closing the door. The splint should also be removed to assess skin integrity.

The nurse is assessing the nose of an adult client. The client asks why assessing the inside of the nose is important. What would the nurse include when explaining the purpose and function of the nose to the​ client? Select all that apply. The nose warms and humidifies air. The nose acts as a filter. The nose is essential to life. The nose is an airway. The nose allows for smelling.

The nose warms and humidifies air. The nose acts as a filter. The nose is an airway. The nose allows for smelling. The nose is the pathway to the lungs and establishes an open airway for breathing. The nose​ filters, warms, and humidifies air taken into the lungs during breathing and prevents particles from entering the respiratory tract. The nose also houses centers for​ olfaction, or smelling. The nose is an​ airway, but it is not essential to life. There are other ways to establish an airway if the nose is obstructed or destroyed.

The nurse is inspecting the hips of an older client. The nurse finds slight swelling on the right hip​ area, and the right leg is shorter than the left. What action indicates that the nurse understands the principles of palpating and positioning an older client with a possible injury to the​ hip? The nurse puts slight pulling pressure on the right leg when palpating the right hip area. The nurse uses abduction and adduction to assess the movement of the hip joint. The nurse puts the client​'s hip through full range of motion regardless of pain. The nurse gently palpates the injured hip and uses the log rolling technique to complete the assessment.

The nurse gently palpates the injured hip and uses the log rolling technique to complete the assessment. The nurse who understands the principles of palpation knows a consideration when palpating is to cause no further pain or injury. Log rolling a client to assess the posterior aspect of the swollen area is appropriate. Putting a client through full range of motion or abduction or adduction movements without an​ x-ray may cause further injury.

The nurse is assessing an older child who has been treated frequently for fractures of the long bones. The child presents with bruises and another fracture. What would be an important consideration when assessing this​ child? The nurse would consider educating the older child on safety measures to prevent further fractures. The nurse would consider the number of siblings in the home. The nurse would consider prescribing vitamins. The nurse would consider the possibility of physical abuse.

The nurse would consider the possibility of physical abuse. The nurse would consider the possibility of physical abuse whenever a client presents with injuries and has a past history of many fractures. Health care providers are obligated by law to report suspected physical abuse. Education on​ safety, prescribing​ vitamins, and questioning about siblings may be part of the overall treatment​ plan, but the assessment must immediately rule out the possibility of physical abuse.

The nurse is assessing a youth who complains of neck and shoulder pain. What would be the first action of the nurse when assessing this​ client? The nurse would palpate the neck first. The nurse would palpate the shoulders first. The nurse would ask the client to bring the chin to the chest. The nurse would inspect the neck and​ shoulders, and compare the right side to the left side.

The nurse would inspect the neck and​ shoulders, and compare the right side to the left side. Before​ palpation, the nurse would inspect the area and compare sides for abnormal signs of injury or swelling. Inspection of the neck and shoulders is the first step in assessing the musculoskeletal system to visualize any abnormalities. Because this could cause further pain and​ injury, the nurse would not ask the client to bring the chin to the chest before inspecting the area for abnormalities.

What would the nurse include when using the skill of inspection to determine the state of the adult client​'s musculoskeletal ​system? Select all that apply. The nurse would observe the finger joints for swelling or redness. The nurse would palpate the neck and shoulders for lumps. The nurse would observe the areas of joints such as​ shoulders, elbows, and knees for deformities. The nurse would inspect the client from head to​ toe, looking at the client​'s posture and stance. The nurse may have the client walk a few steps while observing for balance and gait.

The nurse would observe the finger joints for swelling or redness. The nurse would observe the areas of joints such as​ shoulders, elbows, and knees for deformities. The nurse would inspect the client from head to​ toe, looking at the client​'s posture and stance. The nurse may have the client walk a few steps while observing for balance and gait. The nurse would do a​ head-to-toe inspection of the​ joints, muscles, and bones of the adult client. It is important to look at​ balance, skin​ color, swelling, or deformity on one side or the​ other, and the overall way the client sits and stands. Having a client walk a few steps would give information about the​ weight-bearing joints and balance. It is important to note a difference in the​ contour, color, or shape of an extremity on the left side compared to the right side. Palpation is the second assessment technique for the musculoskeletal system.

The nurse palpates the chest wall of an adult client. The client​ states, "Wow, it hurts when you touch around this​ area." The client points to the​ mid-clavicular line of the right chest wall. What would be the best action by the​ nurse? The nurse would recognize this finding as abnormal and gently palpate the region again. The nurse would continue to palpate but with a firm hand. The nurse would ignore the complaint of pain and continue. The nurse would stop the exam.

The nurse would recognize this finding as abnormal and gently palpate the region again. It would be appropriate for the nurse to continue with gentle palpation in the area to determine the size and extent of the area of pain. The nurse would then continue with the exam by palpating the opposite side of the chest to determine if there is pain in that area. Stopping the exam or ignoring the complaint would not assist the nurse in discovering the source of the pain. Continuing with the exam would help to determine if a consultation with other health care providers is going to be needed to address this pain.

The nurse is performing an initial assessment on a client. Which data would the nurse collect when assessing the client​'s ​pulse? Select all that apply. The pattern of beats and intervals between beats A swooshing sound The number of beats per minute The force of each beat The elasticity of the arterial wall

The pattern of beats and intervals between beats The number of beats per minute The force of each beat The elasticity of the arterial wall The pulse rhythm is the pattern of the beats and the intervals between the beats. The nurse should assess this every time the pulse is measured. Pulse​ volume, also called the pulse strength or​ amplitude, refers to the force of each beat determined by the force with which blood is pumped through the artery. The nurse also should assess this every time the pulse is assessed. The elasticity of the arterial wall reflects its expansibility or its deformities. A​ healthy, normal artery feels​ straight, smooth,​ soft, and pliable. Older adults often have inelastic arteries that feel twisted​ (tortuous) and irregular upon palpation. The heart rate of the pulse is measured as the number of pulsating sensations felt per minute. The apical pulse is the only pulse that is usually auscultated and it does not normally reveal a swooshing sound.

Which statements are true regarding documentation in a​ long-term care​ setting? Select all that apply. Documentation of the​ client's progression toward goals or outcomes is required every 3 months. Visits and phone calls received are not a routine part of the record. The plan of care is revised every 3​ months, at a minimum. Documentation should include measures implemented in response to changes in a​ client's condition. Within 24 hours of a​ client's condition​ change, the health care provider and​ client's family must be notified.

The plan of care is revised every 3​ months, at a minimum. Documentation should include measures implemented in response to changes in a​ client's condition. Within 24 hours of a​ client's condition​ change, the health care provider and​ client's family must be notified. The plan of care is revised every 3​ months, or more often if the​ client's condition changes. If a​ client's condition​ changes, the health care provider and​ client's family must be​ notified; any measures implemented to address these changes should be documented. Documentation of the​ client's progression toward goals or outcomes should be completed​ routinely, not just every 3 months. Any visits and phone calls should be recorded in the progress notes of the​ client's chart.

Mr. Heller was transferred to your​ long-term care facility for rehabilitation after a hip fracture. What principles of documentation would you consider when documenting in Mr.​ Heller's chart? Progress notes should be​ vague, if​ possible, to be more accurate. Document any change in Mr.​ Heller's condition to the health care provider and​ client's family within 4 hours. Documentation of a change in client condition should occur only if the change is significant. The plan of care should be revised every 3 months or whenever Mr.​ Heller's health status changes.

The plan of care should be revised every 3 months or whenever Mr.​ Heller's health status changes. In​ long-term care, the plan of care is revised when the​ client's status​ changes, or at minimum every 3 months. Changes in the​ client's condition should be reported within 24 hours. Changes in the​ client's condition may be subtle and​ small, but should be documented regardless of whether you deem them as significant or not. Documentation should always be clear and​ factual, not vague.

What is a normal finding when assessing the kidneys of a very thin female​ client? The kidney is smooth and tender on palpation. The right kidney pole may be palpable. The​ kidneys, when​ palpable, are soft and spongy. The left kidney is palpable.

The right kidney pole may be palpable. The right kidney pole may be palpable in a very thin client and would be considered normal. The left kidney is normally not palpable. The right kidney​ pole, if palpable in a very thin​ client, would feel​ smooth, firm, and nontender.

The nurse is evaluating the pain level of a client with somatic pain. Which would the nurse​ assess? Respirations The renal system Liver tenderness The skeletal system

The skeletal system Somatic pain is a subcategory of physiologic pain that includes the skin and musculoskeletal systems. Visceral pain describes pain of organs such as the​ lungs, liver, or renal systems.

The charge nurse sends a student nurse to do a respiratory assessment of a child. What action demonstrates the student nurse​'s ability to properly assess the​ child? The student nurse inspects the child for signs of a rash. The student nurse palpates the child​'s extremities for lumps or masses. The student nurse places a stethoscope on each side of the child​'s chest and listens for breath sounds and air​ movement, and counts respirations. The student nurse uses percussion to assess the abdomen of the child.

The student nurse places a stethoscope on each side of the child​'s chest and listens for breath sounds and air​ movement, and counts respirations. The student nurse would use a stethoscope and auscultation to determine airflow in the lungs. Percussion on the anterior and posterior chest would be done to assess for sounds that might indicate fluid or too much air in the lungs below the chest wall. Palpation is done to assess for​ tenderness, masses, or lumps on the chest. The student nurse would inspect the rise and fall of the chest for the rate and depth of respirations and any signs of respiratory distress. Other things to inspect for during a respiratory exam would be color and retractions.

Which of the following are correct statements about methods used to assess core​ temperature? Select all that apply. The use of a rectal thermometer in a client who is actively having seizures is contraindicated. The oral method is most accurate when glass thermometers are utilized. A temporal thermometer scans across the temporal artery in the neck. The axillary temperature method is least invasive and most accurate. The tympanic route assesses temperature via the ear canal.

The use of a rectal thermometer in a client who is actively having seizures is contraindicated. The tympanic route assesses temperature via the ear canal. The tympanic route is a​ safe, quick way to assess core body temperature. The use of a rectal thermometer in a client who is actively having seizures is contraindicated. The mucosa of the rectum can be perforated if a client is​ confused, does not comprehend the demands being asked of​ him/her, and physically resists care. It can also occur during the uncontrolled muscle contractions during seizure activity. Although the axillary method is equally safe and​ quick, axillary temperatures are not as accurate as those assessed using other methods. Glass thermometers can break​ easily, leading to potential mercury exposure to the client and the​ nurse; they are unsafe and are not utilized for assessment. A temporal thermometer scans over the forehead across the temporal artery.

The nurse is assessing the abdomen of an adult client with above average weight. What characteristic of obesity would make it difficult for the nurse to palpate the abdomen of this​ client? Solid tissue or​ fluid-filled tissue Accumulation of fluid in the abdomen Accumulation of air in the colon Thickened abdominal wall and fat deposits in the mesentery and omentum

Thickened abdominal wall and fat deposits in the mesentery and omentum Obesity is characterized by a thickened abdominal wall and fatty deposits in the mesentery and omentum that make it difficult to palpate the organs. Accumulation of fluid in the abdomen is ascites. Accumulation of air in the colon is gas. Solid tissue or​ fluid-filled tissue is a cyst or mass.

When palpating a client​'s breasts you notice a thickening of the skin in one area. What condition might this​ indicate? Lymphoma Intraductal papilloma Galactorrhea Underlying carcinoma

Underlying carcinoma Thickening of the skin in a particular area on the breast may indicate an underlying carcinoma. Galactorrhea is lactation that is not associated with breastfeeding. Intraductal papilloma is a benign tumor that grows in a milk duct in the breast. Lymphoma may be indicated if the client has​ hard, fixed axillary lymph nodes.

The nurse is caring for a client in the diabetic care clinic. After the general​ assessment, the nurse asks the client to verbally recall everything the client consumed in the last 24 hours. What are the advantages of using diet recall with this ​client? Select all that apply. This assessment can be done with clients who have low literacy. It can be performed quickly. The client will be open about poor nutritional habits during the recall. Food recall allows for the determination of the use of fortified foods. The assessment will assist the nurse in identifying normal eating habits.

This assessment can be done with clients who have low literacy. It can be performed quickly. The advantages of a​ 24-hour food recall are it can be performed quickly and be used with clients who have low literacy because it is done verbally.​ Disadvantages are that its accuracy is based on the client​'s openness during the recall and the inability to review food labels to determine nutrient content. A​ 24-hour recall allows the nurse to identify foods eaten in that time period and does not pick up normal eating habits.

Which information would the nurse include when assessing the family history of a client to be documented as a​ genogram? The​ client's siblings and children only Three to five generations The​ client's nuclear family only One or two generations of relatives

Three to five generations The family history is a review of the health of members of the​ client's family to determine if any genetic or familial patterns of health or illness might shed light on the​ client's current health status. The family history begins with a review of the immediate​ family, parents,​ siblings, children,​ grandparents, aunts,​ uncles, and cousins. The client would be encouraged to remember as much information about as many generations as possible. The health history of adopted​ children, spouses, and other individuals living with the client would be reviewed​ because, although they are not related to the​ client, the​ client's concerns may have an environmental basis.

The nurse is assessing an older adult client and understands that certain medications can affect the musculoskeletal system of an older adult. What types of medications would compromise bone strength in an older ​adult? Select all that apply. Calcium Thyroid hormones Vitamin supplements Glucocorticoids Anticonvulsants

Thyroid hormones Glucocorticoids Anticonvulsants Bone strength in older adults can be compromised by​ anticonvulsants, glucocorticoids, and thyroid hormones. Calcium is found in bones and helps to strengthen them.​ Vitamins, including​ D3, promote bone health and would not compromise bone strength. Older adults would be assessed for bone health by determining what medications they take that might compromise bone strength.

Identify the population group based on the musculoskeletal considerations related to them. ​Instructions: Use the dropdown menus in the left column to select the population group for each consideration in the right column. Population Group Consideration Older Adults Pregnant Females Infants and Children Tibial torsion and metatarsal adductus Waddling gait Scoliosis Bone softening due to estrogen and progesterone Flat feet Kyphosis Bowlegs Bone changes due to decreased calcium absorption Genu valgum Center of gravity​ shifts, causing pressure on lower spine Height decreases Broken clavicles Degeneration of joints Congenital hip dislocation Bone strength compromised by medications Decreased flexibility

Tibial torsion and metatarsal adductus- infants and children Waddling gait- pregnant females Scoliosis- infants and children Bone softening due to estrogen and progesterone- pregnant females Flat feet- infants and children Kyphosis- older adults Bowlegs- infants and children Bone changes due to decreased calcium absorption- older adults Genu valgum- infants and chilren Center of gravity​ shifts, causing pressure on lower spine- pregnant females Height decreases- older adults Broken clavicles- infants and children Degeneration of joints- older adults Congenital hip dislocation- infants and children Bone strength compromised by medications- older adults Decreased flexibility- older adults

For what purpose would the nurse ask an adult client to raise the arms over the head during inspection of the​ breasts? To assess for changes or variations in breast contour or symmetry To elicit a pain response To test for agility To look for changes in color of the breasts

To assess for changes or variations in breast contour or symmetry Inspection of the breasts with the arms over the head allows for better viewing of the breasts and axillae and the position of the muscles that support the breasts for changes in contour or symmetry and for abnormal lumps or masses. The client​'s arms do not need to be above the head to inspect for color changes in the breasts. The purpose of any inspection is to observe and not to elicit a pain response or test for agility.

The nurse is conducting a nutritional assessment of an adult client. What risk factors would the nurse consider in the nutritional assessment of this ​client? Select all that apply. Undernutrition Overnutrition Obesity Growth retardation Heart murmurs

Undernutrition Overnutrition Obesity Adults are at risk for overnutrition and​ undernutrition, and obesity is a risk for most adults. Growth retardation is not a risk for adults. Heart murmurs are not the focus of nutritional assessments.

The nurse is interviewing and educating a client about anatomy and knows that the ribs and muscles surround the​ thorax, or the chest. What explanation by the nurse would describe the primary purpose of the ribs and muscles in the​ chest? To push the lungs during deflation To aid in exhalation To aid in inspiration To protect the lungs from external injury

To protect the lungs from external injury The main job of the ribs and muscles of the​ chest, or​ thorax, is protecting the more fragile lungs and heart from injury during daily activity. Each set of muscles assists with​ respiration, but the main job is to protect the lungs from​ puncture, bruising, and injury.

The nurse is assessing a child who is having difficulty with speech and learning to talk. The pediatrician diagnosed the child with ankyloglossia. What describes this abnormal​ condition? Tonsillitis Black hairy tongue Gingival hyperplasia Tongue attached to the floor of the mouth

Tongue attached to the floor of the mouth Ankyloglossia is caused by the frenulum being short and attached to the floor of the mouth. Black hairy tongue is due to the inhibition of normal bacteria and an overgrowth of fungus in the​ mouth; it does not affect speech. Gingival hyperplasia is the enlargement of gums during pregnancy. Tonsillitis is the inflammation of the tonsils and would not be the cause of a child not learning to talk.

Mr. Gomez is a​ 22-year-old man who came into the clinic to be assessed for a possible thyroid problem. The thyroid lies over what​ structure? Isthmus of thyroid Carotid artery Trachea Hyoid bone

Trachea The thyroid is located in the anterior portion of the neck and lies over the trachea. The hyoid bone is suspended in the neck above the larynx. The isthmus of thyroid connects the right and left lobes of the thyroid gland. The carotid arteries are in the neck and are palpated in the groove between the trachea and the sternocleidomastoid muscle below the angle of the jaw.

Which of the following muscles of respiration are considered accessory​ muscles? Select all that apply. Sternohyoid Trapezius Rectus femoris Scalene Sternocleidomastoid

Trapezius Scalene Sternocleidomastoid The​ scalene, sternocleidomastoid, and trapezius muscles are accessory muscles that play a major role in​ respiration, especially during periods of increased oxygen demand and with certain pathological conditions. The rectus femoris is a muscle in the​ thigh, and the sternohyoid is a muscle in the neck. Next Question

The nurse is planning to assess the subcutaneous fat of an adult client in the clinic. What is the best nutritional assessment tool for the nurse to use for this​ client? Midarm muscle circumference ​Self-reporting Waist circumference Triceps skinfold​ (TSF) test

Triceps skinfold​ (TSF) test The triceps skinfold​ (TSF) is most often used to estimate subcutaneous fat because of easy access to this measurement.​ Self-monitoring is viewed as unreliable and would not be the best choice for assessing subcutaneous fat in this client. Midarm muscle circumference requires the use of a general formula to correct for underlying bone and is used less commonly. Waist circumference is used to assess for abdominal fat​ deposition, which is an independent risk factor for cardiovascular disease.

Ms. Rodrigues is a​ 26-year-old woman​ who, during the course of your​ examination, has presented with a symptom that indicates compulsive behavior. What condition describes a compulsive​ behavior? Unkempt appearance Calluses Paronychia Trichotillomania

Trichotillomania Trichotillomania is an​ obsessive-compulsive disorder where the person pulls out​ hairs, including eyebrows and eyelashes. Paronychia is an infection of the skin and would be found in someone who works in an environment where the hands are immersed in water for long periods of time. Calluses form on body parts that are regularly exposed to​ pressure, weight​ bearing, or friction and are generally found on the​ fingers, toes,​ palms, and soles of the feet. A client with an unkempt appearance may indicate a demanding​ job, illness, or depression.

An adult client who has been visiting outside the United States mentions coughing up​ rust-colored sputum for several days. The nurse knows the​ rust-colored sputum is associated with what respiratory disease that will require further​ intervention? Head colds Bacterial infections Pulmonary edema Tuberculosis

Tuberculosis ​Rust-colored sputum is associated with tuberculosis or pneumonia. The fact that the client was out of the country and does not have any other complaints would lead the nurse to suspect tuberculosis. Pink frothy sputum is associated with pulmonary​ edema, white is associated with viral​ infections, and green or yellow is usually indicative of a bacterial infection.

The nurse is assessing an older female client with rheumatoid arthritis. What deformities occur with chronic rheumatoid arthritis and interfere with client ​safety? Select all that apply. Ulnar deviation Olecranon bursitis Carpal tunnel Boutonniere deformities ​Swan-neck deformity

Ulnar deviation Olecranon bursitis Boutonniere deformities ​Swan-neck deformity Ulnar deviation results from the chronic inflammation of the joints in the hand and wrist of clients with rheumatoid arthritis. The proximal interphalangeal joints are​ hyperextended, and the distal joints are fixed in​ swan-neck deformities. Boutonniere deformities are caused by the proximal interphalangeal joints being flexed and out of alignment. Olecranon bursitis is inflammation of the bursae in the elbow from trauma or rheumatoid arthritis. Carpal tunnel is inflammation and compression of the medial nerve in the wrist from repetitive motion and is not necessarily related to rheumatoid arthritis.

Identify the interview phase based on the description. ​Instructions: Use the dropdown menus in the left column to select the phase of interview for each description in the right column. Interview Phase Description Preinteraction Focused Interview Initial Interview Update and identify new diagnostic cues Introduce yourself and explain reason for questioning Choose comfortable setting for interview Gather missing information about a health concern Gather data Collect data from medical record Guide direction of physical assessment Use health history form to collect data Previous health screenings Identify or validate probable nursing diagnoses Previous health risk appraisals Clarify previously obtained assessment data Assessments of previous health care professionals

Update and identify new diagnostic cues- focused interview Introduce yourself and explain reason for questioning- initial interiew Choose comfortable setting for interview- preinteraction Gather missing information about a health concern- focused interview Gather data- initial interview Collect data from medical record- preinteraction Guide direction of physical assessment- focused interview Use health history form to collect data- initial interview Previous health screenings- preinteraction Identify or validate probable nursing diagnoses- focused interview Previous health risk appraisals- preinteraction Clarify previously obtained assessment data- focused interview Assessments of previous health care professionals- preinteraction

In which quadrant of the breast is the tail of Spence​ located? Upper outer Lower outer Lower inner Upper inner

Upper outer The tail of Spence is an area of​ cone-shaped breast tissue that projects into the axillae and is considered part of the upper outer quadrant of the breast.

What male anatomic structures serve as both genitourinary and sexual ​organs? Select all that apply. Urethra Kidneys Renal arteries Penis Ureters

Urethra Penis The penis serves as both the genitourinary and reproductive tract for the male. The urethra transports both urine and semen out of the body and thus serves as both a genitourinary and a reproductive organ. The kidneys are located in the abdomen on the right and left of the spinal column and serve to filter wastes from the blood. The ureters are muscular tubes that connect the kidneys to the bladder and carry urine from one point to another. The renal arteries supply a large portion of the blood to the kidneys in the genitourinary system.

The nurse is caring for a client with a neurologic disease. What concern is not considered a psychosocial problem that is caused by change to the neurological​ system? Inability to concentrate Urinary incontinence Social isolation Low​ self-esteem

Urinary incontinence Urinary incontinence is a physical problem that can be a manifestation of a neurologic​ condition, not a psychosocial one. Low​ self-esteem, social​ isolation, and inability to concentrate are all psychosocial changes that can occur with neurologic problems.

Identify the general guidelines to follow to ensure accurate documentation and pitfalls to avoid while documenting a​ client's record. ​Instructions: Use the dropdown menus in the left column to select whether the guideline given in the right column is considered as an acceptable documentation or a pitfall to avoid. Category Guideline Pitfalls of Charting to Avoid Acceptable Documentation Use late entry if unable to chart timely Leave blank space for later charting Include teaching Use vague terms Chart changes in condition with​ follow-up actions Chart in advance of events Record​ client's words with quotes Alter record only if requested by a supervisor

Use late entry if unable to chart timely- Acceptable Documentation Leave blank space for later charting- Pitfalls of Charting to Avoid Include teaching- Acceptable Documentation Use vague terms- Pitfalls of Charting to Avoid Chart changes in condition with​ follow-up actions- Acceptable Documentation Chart in advance of events- Pitfalls of Charting to Avoid Record​ client's words with quotes- Acceptable Documentation Alter record only if requested by a supervisor- Pitfalls of Charting to Avoid

What is the best way to organize family history data when documenting in a client​'s medical​ record? Use of family tree Use of timeline Use of an ancestral chart Use of genogram

Use of genogram The best way to organize family history data in the client​'s medical record is by using a​ genogram, a pictorial representation of family relationships and medical history. A timeline reflects events over​ time, a family tree is a generational​ chart, and an ancestral chart records descendantslong dashnone of these choices include a family​'s medical history.

When palpating the​ breasts, what are the appropriate techniques to ​use? Select all that apply. Palpate the axillae. Use the pads of three fingers in a circular motion. Have the client lie flat. Have the client sit comfortably. Palpate skin texture and contour.

Use the pads of three fingers in a circular motion. Have the client lie flat. Palpate skin texture and contour. Lying flat allows the breast to​ relax, making palpation easier to perform on all quadrants of the breast. Performing palpation with the pads of the three fingers in a circular motion prevents pain and injury to the client during the exam and allows the best opportunity to feel abnormal lumps. The skin should feel smooth and the contour should be uninterrupted. The axillae are palpated separately from the breasts. The client would be in a sitting position as the nurse palpates the axillae while supporting the client​'s arm with the nondominant arm.

The nurse is preparing to perform a physical assessment on a​ 33-year-old client. Which action by the nurse during percussion indicates a need for the nurse​'s further teaching regarding ​percussion? Select all that apply. Performs blunt percussion when assessing for pain in the abdomen Uses indirect percussion when assessing for tenderness of the liver Taps the body with the fingertips of the dominant hand over the client​'s sinus area Uses direct percussion when assessing the client​'s gallbladder Places the palm of the nondominant hand flat against the abdomen and strikes it with the dominant hand using a closed fist to assess tenderness of the abdomen

Uses indirect percussion when assessing for tenderness of the liver Uses direct percussion when assessing the client​'s gallbladder Blunt​ percussion, not direct or indirect​ percussion, is used in assessing pain and tenderness of the​ gallbladder, liver, and kidneys. Tapping the body with the fingertips of the dominant hand over the client​'s sinus area is an example of direct​ percussion, which is used to examine the thorax in infants and the sinuses in adults. Placing the palm of the nondominant hand flat against the abdomen and striking it with the dominant hand using a closed fist to assess tenderness of the abdomen is an example of blunt percussion.

Which action indicates a need for the further teaching of the nurse regarding​ percussion? Uses two strikes then repositions before striking again Releases plexor finger immediately after the delivery of two sharp strikes Uses the pad of the striking finger when delivering the blow Uses the motion of the​ wrist, not the forearm or plexor finger

Uses the pad of the striking finger when delivering the blow The nurse would use the tip of the striking​ finger, not the finger​ pad, when delivering the blow. Releasing the plexor finger immediately after the delivery of two sharp strikes will help ensure the successful technique of percussion. Use two strikes then​ reposition; delivery of more than two​ rapid, consecutive strikes creates the open double quote"woodpecker syndromeclose double quote" and sounds are muffled. Ensure that the motion is from the​ wrist, not the forearm or plexor finger.

A client expresses anxiety about a surgical procedure. Which are the most appropriate ways for the nurse to handle this situation to insure effective ​communication? Select all that apply. Use​ open-ended questions. Offer reassurance. Use silence. Use reflection. Probe and reject the​ client's comment.

Use​ open-ended questions. Use reflection. Asking​ open-ended questions that cannot be answered with one word is particularly useful when helping clients to explore their feelings. Reflecting involves the nurse taking in the​ client's feelings from verbal and nonverbal communication and directing them back to the client. Reflecting helps the client to recognize and accept​ his/her feelings or concerns. Probing or rejecting client comments serves as a barrier to communication and makes the client feel as though the nurse is prying and being unsupportive. The use of silence can be helpful once the client begins freely sharing​ ideas, but it will not be useful in getting the conversation started. In order to help​ client's share their​ feelings, the nurse is best served by​ open-ended questions. Unwarranted reassurance can damage the nursedashclient relationship because there is no way for the nurse to know the outcome of surgery.​ Further, the use of clich​és or comforting statements acts as a barrier to​ clients' expression of their feelings.

What female anatomical structures directly relate to ​reproduction? Select all that apply. Uterus Ovaries Labia Vagina Fallopian tubes

Uterus Ovaries Vagina Fallopian tubes The ovaries produce the​ ovum, or​ egg, that eventually becomes a fetus when fertilized. The vagina is the muscular tube that connects to the cervix and uterus. It functions as the birth​ canal, the exit for menstrual​ flow, and the area of sexual intercourse. The fallopian tubes are the pathways for the ovum or eggs to travel to the uterus. The uterus houses the fertilized egg that forms the fetus and nourishes the fetus throughout its development. The labia are the fleshy areas outside the vagina that protect the vagina and the​ urethra; they do not function in reproduction.

A client with a score of 3 on the verbal response section of the Glasgow Coma Scale would display what​ response? Utters inappropriate words Confused and disoriented Oriented and converses normally Makes no sounds

Utters inappropriate words A client who utters inappropriate words would have a score of 3 on the verbal response section of the Glasgow Coma scale. A client who is oriented and converses normally would have a score of 5. A client who makes no sounds would have a score of 1. A client who is confused and disoriented would have a score of 4.

Ms. Lopez is a​ 22-year-old woman who has come to the clinic with a sore throat. Which of the following is not a normal finding when inspecting the​ throat? Uvula is pink. Uvula is swollen. Soft palate rises. Tonsils are without inflammation.

Uvula is swollen. When inspecting a​ client's throat, a tongue depressor is used to clearly visualize the throat. A swollen uvula is an indication of inflammation and an abnormal finding. The soft palate rising is a normal response to depressing the middle of the arched tongue with the tongue depressor. The​ uvula, tonsils, and pharynx being pink and without swelling or inflammation are normal findings.

Mrs. Levy is a​ 60-year-old woman who came into the clinic because she noticed a protrusion on her neck. Upon​ inspection, you ascertain that her right carotid artery is prominent. What might this​ indicate? Infection Arthritis Muscle spasms Vascular disorder

Vascular disorder Distention or prominence of the carotid artery may indicate that the client has a vascular disorder. Arthritis would be indicated by excessive rigidity of the neck or pain or limitation of the motion of the neck. Swelling of lymph​ nodes, not prominence of the carotid​ artery, would indicate infection. Muscle spasms would be noted by pain or limitation of the motion of the neck and an inability to hold the neck erect.

What observations would be included during an inspection of the breasts for a female ​client? Select all that apply. Venous pattern Bra cup size Symmetry and color Areolae and nipples Moles or markings

Venous pattern Symmetry and color Areolae and nipples Moles or markings A slight difference in the size of the breasts is​ normal, and the color of the breasts should be consistent with the rest of the body. The venous pattern should be bilateral and may be more obvious during pregnancy. Moles and markings that remain unchanged and are nontender are not worrisome. The areolae and nipples should be the same color and shape. The actual cup size of a client is not important unless it has changed or the breasts are not symmetrical.

Which of the following landmarks is visible and easily palpable and is the largest spinous​ process? Vertebral prominence Posterior triangle Hyoid bone Atlas

Vertebral prominence The vertebral​ prominence, or the 7th cervical​ vertebra, is the largest spinous process and a landmark during assessment of the neck. The​ atlas, the first cervical vertebra ​(C1​), carries the skull. The posterior triangle is formed by the trapezius​ muscle, the sternocleidomastoid​ muscle, and the clavicle. The hyoid bone is palpable just inferior to the angle of the jaw and serves as a landmark for assessing structures of the​ neck, especially the trachea and thyroid bone.

Mr. Heida is admitted to the surgical unit after a hip replacement. You review the notes from the preoperative nurse in order to provide excellent care to Mr. Heida. The notes​ read: "VS WNL during preop care. NPO since​ midnight." What is the best way to interpret the​ documentation? Vital signs within natural limits during preoperative care. Client has had a nasogastric tube since midnight. Vital signs without normal limitations during preoperative care. Client has had no food since midnight. Various symptoms without noted limitation during preoperative care. No urine output since midnight. Vital signs within normal limits during preoperative care. Client has had nothing by mouth since midnight.

Vital signs within normal limits during preoperative care. Client has had nothing by mouth since midnight. The notes should be interpreted as​ "vital signs within normal limits during preoperative care. Client has had nothing by mouth since​ midnight." VS is the abbreviation for​ "vital signs," WNL is the abbreviation for​ "within normal​ limits," Preop is the abbreviation for​ "preoperative," NPO is the abbreviation for​ "nothing by​ mouth."

The nurse is caring for a​ 44-year-old client who reports very dry hair and scalp. What would the nurse recognize as possible causes of dry hair and ​scalp? Select all that apply. Vitamin B deficiency Imbalance in adrenal hormones Scalp trauma due to chemicals Excessive washing of hair Pregnancy

Vitamin B deficiency Excessive washing of hair Dry hair and dry scalp or dandruff can be caused by vitamin B​ deficiency, excessive​ washing, or washing with harsh shampoos that can remove protective oils and dry the hair. Loss of​ hair, not​ dryness, can be due to scalp trauma from​ chemicals, pregnancy, or imbalances in adrenal hormones.

The nurse is caring for a​ 45-year-old male African American client who weighs 110 kg​ (242 lb). The nurse is preparing to assess the client​'s blood pressure. Which would the nurse recognize as factors that will affect this client​'s blood ​pressure? Select all that apply. Weight Hormones Gender Age Ethnicity

Weight Gender Ethnicity After​ puberty, females demonstrate a lower blood pressure than males.​ In the United​ States, Caucasian clients will usually have a lower blood pressure compared to African American clients. Obesity is a risk factor for hypertension.​ Hormones will affect the client​'s temperature but should not affect the client​'s blood pressure. By age 18 the blood pressure is stabilized. Although age can affect the blood pressure of the young and the​ elderly, this client​'s blood pressure should not be affected by his age.

What are the characteristics of​ fibroadenoma? Select all that apply. ​Well-defined, round, firm tumor Associated with malignant neoplasms Excision is usual treatment Usually occurs as a single tumor Asymptomatic

Well-defined, round, firm tumor Usually occurs as a single tumor Asymptomatic Fibroadenomas are characterized by​ well-defined, round, firm tumors that are approximately 1 to 5 cm in size. Tumors are usually a single occurrence near the nipple or upper outer quadrant. Careful surveillance is the usual treatment unless findings are inconclusive. A relationship has not been established between fibroadenomas and malignant neoplasms.

When performing an assessment of the musculoskeletal system on a​ client, which questions are most​ relevant? Select all that apply. What time of day does your joint pain​ occur? Are you having any problems with your​ bones? Do you have joint​ pain? Is your joint pain aggravated by​ movement? Do you have intestinal​ pain?

What time of day does your joint pain​ occur? Are you having any problems with your​ bones? Do you have joint​ pain? Is your joint pain aggravated by​ movement? Joint pain is one of the most common concerns during a musculoskeletal assessment. Most joint​ pain, except that associated with rheumatoid arthritis​ (RA), is increased by movement. Time of day can indicate the source of the​ pain; RA is worse in the morning and improves during the day. Bone pain or limitations in movement may indicate joint injury. Intestinal pain would not be indicative of a musculoskeletal problem.

The nurse is caring for an​ 8-year-old child in the emergency room who was stung by several bees in the lower leg. What assessment finding would the nurse expect to​ find? Vesicles Pustules Papules Wheals

Wheals Wheals are seen with insect bites and hives. Papules are elevated​ moles, warts, and lichen planus. Vesicles are​ fluid-filled, round or​ oval-shaped palpable masses seen with conditions like herpes simplex or herpes zoster. Pustules are seen in conditions like acne and impetigo.

Identify the abbreviation for the given expanded form. ​Instructions: Use the dropdown menus in the left column to select the abbreviation for the expanded form in the right column. Abbreviation Expanded Form Ad lib PRN qid ADL When necessary Four times a day Activities of daily living As desired

When necessary- PRN Four times a day- qid Activities of daily living- ADL As desired- ad lib

The nurse is caring for a client on a medicaldash-surgical floor. Which behaviors would the nurse recognize as signs of chronic​ pain? Anxious and restless Crying and tense Withdrawn and depressed Increased blood pressure and heart rate

Withdrawn and depressed The client with chronic pain may be withdrawn and depressed. The parasympathetic nervous system response in chronic pain is normal vital​ signs, skin that is dry and​ warm, and normal or dilated pupils. The client experiencing acute​ pain, not chronic​ pain, may be anxious and restless. The client with chronic pain often lacks pain behaviors like crying and tenseness.

Mrs. Henry is a​ 67-year-old client who came to the clinic for a routine examination. Which action by​ you, a novice​ nurse, during auscultation requires further instruction by the charge​ nurse? You perform auscultation with the stethoscope on top of the​ client's thin gown to prevent shivering of client. You place the stethoscope against the​ client's abdomen underneath the​ client's gown. You document the​ client's quality of speech and respirations before auscultating lung sounds. You document the duration and pitch of auscultated heart tones.

You perform auscultation with the stethoscope on top of the​ client's thin gown to prevent shivering of client. The stethoscope should be placed against bare skin rather than on top of​ clothes, though the client should be kept warm during assessment. Placing the stethoscope underneath the gown is a good way to keep the client warm while still being able to obtain a complete assessment. You should document all sounds​ noted; this includes audible speech and respiratory sounds as well as auscultated heart tones.

The nurse has completed the skin assessment for a​ 33-year-old client. The nurse found several lesions that appeared in a linear manner along the client​'s lower back. What type of lesion would the nurse​ report? Annular lesion Zosteriform lesion Target lesions Discrete lesions

Zosteriform lesion Zosteriform lesions are arranged in a linear manner along a nerve route. Discrete lesions are separate and identifiable as a single or discrete lesion. Annular lesions are circular in shape. Target lesions are centric circles of color.

Arrange the assessment techniques of a physical assessment of all body​ systems, except the​ abdomen, in the correct sequence.

inspection palpation percussion auscultation

The nurse is preparing to complete a physical assessment on the client. In which order would the nurse perform a physical​ examination? ​Instructions: Click on the down arrow for each response in the right column and select the correct choice from the list. Order Responses Percussion Auscultation Inspection Palpation

inspection palpation percussion auscultation

The nurse is preparing to complete a health assessment interview on a client in the healthcare​ provider's office. In which order would the nurse complete the health​ history? ​Instructions: Click on the dropdown arrow for each phase of the interview in the right column and select the correct choice from the list. Order Phases of Interview 1st ▼ Initial Interview Focused Interview Preinteraction 2nd ▼ Initial Interview Focused Interview Preinteraction 3rd ▼ Preinteraction Initial Interview Focused Interview

preinteraction initial interview focused interview

What movement would the nurse be assessing when asking the adult client to touch the thumb of one hand to the fingertip of each finger on the same​ hand? Supination Opposition Adduction Abduction

opposition Opposition refers to the ability of the thumb on one hand to touch the fingertips of the same hand. It is used to assess​ coordination, mobility, and nerve and spinal control. Abduction refers to moving away from the midline. Adduction refers to moving toward the midline. Supination refers to the movement of the radius around the ulna with palms anterior. Understanding the movement of the musculoskeletal system is essential to prevent further injury and to determine abnormal movement.

A negative response to what question indicates that the client could be at risk for urinary tract​ infections? ​"Do you have difficulty starting the flow of​ urine?" ​"Are you able to empty your bladder completely with each​ urination?" ​"Do you get up during the night to​ urinate?" ​"Has your urination pattern changed​ recently?"

​"Are you able to empty your bladder completely with each​ urination?" Retaining urine due to the inability to completely empty the bladder with each urination can cause urinary tract infections. Changes in urination pattern may indicate heart​ disease, kidney​ disease, or nutritional problems. Whether or not a client gets up during the night to urinate is influenced by heart or kidney disease or the amount and timing of the ingestion of fluids or diuretics. Difficulty starting the flow of urine indicates that the client may have a mass or tumor.

What question related to the urinary system would be most appropriate for the nurse to ask a female client after making the following ​observations? ​"Do you get up during the night to​ urinate?" ​"Are you experiencing any pain or discomfort in your​ abdomen?" ​"When was your last menstrual​ period?" ​"How often do you​ urinate?"

​"Are you experiencing any pain or discomfort in your​ abdomen?" A temperature of 100.4​°F may indicate an infection. Since there are no outward or obvious signs of ​illness,​ the nurse would ask about pain or discomfort in the abdomen to allow the client to tell the nurse about problems she is experiencing as part of the initial​ questions, as urinary infections can go undetected. Asking about the client​'s last menstrual period may be important at some point but is not an initial interview question. Asking the client if she gets up during the night to urinate could be included as a​ follow-up question to gain more​ information, but asking about pain or discomfort first would be appropriate. Frequency of urination is influenced by several​ factors, but this information would not be as important as whether the client has pain or discomfort with urination.

What is the first question that would be important for the nurse to ask an adult presenting to the emergency room triage with complaints of not feeling​ well? ​"Are you having chest​ pain?" ​"What medications are you taking to relieve your​ symptoms?" ​"When did you eat​ last?" ​"What are your​ allergies?"

​"Are you having chest​ pain?" An adult who complains of not feeling well and is ill enough to come to the emergency room warrants a complete assessment to rule out a cardiac problem. Although asking about​ meals, medications, and allergies are part of an assessment in​ general, adults are at higher risk for heart attack and heart​ disease, so it must be determined if the illness is stemming from chest pain or other indications of serious cardiac illness.

What questions would the nurse ask when gathering subjective data about the urinary health of a male ​client? Select all that apply. ​"Do you get up to urinate at​ night?" ​"Have you had the​ mumps?" ​"Do you have difficulty starting your​ stream?" ​"How often do you urinate each​ day?" ​"Do you have difficulty achieving an​ erection?"

​"Do you get up to urinate at​ night?" ​"Do you have difficulty starting your​ stream?" ​"How often do you urinate each​ day?" Many things affect frequency of urination and male urinary health. The need to urinate frequently and at night can indicate kidney or heart disease or the amount of fluid intake a client had during the day. Difficulty starting the urinary stream can be caused by an obstruction or tumor. Having the mumps as a child can cause male​ sterility, but this is related to reproduction and not urinary health. Physiological factors and state of mind can affect the ability to have an​ erection; this is related to reproductive​ health, not urinary health.

The nurse suspects that the adult client being assessed is having difficulty with activities of daily living and may not be safe at home. What interview questions would help the nurse perform a focused assessment of the musculoskeletal ​system? Select all that apply. ​"Are you having difficulty with movement or​ balance?" ​"What insurance do you currently have for joint​ injuries?" ​"Are you able to brush your teeth and​ shower?" ​"Do you live alone or is there family help available to assist you with​ activities?" ​"Does your work require repetitive movements or heavy​ lifting?"

​"Are you having difficulty with movement or​ balance?" ​"Are you able to brush your teeth and​ shower?" ​"Do you live alone or is there family help available to assist you with​ activities?" ​"Does your work require repetitive movements or heavy​ lifting?" It is important to ask the client about the type of pain​ he/she is experiencing and if it interferes with activities of daily living. The nurse needs to know if the pain or condition causes the client to fall or be off balance so that it can be determined if it is safe for the client to be at home. It is also important to know if the client can do activities of daily living such as brushing teeth and toileting. If a client cannot do these​ things, it may mean the joint is​ injured, and the client would need further assistance at home or a consult with physical therapy. The type of work a client does is important and can be directly related to the pain or injury. Insurance coverage is not pertinent during the assessment of the musculoskeletal status of a client.

Identify whether the questions asked during an interview helps to assess the internal or external environmental factors that contribute to illness and disease of the female genitourinary system. ​Instructions: Use the dropdown menus in the left column to select the type of environmental factor for each question in the right column. Environmental Factor Question External Environment Internal Environment ​"Are you in a sexual​ relationship? If​ so, are you able to talk about your sexual​ needs?" ​"Have you traveled outside the United States​ recently?" Was diethylstilbestrol​ (DES) prescribed to your mother to prevent pregnancy​ problems?" ​"Does your family support your​ relationship?" ​"Are you taking dietary​ supplements, vitamins, or protein​ powders?" ​"Do you work with toxic​ chemicals?" ​"Are you currently taking prescribed or​ over-the-counter medications?" ​"Are you exposed to toxic chemicals in the environment where you​ live?"

​"Are you in a sexual​ relationship? If​ so, are you able to talk about your sexual​ needs?" external ​"Have you traveled outside the United States​ recently?" external Was diethylstilbestrol​ (DES) prescribed to your mother to prevent pregnancy​ problems?" interrnal ​"Does your family support your​ relationship?" external ​"Are you taking dietary​ supplements, vitamins, or protein​ powders?" internal ​"Do you work with toxic​ chemicals?" external ​"Are you currently taking prescribed or​ over-the-counter medications?" internal ​"Are you exposed to toxic chemicals in the environment where you​ live?" external

The nurse is teaching a​ 65-year-old client about the normal aging process. What statement by the client indicates a need for further​ teaching? ​"I may notice a decrease in my sense of smell and taste as I​ age." ​"My muscle mass will decrease as I​ age." ​"I might start to have a shuffling​ gait." ​"As I age my coordination will​ improve."

​"As I age my coordination will​ improve." Coordination decreases with age. This statement indicates a need for further instruction of the client on the changes with normal aging. A decrease in the senses and muscle​ mass, and a​ shorter, shuffling or uncertain gait are all seen with aging.

After you welcome and help Mrs. Nelson settle into her new room on your​ unit, she enjoys visiting with friends then takes an​ hour-long nap. Upon awakening with a startled​ feeling, unsure where she​ is, you take her vital signs and discover a blood pressure of​ 146/94. When documenting this​ information, which describes your nursing​ assessment? ​"Client's blood pressure within expected limits. Will reevaluate in 4​ hours." ​"Client has symptoms of orthostatic​ hypotension; educated to call for assistance with​ ambulating." ​"Client is currently hypertensive. Will reassess blood pressure in 15​ minutes." ​"Client has signs of hypotension. Will educate client to increase fluid intake and recheck in one​ hour."

​"Client is currently hypertensive. Will reassess blood pressure in 15​ minutes." This blood pressure reading falls in the hypertensive range for an adult and warrants a​ 15-minute recheck. Awakening with a​ startle, anxiety, or recent physical activity can elevate the blood pressure. Hypotension is a​ lower-than-expected blood​ pressure, which can be attributed to dehydration and corrected with fluids. Orthostatic hypotension is a blood pressure drop of more than 20 mmHg as​ one's position changes from lying supine to standing up. This can cause​ lightheadedness/dizziness and can lead to​ falls/injury.

The nurse is teaching a new nurse about documentation. Which statement by the new nurse indicates understanding of the purpose of a​ client's ​records? Select all that apply. ​"Client records are used to assist in planning the client​'s ​care." ​"Client records are used to provide a data source for​ research." ​"Client records improve the quality of care​ provided." ​"Client records are necessary for ensuring financial​ reimbursement." ​"Client records help reduce the risk of​ lawsuits."

​"Client records are used to assist in planning the client​'s ​care." ​"Client records are used to provide a data source for​ research." ​"Client records are necessary for ensuring financial​ reimbursement." Client records can serve as excellent sources of data for conducting research. The medical record plays an important role in the planning of the client​'s care. Documentation helps a facility receive reimbursement from the federal government and private insurance companies for care delivered. The purpose of medical records is not to reduce the risk of​ lawsuits, although accurate documentation can help protect the health care worker during a lawsuit. Although the medical record may be used to evaluate the quality of​ care, it does not improve the quality of care delivered.

What statement by the nurse to the client with knee pain indicates an understanding of the anatomy of the knee​ joint? ​"The knee joint is separated by a joint cavity filled with synovial​ fluid." ​"The knee is a fibrous​ joint." ​"The knee is a hinged joint with bone on​ bone." ​"The knee has​ cartilage, making it a cartilaginous​ joint."

​"The knee joint is separated by a joint cavity filled with synovial​ fluid." The knee is both a hinged joint and a joint separated by a​ fluid-filled cavity, making it a synovial joint. The knee also has cartilage and tendons that help to connect it to the upper and lower bones in the leg.​ However, the joint is separated by a​ fluid-filled cavity, not cartilage. A joint connected by fibrous tissue is a fibrous​ joint, for​ example, the bones in the skull.

The nurse is completing a health history on a​ client's cardiac history. Which question by the nurse is an example of the technique known as​ focusing? ​"How do you feel about your overall​ health?" ​"Describe how you take your blood pressure at​ home." ​"What did you mean when you said you​ don't understand why you have to take your blood​ pressure?" ​"Sounds like you are monitoring your blood pressure every​ day, is that​ right?"

​"Describe how you take your blood pressure at​ home." ​"Describe how you take your blood pressure at​ home" is an example of focusing. Focusing is a technique used to help the client zero in on a topic. Saying to the​ client, "Sounds like you are monitoring your blood pressure every​ day, is that​ right?" is an example of paraphrasing. Paraphrasing is used to restate what the nurse thinks the client has said and is used to gain clarity.​ "What did you mean when you said you​ don't understand?" is an example of leading. Leading is used to help the nurse glean specific information from the client.​ Open-ended questions are asked to explore the​ client's thoughts and feelings on topics.​ "How do you feel about your overall​ health?" is an example of an​ open-ended question

You educate your​ client, Mr.​ Nyambi, that blood​ pressure, peripheral vascular​ resistance, and blood flow all affect arterial circulation. Mr. Nyambi​ states, "I do not understand the difference between systolic and diastolic​ pressure." You should include which statement in the education of the​ client? ​"Systolic pressure occurs when the right atrium​ contracts." ​"Diastolic pressure occurs during the​ heart's contraction​ period." ​"Diastolic pressure occurs when the heart is at rest​." ​ "Systolic pressure happens when the heart​ relaxes."

​"Diastolic pressure occurs when the heart is at rest​." Diastolic pressure is defined as the amount of force exerted by blood against the arterial walls when the heart muscle is at rest. Systolic pressure is defined as the amount of force exerted by blood against the arterial walls during left ventricular muscle contraction. Although important in delivering blood into the right​ ventricle, the right atrial muscle contraction does not play a primary role in systolic pressure. Systolic pressure results from left ventricular contraction with subsequent propulsion of blood throughout important body systems. Next Question

The nurse is percussing the chest of an adult client and observes a lateral scar on the left posterior chest indicating a past surgical procedure. What questions would be appropriate for the nurse to ask to get a complete medical history during this ​exam? Select all that apply. ​"Did the surgery involve the lungs or the​ heart?" ​"When did the surgery​ occur?" ​"Were you placed on a respirator after the surgical​ procedure?" ​"What type of surgery did you have to your​ chest?" ​"Was the procedure inpatient or​ outpatient?"

​"Did the surgery involve the lungs or the​ heart?" ​"When did the surgery​ occur?" ​"Were you placed on a respirator after the surgical​ procedure?" ​"What type of surgery did you have to your​ chest?" It would be important to know what procedure occurred and​ when, and whether it was performed to address a​ lung/thorax issue or a cardiac issue. The result of the surgery and the diagnosis would also be important. The necessity for a respirator after surgery would be important to the current assessment of the chest and respiratory status of this client.

The nurse is assessing an adult client with a cough. A positive answer to what question about the cough would require further evaluation and questioning of this​ client? ​"Do you have clear drainage from the​ nose?" ​"Are you experiencing a dry​ cough?" ​"Do you cough up blood and how​ often?" ​"Do you cough up clear sputum and how​ often?"

​"Do you cough up blood and how​ often?" Many questions arise when asking a client about a​ cough, but the nurse would determine first and foremost whether the client is suffering from​ hemoptysis, or coughing up blood. This is a serious symptom and would help the nurse to determine the line of questioning to pursue. Whether the client has clear drainage from the nose or is coughing up clear sputum are the least worrisome factors to consider. The kind of cough​ (dry, hacking,​ moist, barky, or​ wheezy) would give the assessment a direction but is not as important as discovering if the cough produces bloody sputum.

The nurse is assessing the abdomen of an adult male client who has not been feeling well. What would be appropriate questions for the nurse to ask the ​client? Select all that apply. ​"Do you experience nausea after​ eating?" ​"Do you have pain in your abdomen and if​ so, when does it​ occur?" ​"Have you traveled out of the country in the last 3​ months?" ​"How many children do you​ have?" ​"Do you experience vomiting and if​ so, how​ often?"

​"Do you experience nausea after​ eating?" ​"Do you have pain in your abdomen and if​ so, when does it​ occur?" ​"Have you traveled out of the country in the last 3​ months?" ​"Do you experience vomiting and if​ so, how​ often?" Nausea is a symptom of a problem in the digestive system. Vomiting can indicate an abdominal problem due to illness or obstruction. Pain in the abdomen is not normal and can indicate a problem depending on when it occurs. Asking if a client traveled outside of the country can help determine if the client was exposed to parasites and​ bacteria, which can cause illness and abdominal problems. The number of children this client has is not relevant to his abdominal assessment but may be helpful when assessing a female client of childbearing age who has pelvic pain.

The nurse is assessing the abdomen of an older adult client. What questions would be appropriate when assessing the nutrition and digestion of an older ​adult? Select all that apply. ​"Do you feel bloated or uncomfortable after​ eating?" ​"What color is your​ stool?" ​"Do you have pain in your​ joints?" ​"When is the last time you had a bowel​ movement?" ​"How is your​ appetite?"

​"Do you feel bloated or uncomfortable after​ eating?" ​"What color is your​ stool?" ​"When is the last time you had a bowel​ movement?" ​"How is your​ appetite?" Assessing for symptoms of bloating in an adult client would give data that can help the nurse to determine the condition of the digestive system. It is important to assess bowel​ function, the color of​ stool, and the appetite of an older adult when assessing the abdomen and the digestive system. Questions about joint pain are not part of a focused abdominal assessment.

The charge nurse is assisting the student with a list of questions to ask an adult client when assessing the mouth and throat. What questions would be most helpful in doing this ​assessment? Select all that apply. ​"Do you have any pain in your​ teeth, gums, or​ jaw?" ​"Do you have any difficulty​ swallowing?" ​"Do you have any difficulty breathing through your mouth or​ nose?" ​"Do you have any difficulty with your bowel or​ bladder?" ​"Do you have pain in your​ joints?"

​"Do you have any pain in your​ teeth, gums, or​ jaw?" ​"Do you have any difficulty​ swallowing?" ​"Do you have any difficulty breathing through your mouth or​ nose?" Assessing the​ client's ability to swallow without choking is an important piece of information for an adult assessment. Determining if an adult has pain in the​ teeth, gums, or jaw can help to assess the client​'s overall dental condition and areas that are potential sources for infection. Asking about breathing issues opens the discussion about​ snoring, nasal​ stuffiness, or difficulty taking a​ breath, which could be related to other health concerns. Asking about joint pain may be part of an overall assessment but does not give any information about the mouth. Bowel and bladder issues are an important part of a physical assessment but are not relevant when assessing the mouth of a client.

The nurse is assessing a child who presents with a fever and dental caries. What question might help the nurse assess the cause of the fever in this​ child? ​"Do you have any​ gum, teeth, or mouth​ pain?" ​"When is the last time you had anything to​ eat?" ​"Can you stick out your​ tongue?" ​"How old are​ you?"

​"Do you have any​ gum, teeth, or mouth​ pain?" If the child has pain in the​ gums, teeth, or​ mouth, the fever may be the result of a tooth or mouth infection. It would be important for the health care provider to assess the condition of the mouth as part of the exam when looking for a source for the fever. Determining when the child last ate helps to assess the overall health of the client but may not be important when assessing the mouth as a source of fever. Sticking out the tongue as part of the exam is​ normal, but as a single source of information about the fever it would not be as important. Asking about the child​'s age and making conversation is an important​ rapport-building activity but does not help to assess the source of the fever.

The nurse is assessing an adult client for complaints of nasal stuffiness. What questions would be most helpful in assessing the nasal symptoms of the ​client? Select all that apply. ​"Do you use any recreational​ drugs, over-the-counter nasal​ sprays, or other nasal​ medications?" ​"Do you have pain in your​ neck?" ​"Do you have difficulty breathing out of your​ nose?" ​"Have you ever had a nose injury or had any recent trauma to the​ nose?" ​"Have you had any recent​ nosebleeds?"

​"Do you have difficulty breathing out of your​ nose?" ​"Have you ever had a nose injury or had any recent trauma to the​ nose?" ​"Have you had any recent​ nosebleeds?" Nasal stuffiness does not always interfere with​ breathing, so asking a specific question about difficulty breathing through the nose helps the client begin discussing symptoms. Determining if the client has had any nosebleeds helps to assess the overall condition of the nasal passages. Determining a recent injury or obstruction may help in assessing the current reason for nasal stuffiness. Determining if the client uses​ drugs, over-the-counter​ medications, or nasal sprays that may affect the condition of the nose is important when doing a complete assessment. Asking about pain in the​ nose, sinus​ area, ears, and throat are important initial​ questions, but pain in the neck is not an initial area of inquiry when assessing the nose.

The nurse is doing a musculoskeletal assessment on a​ 40-year-old adult client. A positive answer to what question would indicate the need for further evaluation and consultation with a​ specialist? ​"Is your pain more of a muscle​ pain?" ​"Do you have muscle​ weakness, numbness, or​ tingling?" ​"Do you have decreased mobility in a​ joint?" ​"Have you had ankle pain for 2 days or​ less?"

​"Do you have muscle​ weakness, numbness, or​ tingling?" Muscle​ weakness, numbness, and tingling may indicate a neurologic problem that can be serious. These symptoms require further evaluation or consult. The complaint of ankle​ pain, the decreased​ mobility, and general muscle pain can be treated symptomatically and​ re-evaluated. The muscle​ weakness, numbness, and tingling are more serious and should be further evaluated by a specialist.

The nurse is performing a musculoskeletal system assessment on an adult client. What questions would be appropriate to help the nurse collect data about the client​'s joint ​health? Select all that apply. ​"Do you have swelling or redness in your​ joints?" ​"At what time of day does the pain occur or seem​ worse?" ​"Do you have problems with your​ joints?" ​"Are you married or​ single?" ​"How long have you had pain in your​ joint?"

​"Do you have swelling or redness in your​ joints?" ​"At what time of day does the pain occur or seem​ worse?" ​"Do you have problems with your​ joints?" ​"How long have you had pain in your​ joint?" It is important to establish rapport with the client and determine if joint pain and mobility are a problem. When pain in the joints is​ confirmed, the assessment would focus on when the pain​ occurs, if there is swelling or​ redness, if the pain is new or​ chronic, and other characteristics of the pain. Marital status is unrelated.

What question related to pregnancy and childbearing is important when assessing a client​'s risk for breast​ cancer? ​"Have you gone through​ menopause?" ​"Are your children​ grown?" ​"How long did you bottle feed your​ children?" ​"Do you have​ children?"

​"Do you have​ children?" Female clients who do not have children are at greater risk for breast cancer. The length of time a client bottle fed her children is not​ important, but the length of time the client breastfed influences the risk for breast cancer. Clients who breastfeed for at least a year have a decreased risk for breast cancer. Menopause is related to hormone levels and is not specific to pregnancy and childbearing. The ages of the client​'s children have little relevance to the client​'s risk for breast cancer.

The nurse is examining the lymphatic system of an adult client for the first time. What question would not be relevant to the focused assessment of the lymphatic​ system? ​"Do you notice any swelling in your lymph​ nodes?" ​"Do you have​ headaches?" ​"Do you have pain in your lymph​ nodes?" ​"How long have you experienced pain in your lymph​ nodes?"

​"Do you have​ headaches?" Headaches are not necessarily related to the lymphatic system. Asking about pain in the area of lymph nodes and how long the pain has been present is pertinent. Any swelling the client might notice in the lymph nodes is also pertinent.

The nurse is assessing an elderly adult who is complaining of chest pain. What questions would be helpful in assessing the relationship between the chest pain and ​breathing? Select all that apply. ​"Do you tire​ easily?" ​"How frequently do you​ cough?" ​"How long have you been​ married?" ​"Do you have a cough with this chest​ pain?" ​"If a cough is​ present, when did it​ start?"

​"Do you tire​ easily?" ​"How frequently do you​ cough?" ​"Do you have a cough with this chest​ pain?" ​"If a cough is​ present, when did it​ start?" To determine the extent of the influence of the chest pain on overall breathing and to help diagnose if the pain is cardiac or from some other​ illness, the questions about the chest pain would be related to​ breathing, cough and mucus​ production, and fatigue. The answers would help to assess the cardiac status related to breathing for an adult complaining of chest pain. Asking questions regarding memory are important during a neurological assessment but are not relevant when assessing chest pain.

The nurse is assessing an adult client who complains of chest pain. What questions are the most revealing when assessing the client​'s chest ​pain? Select all that apply. ​"Does the pain travel anywhere​ else?" ​"Does the pain make you sick to your​ stomach?" ​"Can you describe the​ pain?" ​"Are you allergic to​ cats?" ​"Does anything make the pain​ worse?"

​"Does the pain travel anywhere​ else?" ​"Does the pain make you sick to your​ stomach?" ​"Can you describe the​ pain?" ​"Does anything make the pain​ worse?" Chest pain related to heart disease or heart attack can often radiate to​ arms, elbows, or the jaw. It may make the client sick to the stomach or break out in a sweat. It is important to determine if the pain is from an injury and is reproduced with movement versus being constant and unrelieved. And it is important to describe the pain in terms of​ dull, sharp, or aching to help rule out other causes unrelated to the heart. It is important to ask about drug​ allergies, but an allergy to cats is low on the priority list related to chest pain and cardiac disease.

What question for the parents of an​ 8-year-old child would address a social or emotional issue related to the child​'s urinary​ system? ​"Does the child have any redness or swelling in the genital​ area?" ​"Does the child have any kidney​ disease?" ​"Does your child have involuntary urination such as​ bed-wetting?" ​"Does the child play any​ sports?"

​"Does your child have involuntary urination such as​ bed-wetting?" Bed-wetting is an emotional issue for children and impacts the​ social, mental, and physical​ well-being of the child and family. Determining any previous kidney disease opens the conversation with parents and helps to focus the assessment and treatment but is not initially related to an emotional issue for the child. A question about the outward appearance of the genital area helps to focus the exam but is not specific to the urinary system. Whether the child plays sports is not relevant to a social or emotional issue related to the child​'s urinary system.

The nurse is preparing to palpate the client​'s skin during a focused skin assessment. The client asks the nurse why the nurse is touching the skin. What statements by the nurse are correct about palpating the ​skin? Select all that apply. ​"During palpation, I am also trying to determine your skin​'s ​elasticity." ​"While I am palpating your​ skin, I am feeling for any skin​ lesions." ​"I will palpate your skin over your​ forehead, body,​ arms, and legs to determine the temperature of your​ skin." ​"One reason I will touch your skin is to palpate the texture of your​ skin." ​"I am trying to observe the cleanliness of your skin and​ hair."

​"During palpation, I am also trying to determine your skin​'s ​elasticity." ​"While I am palpating your​ skin, I am feeling for any skin​ lesions." ​"I will palpate your skin over your​ forehead, body,​ arms, and legs to determine the temperature of your​ skin." ​"One reason I will touch your skin is to palpate the texture of your​ skin." The nurse would palpate the skin using the dorsal surface of fingers over the​ forehead, body,​ arms, and legs to determine the skin​'s temperature. The nurse would also assess the client​'s skin texture. The texture of the skin should be​ smooth, firm, and even. One reason for palpating the skin is to determine if there are any skin lesions. The skin is pinched together to determine its elasticity during palpation. Observing for cleanliness of skin and hair is a function of​ inspection, not a function of palpation.

The nurse is preparing to assess the client​'s temperature. Which statement by the nurse about temperature regulation requires further teaching from the charge​ nurse? ​"An illness like infection can cause the client​'s temperature to​ increase." ​"Environmental changes in temperature are unlikely to affect the elderly​ client." ​"Infants are highly susceptible to changes in environmental​ temperature." ​"If the client is under a lot of stress their temperature can​ increase."

​"Environmental changes in temperature are unlikely to affect the elderly​ client." The elderly have an increased sensitivity to environmental temperature​ changes, which may be due to a lack of subcutaneous​ fat, lack of​ activity, or inadequate diet. This statement by the nurse indicates a need for further teaching. Infants are highly responsive to a change in environmental temperature and therefore need to be protected from temperature extremes. A central nervous system disorder or an illness can impair the function of the hypothalamus. Hyperthermia is a response to a viral or bacterial​ infection, myocardial​ infarction, malignancy,​ surgery, or trauma. During a stressful incident there is an increase in epinephrine and​ norepinephrine, resulting in an increase in metabolic activity and heat production and thus a rise in core body temperature.

Questions about breast symptoms are intended to determine the significance of symptoms in relation to disease. What questions would be specific to obtaining this ​data? Select all that apply. ​"Have there been any changes in the size or shape of your​ breasts?" ​"Do you have any nipple​ discharge?" ​"Did your insurance company cover your​ mammogram?" ​"Do you have any lumps or thickening in either​ breast?" ​"Do you have any pain or​ tenderness?"

​"Have there been any changes in the size or shape of your​ breasts?" ​"Do you have any nipple​ discharge?" ​"Do you have any lumps or thickening in either​ breast?" ​"Do you have any pain or​ tenderness?" Asking about changes in breast size or shape is​ important, as this can indicate​ pregnancy, lactation, or malignancy. Asking about thickening or lumps is​ pertinent, as these may indicate cysts or tumors. Breast pain can indicate​ pregnancy, lactation, or​ malignancy, and it would be relevant to ask about any pain or tenderness. Asking about discharge from the nipples is​ important, as this can indicate pathology or infection. While a mammogram is important and a client​'s ability to pay for it is also​ important, asking about this is not​ relevant, as it is not related to symptoms.

The nurse is assessing an infant who has no obvious signs of acute illness for respiratory distress and weakness. What questions regarding health history would be important for the nurse to ask when obtaining a history from the ​caregiver? Select all that apply. ​"Have there been any injuries or trauma to the ribs in the last 24​ hr?" ​"Have there been any signs of illness or fever in the last 48​ hr?" ​"Is there a family history of muscle diseases or respiratory​ diseases, for​ example, muscular dystrophy or​ asthma?" ​"Is there a history of muscle weakness or respiratory distress before​ today?" ​"Does the infant have cousins who have been​ ill?"

​"Have there been any injuries or trauma to the ribs in the last 24​ hr?" ​"Have there been any signs of illness or fever in the last 48​ hr?" ​"Is there a family history of muscle diseases or respiratory​ diseases, for​ example, muscular dystrophy or​ asthma?" ​"Is there a history of muscle weakness or respiratory distress before​ today?" If the nurse is seeing this infant for the first time for respiratory distress and weakness without signs of acute​ illness, it would be important to determine if the infant has a congenital disease that has led up to the weakness or if it is an acute onset of symptoms. History of genetic diseases in the​ family, signs of illness in the last 48​ hr, and signs of any injury would be items that pertain to the current symptoms. Information about cousins does not offer any important diagnostic data.

What questions would apply in a focused assessment regarding a urinary ​infection? Select all that apply. ​"Have you had a positive Pap​ test?" ​"Have you ever had a urinary tract​ infection?" ​"Are you having burning or discomfort when you​ urinate?" ​"Have you had recurring infections of the reproductive​ system?" ​"Are you experiencing a change in the color or odor of the​ urine?"

​"Have you ever had a urinary tract​ infection?" ​"Are you having burning or discomfort when you​ urinate?" ​"Are you experiencing a change in the color or odor of the​ urine?" Determining if a client is prone to urinary tract infections gives the interviewer a direction for a focused assessment of the urinary system. A change in the color or odor of the urine and burning or discomfort during urination may indicate a urinary infection. Asking about recurrent infections of the reproductive system could be part of a general assessment but is not relevant when assessing a urinary infection. Results of a Pap test would be part of a focused exam of the reproductive system.

The nurse is performing a general physical on an adult client and notices flaky hair and areas of dry skin around the client​'s forehead. What question by the nurse would establish rapport while performing a more focused assessment of the client​'s ​head? ​"What type of insurance coverage do you​ have?" ​"Do you have a sore​ throat?" ​"Have you had any problems or conditions with your​ scalp?" ​"When was your last dental​ appointment?"

​"Have you had any problems or conditions with your​ scalp?" Asking about conditions and problems with the scalp opens the discussion and allows the client to feel comfortable talking about a potentially embarrassing problem. Asking about the client​'s last dentist appointment would be appropriate when examining the mouth or addressing oral pain or eating but is not pertinent when discussing the scalp. Asking about a sore throat would not prompt the client to talk about the scalp. A more direct question about the scalp would help the client open up and give information about the specific scalp problem the nurse noticed.

The nurse is assessing an adult client with complaints of a new onset of headache. What questions may assist the nurse in doing a focused assessment for this ​client? Select all that apply. ​"Have you had any recent head​ trauma?" ​"Can you describe the​ pain?" ​"When did the headache begin and how long have you had​ it?" ​"Have you ever had a​ migraine?" ​"Have you ever had the​ flu?"

​"Have you had any recent head​ trauma?" ​"Can you describe the​ pain?" ​"When did the headache begin and how long have you had​ it?" ​"Have you ever had a​ migraine?" Discovering when the headache started and how long it has occurred would help assess the severity of the symptoms. Having the client describe the headache pain would help determine the type of headache. Have the client describe whether the pain​ is, for​ example, dull,​ sharp, or stabbing. Determining if the client has had a bump or trauma to the head related to when the pain started would help assess the type of headache the client is experiencing. Knowing if the client has had a migraine and whether the current headache is similar would help the nurse determine the appropriate treatment. Determining if the client has ever had the flu would not be pertinent to the assessment of the current​ headache; however, assessing for signs of illness along with the headache would help focus the assessment.

What question is least relevant when interviewing a client with breast​ tenderness? ​"Have you experienced breast​ trauma?" ​"When was your last​ period?" ​"Are you​ pregnant?" ​"Have you had genetic​ testing?"

​"Have you had genetic​ testing?" Genetic testing would not be discussed with a client who presents with breast tenderness. Breast tenderness is associated with pregnancy and the menstrual cycle and could be the result of recent breast trauma.

The nurse is performing a physical assessment of an adult client. Working to establish rapport with the client about​ weight, what is an appropriate and focused question to pose to the​ client? ​"Do you understand that obese clients die at a young​ age, and you are in that​ category?" ​"Have you had significant or unexplained weight gain or loss over the last 3​ months?" ​"What are you trying to​ do, starve​ yourself?" ​"Do you realize you are grossly​ overweight?"

​"Have you had significant or unexplained weight gain or loss over the last 3​ months?" The question about weight gain or loss is focused and​ nonthreatening, and it helps to establish rapport with the client during the exam. The questions about being grossly overweight and about obese clients dying at a young age are accusatory and may ruin the rapport the nurse is trying to establish with the client. The way the question about dying at a young age is stated is​ unprofessional, as is the question about starving. Questions from the nurse need to be nonthreatening and open ended so that the client feels comfortable being​ honest, and the nurse can gather the data necessary to make an accurate assessment.

The nurse is assessing an older adult client in the wellness clinic. What question would be most important when assessing changes in the hearing of an older​ adult? ​"Do you have a history of ear​ pain?" ​"Have you noticed any difficulty hearing normal conversations​ recently?" ​"Do you take any medications on a daily​ basis?" ​"How many children do you​ have?"

​"Have you noticed any difficulty hearing normal conversations​ recently?" Asking an older client if he or she has experienced difficulty hearing normal conversation recently helps to establish rapport and gives the provider an idea about the​ client's perception of his or her hearing. How many children a client has may help to establish rapport but would not help to assess the current state of the​ client's hearing. Asking about the medications a client takes daily is important and can answer questions about current state of health and possible effects on hearing but is not the best initial question to ask the client. Determining if the client has had ear pain is an important​ follow-up question but is not the best initial question to ask when assessing for hearing changes in the older adult.

What question by the student nurse would indicate to the charge nurse that the student has an appropriate interpretation of the relationship of family history and the client​'s risk for heart​ disease? ​"Have your parents or siblings ever had a heart​ attack?" ​"Do you​ smoke?" ​"What is your current​ weight?" ​"How many siblings do you​ have?"

​"Have your parents or siblings ever had a heart​ attack?" A parent or sibling diagnosed with a heart attack or other cardiovascular disease would increase the risk for the adult client being interviewed. Current weight does not lead the interviewer to connect a relationship of family history to heart​ disease, although obesity may be a risk factor. Smoking is a risk​ factor, but the question does not indicate a relationship between family history and cardiovascular disease. The number of siblings the client has does not indicate a relationship to family history and heart disease.

The nurse is using a stethoscope to listen to bowel sounds of an adult client. What statements indicate the nurse​'s understanding of bowel sounds and their ​significance? Select all that apply. ​"High-pitched, rushing sounds may indicate gastroenteritis and​ diarrhea." ​"Absence of bowel sounds indicates a bowel obstruction or paralytic​ ileus." ​"Slow, sluggish sounds indicate the client has had bowel surgery or has an​ end-stage bowel​ obstruction." ​"Friction rubs indicate abdominal​ masses." ​"Bruit indicates a bowel​ obstruction."

​"High-pitched, rushing sounds may indicate gastroenteritis and​ diarrhea." ​"Absence of bowel sounds indicates a bowel obstruction or paralytic​ ileus." ​"Slow, sluggish sounds indicate the client has had bowel surgery or has an​ end-stage bowel​ obstruction." Fast and​ high-pitched sounds are the result of rapid​ peristalsis, which occurs in gastroenteritis and diarrhea.​ Slow, sluggish sounds may be a result of peristalsis starting again after bowel surgery or slowing from an impending bowel obstruction. No bowel sounds indicates a bowel obstruction or paralytic ileus. Bruit indicates an arterial occlusion and is the sound heard during a vascular assessment of the abdomen. Friction rub is heard over the spleen or the liver and does not indicate a mass.

Mrs. Lang is a​ 57-year-old woman who has come into the clinic for a breast exam. Which of Mrs.​ Lang's statements indicate that she may have an increased risk for breast​ cancer? ​"I have maintained a healthy​ weight." ​"I began menstruation at age​ 14." ​"I have breast​ implants." ​"I am on hormone replacement​ therapy."

​"I am on hormone replacement​ therapy." Hormone replacement therapy has been associated with an increased risk of breast cancer. Menstruation at age 14 is​ normal-females who began menarche before the age of 12 are at a greater risk for breast cancer. Breast implants do not increase the risk for breast cancer. Obesity is a predisposing factor for breast cancer and maintaining a healthy weight helps reduce the risk.

Ms. Klein is a​ 48-year-old woman who came to the clinic because she has been having difficulty walking. Which of her responses may indicate a neurologic​ cause? ​"I am a​ nurse." ​"I take daily​ vitamins." ​"I swim​ regularly." ​"I have a family history of multiple​ sclerosis."

​"I have a family history of multiple​ sclerosis." Multiple sclerosis is a neurologic illness that runs in families and can affect a​ person's ability to walk. While being a nurse indicates that she is on her feet for most of her working​ day, this is not a likely cause of her symptom. Taking daily vitamins and swimming regularly for exercise indicate that she is concerned and proactive about her health and​ wellness, but these activities would not cause walking difficulty.

What client responses indicate that the client has a higher risk for breast​ cancer? Select all that apply. ​"I have had uterine​ cancer." ​"I am still​ menstruating." ​"My sister had breast​ cancer." ​"I have fibrocystic​ breasts." ​"My father has a genetic mutation linked to breast​ cancer."

​"I have had uterine​ cancer." ​"My sister had breast​ cancer." ​"My father has a genetic mutation linked to breast​ cancer." A client with a sister who had breast cancer has a slightly higher risk for breast cancer than normal. A history of uterine cancer in a client raises the risk of getting breast cancer. If the​ client's father has a genetic mutation linked to breast​ cancer, the client potentially has it as well. Menstruation does not increase the risk of getting breast cancer. Fibrocystic breasts do not raise the risk of getting breast cancer but may make it more difficult to detect it.

The nurse is teaching a new nurse about physical assessment while preparing to assess the abdomen of the client during an annual physical exam. Which statement about auscultation by the new nurse indicates understanding of the​ teaching? ​"Percussion lessens bowel​ sounds." ​"When I am palpating the abdomen peristalsis will​ stop." ​"I need to auscultate first because palpation and percussion can stimulate the​ bowel." ​"I should auscultate the client​'s abdomen while the client is​ standing."

​"I need to auscultate first because palpation and percussion can stimulate the​ bowel." During abdominal​ assessment, the nurse inspects and auscultates before percussing or palpating because these actions can stimulate the​ bowel, which can increase bowel motility and heighten bowel sounds. Auscultation is performed when the client is supine. Peristalsis may become more active during palpation. Percussion can increase bowel sounds.

The nurse is preparing to complete a neurologic health history for a client. What statement by the nurse indicates a need for further​ teaching? ​"I should ask the client if​ he/she has ever been diagnosed with a neurologic​ disease." ​"I should not assess the client​'s family history of neurologic​ problems." ​"I should ask the client if​ he/she has any chronic​ diseases." ​"I need to assess the client​'s ability to perform activities of daily​ living."

​"I should not assess the client​'s family history of neurologic​ problems." Some​ diseases/conditions are familial and will appear in different generations. This statement indicates a need for further teaching. Chronic disease can be a predisposing factor to developing later neurologic problems. Neurologic problems can make it difficult to perform activities of daily living. Asking about previous diagnoses of neurologic disease gives the client an opportunity to inform the nurse about any specific diseases the client might have.

The nurse is teaching a student nurse how to document. Which statement by the student nurse indicates an understanding of the​ teaching? ​"I should only chart after I have completed a task or​ assessment." ​"When charting, I will use the​ client's full​ name." ​"I should leave several blank lines for the nurse to chart before I begin my​ charting." ​"To save​ time, I should wait and chart at the end of the​ shift."

​"I should only chart after I have completed a task or​ assessment." The nurse should never​ pre-chart. Charting would only be done after tasks​ and/or assessments are completed. This statement by the nurse indicates an understanding of the principles of documentation. The nurse would not leave a blank space for a colleague to chart later. The nurse would not use the words​ "patient" or​ "client," or the client​'s name when documenting. Charting would be completed as soon as possible after care is provided.

The nurse is being oriented to a​ long-term facility. What statement indicates understanding of client documentation​ requirements? ​"The documentation requirements are the same for​ acute-care and​ long-term care​ facilities." ​"It will be hard to plan documentation needs as they will be unpredictable in the​ long-term care​ facility." ​"I will have to revise the plan of care every 3 months unless there is a change in the clients health​ status." ​"I will have to chart more in the​ long-term care facility than I did in the​ hospital."

​"I will have to revise the plan of care every 3 months unless there is a change in the clients health​ status." The nurse will need to review and revise the plan of care every 3 months or whenever the​ client's health status changes. It is unlikely that the nurse will need to do the same amount of documentation in an acute care facility as in a​ long-term care facility. Older adult clients in​ long-term care facilities tend to have chronic conditions and generally experience​ subtle, small changes in their​ condition, often requiring less documentation. The need for documentation may vary depending on the​ client's condition. Although documentation requirements are usually different between​ acute-care and​ long-term-care facilities, they are usually predictable in​ long-term-care facilities.

The nurse is teaching a new nurse about completing a neurologic health history. What statement by the new nurse indicates a need for further​ teaching? ​"I will need to watch the client​ walk." ​"I need to assess the client​'s ​diet." ​"I should ask if the client has had any previous​ illnesses." ​"When asking about medications I should ask the client about prescription​ medication."

​"I will need to watch the client​ walk." Watching the client walk is part of the physical​ assessment, not the health history. This statement indicates a need for further teaching. A dietary history can provide information regarding a​ well-balanced diet of nutrients and electrolytes needed to maintain a healthy body. Some medications affect the neurologic system. The client may not associate a specific​ illness/disease with the neurologic​ system, so this gives the client a chance to inform the nurse of a disease or illness.

The nurse is preparing to assess the client​'s pain after admission to the medicaldash-surgical unit. Which statement by the nurse indicates the nurse​'s understanding of the importance of pain​ assessment? ​"In order to treat the client effectively I must complete a thorough initial pain​ assessment." ​"I need to assess the client​'s pain at the beginning of each​ shift." ​"If the client does not tell me about the pain I do not need to assess​ it." ​"Pain should only be assessed after surgery or before and after a painful​ procedure."

​"In order to treat the client effectively I must complete a thorough initial pain​ assessment." In an initial pain​ assessment, it is important to ask questions to determine the client​'s health history. Collecting these data enables the nurse to give quality care. Clients of different cultures​ and/or ages may not report pain and the nurse must assess every client for pain. Pain is considered the fifth vital sign and should be part of the routine assessment. The nurse should assess the client​'s pain on admission and as needed throughout the length of stay of the client.

What questions would the nurse ask an adult client to determine risk factors for a future musculoskeletal ​problem? Select all that apply. ​"Is there a history of osteoporosis in your​ family?" ​"Is lifting involved in your​ work?" ​"Have you ever been diagnosed with an allergy to​ penicillin?" ​"Is there a history of arthritis in your​ family?" ​"Do you drink alcohol and how​ often?"

​"Is there a history of osteoporosis in your​ family?" ​"Is lifting involved in your​ work?" ​"Is there a history of arthritis in your​ family?" ​"Do you drink alcohol and how​ often?" Determining risk factors for musculoskeletal diseases like rheumatoid or juvenile arthritis and osteoporosis are important when interviewing an adult client. It is also important to know how much alcohol a client consumes on a daily basis because that can impact nutrition and bone health. It is not important to know about an allergy to penicillin during the assessment of musculoskeletal conditions unless penicillin would be in the treatment plan.

What techniques would the nurse expect to use to complete a cardiovascular exam on an adult​ client? Inspection and auscultation Palpation and auscultation Inspection and palpation ​Inspection, palpation,​ percussion, auscultation

​Inspection, palpation,​ percussion, auscultation The nurse must perform​ inspection, palpation,​ percussion, and auscultation of an adult client to do a complete cardiovascular exam. The inspection would include looking at the face for​ color, carotid artery​ pulsation, external jugular​ veins, and sclera color. Palpation would include feeling for the quality of the pulses in the neck and extremities. Percussion would include tapping to determine if there is density noted in the​ chest, indicating fluid. Auscultation would include listening to the heart at the apex and the base to determine quality and rate of heart sounds.

You plan to educate Mr. Jacobs about a diet to promote general health. What statement would you include in your​ teaching? ​"Over-the-counter supplements are the best way to obtain​ nutrients." ​"A diet primarily consisting of complex carbohydrates is the best​ choice." ​"It is important to drink about​ 1,500 mL of water a​ day." ​"Since you are​ vegetarian, you need to increase your intake of vitamins​ A, D,​ E, and​ K."

​"It is important to drink about​ 1,500 mL of water a​ day." Each person should drink approximately​ 1,500 mL of water a day to promote digestion and hydration. It is best to obtain nutrients from food​ sources, not​ supplements; in​ addition, some supplements are not safe for those with certain health conditions. A vegetarian needs to increase his or her intake of the minerals​ iron, zinc, and​ calcium; these nutrients are usually obtained from animal sources. A healthy diet includes a balance of all types of​ nutrients, including​ carbohydrates, protein,​ fats, and minerals. A focus on just one type can result in undernutrition.

The nurse explains to a new graduate that light palpation is used for certain examinations. Which statement by the​ new nurse indicates understanding of the​ teaching? ​"Light palpation is used to assess​ pulses." ​"The client will not feel light​ palpation." ​"Light palpation is used to determine the shape of the​ abdomen." ​"You examine the position of the liver with light​ palpation."

​"Light palpation is used to assess​ pulses." Light palpation is used for pulses because deep palpation compresses the artery and prevents the nurse from feeling a pulse. Light palpation is used for superficial​ examination; determining the position of the liver requires deep palpation. The client feels light​ palpation, but it is not uncomfortable. The shape of the abdomen is determined by visual inspection.

The charge nurse is explaining to student nurses the general purpose of lymph nodes in the body. What statements about lymph nodes are ​correct? Select all that apply. ​"Lymph nodes produce​ lymphocytes." ​"Lymph nodes provide a​ defense." ​"Lymph nodes are not commonly​ assessed." ​"Lymph nodes produce​ antibodies." ​"Head and neck lymph nodes are the most commonly​ assessed."

​"Lymph nodes produce​ lymphocytes." ​"Lymph nodes provide a​ defense." ​"Lymph nodes produce​ antibodies." ​"Head and neck lymph nodes are the most commonly​ assessed." Head and neck lymph nodes are the easiest ones to palpate and assess. The assessment can be correlated to the complaints of the client during a focused exam. Lymph nodes provide a defense against the invasion of foreign substances like viruses and​ bacteria, produce lymphocytes to help fight​ infection, and produce antibodies that help fight​ infection, viruses, and illness. Lymph nodes in the head and neck are commonly assessed in any​ head-to-toe physical exam and when clients complain of feeling fatigued or ill or of having a sore throat.

What statement about male breast cancer is not​ true? ​"Male breast cancer is​ rare." ​"Cirrhosis and estrogen medications are predisposing factors for male breast​ cancer." ​"Radiation exposure can cause male breast​ cancer." ​"Male breast cancer occurs in response to maternal​ hormones."

​"Male breast cancer occurs in response to maternal​ hormones." Gynecomastia in an infant occurs in response to maternal​ hormones; it is not the cause of male breast cancer. Male breast cancer is rare but can occur if there is a maternal history of breast cancer. Radiation​ exposure, cirrhosis, and estrogen medications can be predisposing risk factors and increase the risk of breast cancer in males.

The nurse is caring for an​ 85-year-old client with pain. Which statement by the nurse indicates a need for further education of the nurse regarding pain management of elderly​ clients? ​"The older adult client may display lethargy and anorexia as signs of​ pain." ​"The older client may be fearful of becoming dependent on pain​ medications." ​"As clients age they may experience decreased sensations of​ pain." ​"Pain is a natural part of the aging​ process."

​"Pain is a natural part of the aging​ process." Older adults may perceive pain as part of the aging process. This statement by the nurse indicates a need for further teaching of the​ nurse, as pain is not part of the aging process but a perception of older adults. Older clients may experience decreased sensations of pain and display​ lethargy, anorexia, and fatigue as indicators of pain. Older clients may withhold complaints of pain due to a fear of treatment or becoming dependent.

The charge nurse is teaching the student nurse about the musculoskeletal assessment of plane joints. What statements by the student nurse indicate an understanding of plane ​joints? Select all that apply. ​"Plane joints can be found between the cervical​ vertebrae." ​"Plane joints are the same as hinge​ joints." ​"Plane joints allow slipping and gliding​ movements." ​"Plane joints are located in the intercarpals and intertarsals of the hands and​ feet." ​"Plane joints are synovial​ joints."

​"Plane joints allow slipping and gliding​ movements." ​"Plane joints are located in the intercarpals and intertarsals of the hands and​ feet." ​"Plane joints are synovial​ joints." Plane joints have a flat articular surface with a​ fluid-filled cavity at the point of the joint. This allows for slipping and gliding movements such as in the intercarpals of the hand and the intertarsals of the foot. Plane joints are also noted in the articular processes of the ribs. They would not be found between the cerviceal vertebrae.

The charge nurse is teaching a new nurse about the body​'s regulation of respirations and respiratory assessment. Which statement by the new nurse indicates a need for further teaching by the charge​ nurse? ​"Respiratory rate is the continuous exchange of oxygen and carbon dioxide in the​ lungs." ​"The chest wall effortlessly rises and falls with each​ breath." ​"In order for the body to have adequate oxygenation the body needs a healthy and intact respiratory​ system." ​"To accurately assess the respiratory rate I should not tell the client I am assessing their respiratory​ rate."

​"Respiratory rate is the continuous exchange of oxygen and carbon dioxide in the​ lungs." The respiratory rate is the measurement of breathing cycles​ (inspiration and​ expiration) in one minute.​ One key factor in the body​'s ability to regulate respirations is having a healthy and intact respiratory system. The chest wall rises and falls symmetrically with each equally spaced breath. To obtain an accurate respiratory rate do not mention to the client that you are assessing their respiratory rate. It can be assessed while the nurse is palpating the radial pulse.

The nurse is assessing a young adult with scoliosis. What statements indicate that the nurse understands the complexity of ​scoliosis? Select all that apply. ​"Scoliosis would be suspected if a client has a notable elevation of one​ scapula." ​"Scoliosis will cause mental delay if​ undiagnosed." ​"Scoliosis is a lateral deviation of the​ spine." ​"Scoliosis is related only to​ dwarfism." ​"The lung capacity of a client with scoliosis can be altered depending on the severity of the​ curvature."

​"Scoliosis would be suspected if a client has a notable elevation of one​ scapula." ​"Scoliosis is a lateral deviation of the​ spine." ​"The lung capacity of a client with scoliosis can be altered depending on the severity of the​ curvature." Scoliosis is recognized by a lateral deviation of the spine. One scapula would be​ elevated, and the client can experience decreased lung capacity. Scoliosis is not limited to clients with dwarfism nor is it related to those with mental challenges. The curvature of the spine can cause​ pain, immobility, and respiratory compromise.

What statement by the nursing student indicates​ his/her understanding of the function of the Bartholin​ glands? ​"The Bartholin glands are located inside the anal opening and lubricate the​ rectum." ​"The Bartholin glands have no function but get infected with sexually transmitted​ infections." ​"The Bartholin glands produce mucus and lie next to the vaginal​ opening." ​"The Bartholin glands are located at the cervix and secrete fluid during sexual​ intercourse."

​"The Bartholin glands produce mucus and lie next to the vaginal​ opening." The Bartholin glands lie next to the opening of the vagina and secrete mucous fluid to lubricate the vaginal opening and make sexual intercourse more comfortable. The Bartholin glands are not located at the cervix or in the anal opening.

Which statements indicate that the nurse correctly interprets the relationship between the electrical impulses and the ECG tracing in the cardiac ​cycle? Select all that apply. ​"The P wave is the first wave of the​ complex." ​"The QRS complex represents the contraction of the ventricles in the​ cycle." ​"The T wave is the ventricular​ depolarization." ​"The PR interval is not important in the cardiac​ cycle." ​"The QT interval represents the time it takes for ventricular contraction in one​ cycle."

​"The P wave is the first wave of the​ complex." ​"The QRS complex represents the contraction of the ventricles in the​ cycle." ​"The QT interval represents the time it takes for ventricular contraction in one​ cycle." The P wave indicates that the impulse is firing from the SA node. The QRS complex indicates the contraction of the ventricles. The QT interval indicates the time it takes for ventricular contraction. The PR interval indicates the time it takes for the signal to reach the AV node and it is important to an effective cardiac cycle. The T wave indicates the ventricular repolarization.

The nurse is assessing a child. What statement by the nurse indicates the correct interpretation of the auscultation assessment of the​ child's heart? ​"Auscultating heart sounds of a child are not accurate because of​ movement." ​"The S2 sound is more prominent at the apex of a​ child's heart." ​"The sounds of a​ child's heart are best auscultated from the​ back." ​"The S1 sound will be louder at the apex of the heart in a​ child."

​"The S1 sound will be louder at the apex of the heart in a​ child." The anatomy of the normal heart would be in the same area for children and adults in proportion to size and body build. The nurse can expect to auscultate the S1 and S2 sounds in the same areas of the apex and base of the heart for a child and an adult. The nurse can also expect the S1 sound to be louder than the S2 sound when listening to the apex of the heart. Assessment of the child would be best done from the front where the apex of the heart can be located and auscultated.

The charge nurse has completed a review of the flow of blood through the heart with a nursing student. What statement indicates that no further teaching is​ necessary? ​"A one-way valve connects the right and left​ atria, through which the blood is pumped into the​ lungs." ​"The blood is pumped from the right side of the heart to the lungs and the left side of the heart to the body​ simultaneously." ​"The mitral valve connects the right and left ventricles and allows blood to flow to the​ body." ​"The blood travels from the right side of the heart to the left side of the heart before it goes to the​ lungs."

​"The blood is pumped from the right side of the heart to the lungs and the left side of the heart to the body​ simultaneously." The blood travels from the right atrium through the tricuspid valve into the right ventricle and to the lungs. The blood comes back into the left atrium through the mitral valve into the left ventricle and is pumped out to the body through the aorta. The right and left atria and right and left ventricles are not normally connected. The tricuspid valve connects the right atria and right​ ventricle, and the mitral valve connects the left atria and the left ventricle.

The nurse is assessing a client​'s level of pain. The client had minor surgery earlier in the day and describes the pain level at 4 on a scale of 1 to​ 10, and denies the need for analgesia. Which statement best describes this client​'s response to​ pain? ​"This client is experiencing​ hyperalgesia." ​"This client has a low pain​ tolerance." ​"The client has a high pain tolerance​ level." ​"The client has a low pain​ threshold."

​"The client has a high pain tolerance​ level." The client with a high pain tolerance is willing to experience greater amounts of pain without requiring pain relief. The client with a low pain threshold would request relief with minimal painful stimuli. Hyperalgesia is a heightened response to painful stimuli. The client with a low degree of pain tolerance would most likely request medication for relief of pain.

The nurse is assessing the abdomen of an adult. Which statements indicate the nurse​'s understanding about the anatomy and physiology of the ​abdomen? Select all that apply. ​"The digestive organs include the alimentary​ canal." ​"The reproductive organs are part of the abdominal​ assessment." ​"The ribs are assessed when assessing the​ abdomen." ​"The assessment provides data about​ digestion." ​"The urinary system is included in the​ assessment."

​"The digestive organs include the alimentary​ canal." ​"The assessment provides data about​ digestion." The abdominal assessment includes all the digestive​ organs, including the alimentary​ canal, and gives data about the digestive health of the client.​ The ribs and the sternum are part of the thoracic cavity and are not included in an abdominal assessment. The reproductive organs and the urinary system are part of the genitourinary​ assessment, not the abdominal assessment.

When interviewing and assessing an adult​ client, the nurse explains the anatomy of the lungs and why auscultation of all fields will be done. What would be appropriate for the nurse to include in client education before the ​exam? Select​ all that apply. ​"The right lung has three​ lobes: an​ upper, a​ middle, and a lower​ section." ​"Oxygen is supplied to the body through​ exhalation." ​"The left lung has only two​ lobes: an upper and a lower​ section." ​"It is important to avoid smoking to keep the respiratory system​ healthy." ​"One respiratory cycle is one inspiration followed by one​ expiration."

​"The right lung has three​ lobes: an​ upper, a​ middle, and a lower​ section." ​"The left lung has only two​ lobes: an upper and a lower​ section." ​"It is important to avoid smoking to keep the respiratory system​ healthy." ​"One respiratory cycle is one inspiration followed by one​ expiration." To gain the​ client's cooperation during an​ exam, it is important to explain the procedure and to educate the client on wellness. If a client understands that there are three lobes on the right and two on the​ left, he/she will then understand the purpose of the exam. Information on smoking and on the respiratory cycle is part of wellness education and can be used to initiate conversation and questions from the client during the exam. Oxygen is not supplied to the body through exhalation.

The nurse is preparing to assess the client​'s skin. The client asks the​ nurse, "What does the skin​ do?" What reply indicates a need for further teaching for the​ nurse? ​"The skin does not help regulate the body​'s ​temperature." ​"The skin is responsible for synthesizing vitamin​ D." ​"The skin helps with personal​ identification." ​"The skin helps protect against loss of water and​ electrolytes."

​"The skin does not help regulate the body​'s ​temperature." One of the major functions of the skin is to regulate body temperature. Other functions of the skin are to protect against the loss of water and​ electrolytes, to synthesize vitamin​ D, and to allow for identification through the uniqueness of facial​ contours, skin and hair​ color, and fingerprints.

The nurse is teaching a class for student nurses about effective pain control. Which statements by the students indicate an understanding of the ​teaching? Select all that apply. ​"A-delta fibers transmit​ dull, aching​ pain." ​"The thalamus sends a signal to the cortex so that pain perception​ begins." ​"C fibers transmit sharp pain​ impulses." ​"Pain impulses are transmitted directly to the​ brain." ​"Pain impulses travel from the site of injury to the spinal cord​ first."

​"The thalamus sends a signal to the cortex so that pain perception​ begins." ​"Pain impulses travel from the site of injury to the spinal cord​ first." Pain impulses travel first from the site of injury to the spinal cord. The thalamus sends a signal to the​ cortex, where pain perception takes place.​ A-delta fibers transmit​ sharp, not​ dull, pain sensations. C fibers transmit​ dull, aching​ impulses, not sharp pain. Pain impulses travel from the site to the spinal cord and ascend to the thalamus and then to the​ cortex; they do not transmit directly to the brain.

The nurse is preparing to assess the client​'s temperature. Which statements by the nurse indicate the nurse​'s understanding of temperature ​assessment? Select all that apply. ​"The tympanic temperature is taken with an electronic thermometer using an infrared​ probe." ​"The axillary method of measuring temperature is the most accurate​ method." ​"Not all clients are candidates for assessing temperature via the oral​ method." ​"The temporal artery thermometer is quick and scans over the temporal artery on the​ forehead." ​"A rectal temperature may be obtained if the client is​ comatose, confused, having​ seizures, or is unable to close his or her​ mouth."

​"The tympanic temperature is taken with an electronic thermometer using an infrared​ probe." ​"Not all clients are candidates for assessing temperature via the oral​ method." ​"The temporal artery thermometer is quick and scans over the temporal artery on the​ forehead." ​"A rectal temperature may be obtained if the client is​ comatose, confused, having​ seizures, or is unable to close his or her​ mouth." The oral temperature is one of the most​ accessible, convenient, and accurate methods for assessing​ temperature; however, not all clients are candidates for this method. Examples of clients who are not candidates for using the oral method of temperature assessment include those who are at risk for seizure and those who are unable to keep their mouth closed. Oral temperatures are obtained by using an electronic or digital probe device in a hospital setting. Glass mercury thermometers are no longer used in the clinical setting. The tympanic temperature is taken with an electronic thermometer using an infrared probe. Placing the probe in the client​'s ear gives a core temperature quickly and accurately.​ The temporal thermometer is noninvasive and scans over the forehead across the temporal artery. This convenient method displays the temperature within seconds. A rectal temperature may be obtained if the client is​ comatose, confused, having​ seizures, or is unable to close his or her mouth. The axillary method is one of the safest methods of measuring​ temperature, as it is less invasive compared to the oral or rectal​ route, but it is also considered the least accurate due to the variety of possible probe positions.

The nurse is caring for a​ 45-year-old client who is in the clinic for an annual checkup. The client​'s blood pressure reading is​ 139/82 mmHg. Which statement by the nurse indicates the nurse​'s correct understanding of this client​'s blood​ pressure? ​"The client​'s blood pressure is within the normal range for an​ adult." ​"This client​'s blood pressure reading is​ low." ​"This client is experiencing​ pre-hypertension and will need close​ monitoring." ​"The client is hypertensive and will be placed on blood pressure​ medications."

​"This client is experiencing​ pre-hypertension and will need close​ monitoring." Blood pressure of 120dash-139 mmHg systolic​ and/or 80dash-89 mmHg diastolic is classified as​ pre-hypertension.Therefore, the blood pressure is not considered to be within the normal range for this​ client, nor is it considered low. Since the​ client's blood pressure is considered​ pre-hypertensive, blood pressure medications will not be prescribed at this time.

The nurse is caring for a client with neuropathic pain from peripheral neuropathy associated with diabetes mellitus. Which statement by the client indicates the client​'s understanding of the​ pain? ​"This type of pain is caused by damage to my​ muscles." ​"I should be prepared to experience nausea with this​ pain." ​"The pain is a result of a broken leg I had as a​ child." ​"This type of pain is due to my​ diabetes."

​"This type of pain is due to my​ diabetes." Neuropathic pain is often manifested as peripheral neuropathy in clients with diabetes mellitus. Muscle pain is somatic pain. The client with visceral pain may experience nausea. Skeletal pain is classified as somatic pain.

When auscultating a client​'s ​abdomen, which of the following sounds indicate normal bowel​ sounds? ​Loud, high-pitched, and rushing sounds ​Rough, grating sound Absent bowel sounds ​Irregular, gargling, and​ high-pitched sounds

​Irregular, gargling, and​ high-pitched sounds Normal bowel sounds are​ irregular, gargling, and high pitched. They occur from 5 to 30 times per minute.​ Loud, high-pitched, and rushing sounds indicate hyperactive bowel​ sounds, which may occur more frequently with gastroenteritis or diarrhea. Absent bowel sounds may indicate a paralytic ileus. A​ rough, grating sound indicates friction​ rub, which is caused by organs rubbing together or an organ rubbing on the peritoneum.

The nurse is assessing an adult client with complaints of neck pain. What questions would be appropriate when assessing the neck pain of a ​client? Select all that apply. ​"What other symptoms are you having with the neck​ pain?" ​"How long have you had flaking of the​ scalp?" ​"Were you experiencing any stress at the time the neck pain​ began?" ​"Where is the location of your neck​ pain?" ​"Does anything make the neck pain​ worse?"

​"What other symptoms are you having with the neck​ pain?" ​"Were you experiencing any stress at the time the neck pain​ began?" ​"Where is the location of your neck​ pain?" ​"Does anything make the neck pain​ worse?" The location of the pain may indicate the source of the problem. Symptoms of fever or illness with neck pain can be indicative of meningitis. Specific movements that make the pain worse can help determine the cause of the neck pain. Stress can cause neck pain and stiffness. Scalp flaking is related to conditions of the scalp and not to neck pain.

What questions would be helpful in gathering subjective information about a client​'s breast ​health? Select all that apply. ​"When was the date of your last​ period?" ​"Have you ever had breast​ surgery?" open double quoteHave you ever had an intrathoracic infection such as ​pneumonia?close double quote ​"Can you describe how your breasts feel​ today?" ​"Do you have a history of breast​ disease?"

​"When was the date of your last​ period?" ​"Have you ever had breast​ surgery?" ​"Can you describe how your breasts feel​ today?" ​"Do you have a history of breast​ disease?" Asking about how the breasts feel opens the discussion about breast health. Asking about the date of the client​'s last menstrual period helps provide information about breast changes that occur with hormonal changes during menstruation. Asking about a history of breast disease is important as breast disease may increase the client​'s risk of breast cancer. Asking about breast surgery is relevant as it may have physical and psychologic implications. Having had pneumonia is not as relevant to breast health as other questions. Breast changes from childbearing may appear the same whether the client has one child or several.

A new client tells the nurse about crusts and scales on the arms and legs from eczema. What is the best question for the nurse to ask in​ follow-up? ​"Does anything​ help?" ​"When were you diagnosed with the​ problem?" ​"Do you have an eczema infection​ now?" ​"What is the name of the doctor who diagnosed the​ problem?"

​"When were you diagnosed with the​ problem?" The best​ follow-up question​ is, "When were you diagnosed with the​ problem?" This allows the client to tell the history and provide needed nursing information regarding onset. The diagnosing health care provider​'s name is irrelevant. Asking about an infection might suggest that the nurse does not know what eczema is and might decrease client confidence in the nurse. The question​ "Does anything​ help?" indicates that the nurse lacks knowledge of the condition and its treatment.

Ms. Mason is a​ 38-year-old woman who has come into the clinic for a genitourinary examination. After the exam you determine she has hematuria. What are her​ symptoms? Weight gain ​Bloody, foaming, or dark yellow urine Breakthrough bleeding ​Pain, burning, and discomfort while urinating

​Bloody, foaming, or dark yellow urine Hematuria is indicated by​ bloody, foaming, or dark yellow urine and requires further evaluation by a health care provider. Weight gain can indicate an increase in fluid retention.​ Pain, burning, and discomfort while urinating indicates the presence of an infection. Breakthrough bleeding​ (bleeding outside normal menstrual​ bleeding) can be due to a hormonal imbalance.

Mrs. Radcliffe has come to the clinic with abdominal pain. You are palpating for rebound tenderness. She experiences a​ sharp, stabbing pain as the compressed area returns to its uncompressed state. What is this​ called? ​Murphy's sign Pulsating aorta Psoas sign ​Blumberg's sign

​Blumberg's sign Blumberg's sign is the experience of​ sharp, stabbing pain as the compressed area returns to its normal state. This occurs when there is peritoneal irritation. A positive​ Murphy's sign is indicated when there is sharp abdominal pain upon palpation of the liver and indicates cholecystitis. Psoas sign occurs when there is abdominal pain upon contraction of the psoas​ muscle, indicating peritoneal inflammation or appendicitis. A pulsating aorta would be discovered upon palpation of the aorta and would indicate an aneurysm.

What abnormal finding would be discovered using the Rinne​ test? Hearing sounds better in one ear than the other ​Bone-conducted sound heard as long as or longer than​ air-conducted sound Inability to repeat phrases spoken by you Inability to maintain balance

​Bone-conducted sound heard as long as or longer than​ air-conducted sound An abnormal finding on the Rinne test is when the​ bone-conducted sound of a tuning fork is heard as long as or longer than the​ air-conducted sound. This may indicate that the client has conductive hearing loss. The inability to repeat phrases spoken by you may indicate a loss of the ability to hear​ high-frequency tones and is assessed using the whisper test. The Weber test is used to determine if a client hears sounds better in one ear than the​ other, which could indicate poor conduction or nerve damage. An abnormal response on the Romberg​ test, in which the client is unable to maintain balance or must put feet farther apart in order to maintain​ balance, would indicate that the client has a problem with the function of the vestibular apparatus.

What techniques would be used when assessing the thyroid gland of an adult​ client? Palpation and auscultation Percussion and auscultation ​Inspection, palpation, auscultation Inspection and palpation

​Inspection, palpation, auscultation A complete assessment of the thyroid gland includes inspection and palpation of the gland and auscultation to assess if a bruit is present.​ Normally, a bruit is not present. Percussion is not part of the assessment of the thyroid gland.

A mother is delivering a premature infant with a gestation of 32 to 33 weeks. Some respiratory difficulty at birth is expected due to prematurity. What​ technique(s) would be used to assess a newborn​'s lungs that would also be used to assess the lungs of a​ child, adult, or older​ adult? Inspection ​Inspection, palpation,​ auscultation, percussion Palpation and auscultation Palpation

​Inspection, palpation,​ auscultation, percussion Inspection is used to assess the chest of a client of any age. Palpation is used to assess the rise and fall of the chest wall. Percussion is used to assess for the presence of fluid versus air in the lungs and is done in a gentle manner for an​ infant, but the technique is the same for clients of any age. Auscultation is used to listen to the lung fields. Palpation or inspection alone is not the only way to assess an infant​'s lungs or respiratory status. Palpation and auscultation are only part of the full exam of an infant newborn​'s lungs.

Identify the type of thyroid disorder based on the symptom. ​Instructions: Use the dropdown menus in the left column to select the type of thyroid disorder for each symptom in the right column. Thyroid Disorder Symptom Hypothyroidism Hyperthyroidism ​Irritability/nervousness Fatigue Heat sensitivity Constipation Muscle weakness or tremors Weight gain Enlarged thyroid Memory impairment Amenorrhea Weight loss Intolerance to cold Insomnia Visual disturbances

​Irritability/nervousness- hyper Fatigue- hypo Heat sensitivity- hyper Constipation- hypo Muscle weakness or tremors- hyper Weight gain- hypo Enlarged thyroid- hyper Memory impairment- hypo Amenorrhea- hyper Weight loss- hyper Intolerance to cold- hypo Insomnia- hyper Visual disturbances- hyper

Upon inspection of a client​'s ​breasts, you notice what appears to be Paget disease on one side. What would be the​ observation? Flattening of the breast in one area Retraction of the nipple ​Red, scaly,​ eczema-like area over the nipple Peau d​'orange

​Red, scaly,​ eczema-like area over the nipple Paget disease appears as a​ red, scaly,​ eczema-like area over the nipple. Peau d​'orange is a dimpling of the skin due to edema from blocked lymphatic drainage. Flattening of the breast in one area may indicate a growth or inflammation. Retraction​ (or inversion) of the nipple can be a sign of malignancy.

The nurse is working in an acute care facility when a visitor reports smoke and flames in the family waiting area. The facility uses the acronym RACE to help the staff remember what to do in case of fire. What actions would the nurse take according to​ RACE? ​Rescue, activate​ alarm, confine, extinguish ​Run, alarm,​ call, exit ​Rapid, all​ clients, contain, exit ​Rescue, apply,​ call, extinguish

​Rescue, activate​ alarm, confine, extinguish The nurse would rescue any clients in immediate​ danger, activate the alarm and call the emergency extension to report the fire​ details, confine the​ fire, and extinguish the fire. The other choices are not the accepted universal acronym of RACE used for fire safety.


Related study sets

Chapter 2 - Statistics and APA Ethical Guidelines

View Set

Chemistry: Heterocyclic compounds

View Set

CH 11 Aggregate Output, Price level and the Interest Rate

View Set

chapter 23 vocabulary (Emergency Medical Procedures) study guide

View Set

Business Intelligence Systems Ch.1

View Set

The Nursing Curriculum in the Information Age

View Set

Chemistry - Electrolytes and Osmolality

View Set